You are on page 1of 53

134 'f Section l Anetomic Sciences Section I Anat;;,mic Scier.

ces y 135

a. The peripheral cells are called the tro- (2) Connective tissue derivatives: bone,
phob/asr, with the b!astocele being the cartilage, blood, dentin, pulp, cemen- SAMPLE 0UESTIONS
cavity. tum, periodontal ligament.
b. This hollow ball of cells develops an 3. Endoderm 1. The lateral plerygoid muscle attaches to which 9. Aft of the follou;ing are found in the 1>osterior
inner ceH mass against one waH. a, Forms: of the following? triangle of the neck except one. Which one is
c. The small mass enlarges to form an (I) Gastrointestinal (GI) tract epithe- A. Lateral surface of the lateral pterygoid plate, the exception?
B. Medial surface of the lateral pterygoid plate. A. External jugular vein.
area separating two small cavities. lium and associated glands.
C. Lateral surface of the media) pterygoid plate, B. Subclavlan vein.
(I) This layer is called the embryonic F. Development ol human tissues D. Medial surface of the medial pterygoid plate. C. Hypogtossal nerve.
disc. I. Epithelial structures and derivatives E, Pyramidal process of palatine bone. D. Phrenlc- nerve.
d. Cells lacing the smaller cavity form the a. Mesoderm 2. Which of the following muscles is responsible E. Brachiat ple).us.
primordial ectoderma! layer, and cells (!) Forms the dermis of the epithelium for the formation of the posterior tonsillar 10, Deoxygenated blood from the transverse sinus
lacing the larger one form the primor- and the visceral mesoderm that pillar? drains into tht
dial endodermal layer. covers the yolk sac and subse- A. Stylopharyngeu•. A. Inierlor ti:agittal sinus
5. Yolk sac-implantation of the blastocyst quently becomes the GI tract. B. Tensor veli palatine. B. Confluence of sinuses
causes breakdown of endometriai tissue. b. Ectoderm C. Palatog}ossus, C, Sigmoid sinus
a. This provides food and materials for (I) Initially, the embryo is covered by a D. Pa!atopharyngeus. D. Straight sinus
the embryo for a few weeks. single layer of ectodermal cells. £, Levator veU palatine. E. Internal jugular vein
3. The superior and inferior ophthalmic veins 11. The oestigial cleft of Rathke's pouch in the
b. The embryo sends out branches of (2) By 11 to 12 weeks, this thickens into
drain into the hypophysis is located between the _ _ _.
umbilical arteries and veins. four layers.
A Internal jugular vein A. Anterior and posterior iobes
c. The yolk supplies nutrition to the (3} Later melanocytes invade and pig- B. Pterygoid plexus B. Anterior lobe and hypothalll.mus
embryo through the vitelline arteries. ment the skin. C. Frontal vein C Posterior lobe and hypoth21lamu,
d. At the same time, the placenta and its 2. Nervous system D. lnfraorbitai vein D. Me<ltan eminence and the optic chiasm
blood supply are forming. a. Folds arise in the neural plate in the £. Facial vein 12. Involution of the thymus ,.vould occur following
e. When the yolk sac becomes depleted, third week. 4. The masseter originates from the _ _ _. which yet1r in a healthy individual?
the placenta takes over supplying nutri- (1) The fold facing the smaller cavity will A Condyle of the mandible A. Oyears (al birth).
ents and removing waste. show three areas that will form the B. fnfratemporal <::rest of the sphenoid bone B. 12th year.
f. Five weeks after implantation, the umbil- forebrain, midhrain, and hindbrain. C. Inferior border of the eygomatic arch C. 2llth year.
ical and placental system is in place. b. The forebrain curls down toward the D. Pyramidal process of the palatine bone D. 60th year.
g. Nutrients and wastes cross, but mater- chest as the cranial nerves develop. E. Mastoid process of remporal bone 13. Blood from the internal carotid arterv reaches
5. Which of the following muscle..:. adtlucts the the posterior cerebral artery by the_·_ _,
nal and fetal blood do not mix. 3. Connective tissue
vocal cords? A. Anterior cerebral artery
6. Extraembryonic coelem-at 9 or JO days, a. Develops from somites as fibroblasts A. 1.,ateraJ cricoarytenoid, B. Anterior communlcatmg artery
the trophoblast portion of the blastocyst migrating from either side of the neural B. Posterior cricoarytenoid. C. Posterior communirllting artery
produces mesob!asts, which begin to fill tube. C. Cricothyroid. D. Posterior superior cerehfllar artery
the blastocyst cavity. 4. Cartilage and bone-cartilage cells arise D. Vocalis. £. Basilar artery
a. The blastocyst grows faster than these from the sclerotome and migrate to sur- E. Tensor veH palatine. 14. The infraorbital nerve is a branch of the
cells can fill the space, so they end up round the notochord and spinal cord. 6. Which of the following strata of oral epithelium
lining the cavity around the actual a. The skeleton forms in a segmental pat- is engaged in mitosis? A. Optic nerve
embryo. tern. A. Basale. B. Oculornotor nerve
h. These cells will form the amnion, b. Cartilage grows both by apposition and B. Grnnulosum, C. Ophthalmic nerve
C. Corneum, D. Maxillary nerve
vitelline sac, and chorion. interstitial growth.
D. Spinosum. E, Mandibular nerve
E. Primordial layers c. Bone replaces cartilage through a
7. The auriculotemporal nerve encircles which of 15. Which of the foflawing cells are capal>le of
I. Ectoderm process called endochondral bone the following vessels? mitosis?
a. Forms: development or by intramembranous A. Maxillary artery. A. Smooth muscle.
(1) Nervous system. bone lormation. R Superficial temporal artery. B. Skeletal muscle,
(2) Sensory epithelium of eye, ear, nose. 5. Muscle C. Deep auricular artery, C. Cnrdfac muscle.
(3) Epidermis, hair, nails. By the tenth prenatal week. myoblasts D. Middle meningeal artery. D. Type I pneunmcytes.
( 4) Mammary and cutaneous glands. migrate from the myotome. E. Anterior tympanic artery, E. Neurons.
(5) Epithelium of sinuses, oral and 6. Cardiovascular system-originates from 8. The muscle that is found in the walls of the 16. Which of the following types of epithelium lines
nasal cavities, intraoral glands. angioblasts, which arise from mesoderm heart is characterized by _ _ _, acimar units of salivary glands?
(6) Tooth enamel. in the walls of the yolk sac. A. A peripherally placed nucleus A. Simple squamot1s.
R Multiple nuclei B. Stratified squamous,
2. Mesoderm-arises between the ectoder- a. For the first few weeks, nutrition is pro-
C Intercalated discs C. Simple cuboidal.
rnal and endodermal layers of the embry- vided through the vitelline vascular
D. Fibers with spindle-shaped cells D, Simple columnar.
onic disc. system. £, Pseudostratifierl columnar,
a. Forms: b. The heart starts pumping in the fourth
(1) ;\1uscles. week
1 3 6 v Section I AnatomJc Sci,wces Section l Anatomic: Scienc'l.:t T t 37

17. To which of the following hones is the tensor 23. You decide to take a radiograph of the 30, There are _ _ pairs of true rib.s. 38. Terminal bronchioles are characterized by
tympani attached? fractured tooth. On the first film you miss the
A. Four cells.
A. lncus. apex of the tooth, so you decide to take another fl, Five A. Goblet
B. Malleus. radiograph. Relaxation or which of the C. Seven B. Ciliated cuboidat
C. Stapes. patient's muscles would help you in taking the D. Eleven C. NonciUated -cuhnidrd
D. Hyoid. second film? E Twelve D. Ciliated squamous
E. Mandible A. Geniohyoid. 31. _ _ vertebrae are characterized by a heart- E. Nonclliated squa1n0us
18. In mature dentin> the ratio of inorganic to B. Stylohyold. shaped body. 39. The most superficial layer of the epidermis is
organic matter is approximately _ _. C. Mvlohvold. A Cervical the stratum _ _ .
A. 94:6 D. L~vat~r vell palatine. B. Thoracic A. Spinosum
B. 50:50 E. Palatopharyngeus. C. Lumbar B. Basale
C. 70:30 24. On further examination, you determine that the D. Sacral C, Grnnulosurn
D. 80:20 articular disc of the patient's temporo- E. Coccygeal D. Lucidum
E. Gll:40 mandibular joint has been displaced. If the 32. The sternal angle between the manubriam and E. Corneum
]9. Which of the following cells forms the myelin patient contracts his lateral pterygoid muscle, the sternum marks the position of the _ _ rib. 40. Langerhans cells are located primarily in
sheath around myelinate<l nerves in the central the disc will move _ _ _. A. First stratum
nervous system? A. Posteriorly and medially B. Second A, Corneum
A. Schwann ceHs, B. Anteriorly and medlaUy C. Third B. Luddum
B, Astrocytes. C. Posteriorly and laterally D. Fourth C Granulosum
C. Microglia. D. Anteriorly and laterally E. Fifth D. Spinosum
D. Oligodenc!rocytes. 25. During the examination, the patient observes 33. Which muscle of the anterolateral abdominal £. Ba,ale
E. Amphlcytes. that he cannot feel it when you touch part of wall is described as being belt• or strap-like? 41. Arterfovenous anastomo.ves in deeper skin are
20. Which of the fol/awing nerves supplies taste his cheek and his upper lip. Which of the A External obHque muscle. important in _ _.
se1tsation to the anterior two.thirds of the following nerves was probably damaged during B. Internal oblique muscle. A. tmmunity
tongue? the accident? C. Transversus abdominis muscle. B. Thermoregulatlon
A. Hypoglossal. A. Lingual. D. Rectus abdominis muscle. C. Controlling the arrector pili muscle
B. Glossopharyngeal. B. Maxillary. E. Quadratus lurnhorum muscle. D. Pigmentation
C. Lingual. C. Long buccaL 34. In addition to the esophagus itself, which of the E. Pain sensation
D. Facial. D, Superior alveolar. following structures also passes through the 42. lVhich of the following bones is formed by
E. Mental. E. lnferlor alveolar. diaphragm through the esophageal opening? intramemhranous ossiflCation?
26~ You decide to restore the missing cusp on the A. The aorta. A. Humerus.
patient~s molar. During the administratiort of R The inferior vena cava. B, Lumbar vertebrae.
the inferior alveolar nerve block, which of the
Test items 21-26 refer to the C. The azygos vein. C. Frontal bone of the skull.
follou,ing ligaments is most likely damaged? D. The posterior and anterior vagal trunks.
following test/et. D. Ribs.
A. Sphenomandibular. E. The splanchnic nerves. E. Clavicle.
A 24-year-olcl man presents to your office for an B. Stvlomandibular. 35. The inferior aspect of the diaphragm is 43. Osteocytes are found in _ _ in mature bone.
emergency visit, after being hit on the left side C. T~mporornandibular. supplied with blood by which of the following A. Trabecutae
of his face with a soccer ball. He complains that D, Interdental. arteries? B. Lacunae
his "tooth got knocked out" and that his jaw A. Median sacra! artery. C. The central canal
27. The lateral thoracic wall of the axilla is B. Lumbar arteries, D. Canallc-uU
feels "out of place." He has no other medical
covered by which of the following muscles?
conditions. C. Inferior phrenic arteries:. E, Spicules
A. Pectoralis major. D. Celiac trunk. 44. marks the end of growth in length of long
21. During intraoral examination, you llnd that the
B. Pectoralls minor. E. Superior mesenteric artery. bones.
patient's lower second premolar is missing.
C. Serratus anterior. 36. Oral epithelium is composed of __ _ A. Diaphyseal closure
H'1tich type of alveolodental fibers was least
D. Subscapularis. epithelium. B. Epiphyseal dosure
inuolocd in resisting the force that pulled this
E. Latissimus dorsi. A. Keratinized simple squamous C, o~sification
patient's tooth out of its socket? 28. The trochlea of the humerus bone articulates
A. Apical. B. Keratinized stratified squamous D. Formation of perio5teurn
with the ___.
B. Oblique. C. Nonkeratinized simple e;quamous E. Cessation of bone rernod·c:llng
A. Ulna of the forearm D, Nonkeratinized ~tratifi~d MJu.,,mous 45. The branchial ardtf'.S disappear when the
C. Alveolar crest.
B. Radius of the forearm E. Nonkeratinized stratified columnar branchia/ arch grows down to contact the _ _.
D. lnterradicular. C. Coronoid process of the ulna of the forearm
22. You also notice that a cusp of his mandibular 37. lVhich of tire following statements is true of the A, second: third branthial arch
D. Olecranon of the ulna of the forearm histology of the trachea? B. second; fifth branchlal arch
second molar has fractured off and that dentin E. Medial epicondyle
is exposed. If this patient were to drink A. The mucosa is covered with oral epithelium. C. third; fifth branchia! arch
29. Which of the following muscles of the back is B. Elastic cartilage rings lie deep to the D. first first bran-chlal groove
something cold. what will he sense? supplied by the CN XI?
A Pain. sub mucosa. E. first; sixth branchial groove
A. Levator scapulae. 46. Facial nerves are derived from the _ brancht'al
B. Pressure. C. The cartilage is ring-shaped; the open end of
B. Latissimus dorsi. the ring faces anterior. arch.
C. Vibration,
C. Trapezius. D. The cartilage is covered by a perichondrhtm. A, First
0. Temperature.
D. Major rhomboid. E. Skeletal muscle extends across the open end of B. Second
E. Minor rhomboid. each cartilage. C. Third
D. Fourth
E. Fifth and sixth
I 3 8 • S.ection I Anatomic Sciences Section I Anatomic Sciences T l 39

47. Cytochrome P450 enzymes may be found in 55. Nucleus ambiguus contains the cell bodies of $3. Which of the following organelles i., 69. You decide to remove lhe. caries and prepare
which of the following cellular organelles? whfch of the following cranial nerves? surrounded by a double membrane? the patient for anesthesia. H1hich nerl'e must
A. Mitochon(lria. A. Ill, IV, and V. A. Ribosome, you anesthetize to ensure adequate anesthesia
B. Golgi apparatus. B VII. IX ,and X. B. Go!gi apparatus. for the patient?
C. Lysosome. C. Vil, IX, and XL C. Lysosome. A. Nasopalatlne nerve.
D. Ribosome. D. IX, X, and XL D. Cytoplasmic inclusion. B. Greater palatine nerve.
E. Endoplasmic reticulum. £. IX, X. and XII. E. Mitochondria. C. Anterior superior alveolar nerve.
48. What type of collagen is found in cementum? 56. The articulating surfaces of the 64. Hassal/'s corpuscles are found in the medulla D. Middle superior alveolar nerve.
A. Type I collagen. temporomandibular- joint are covered with of which of the following glands? E. Posterior superior alveolar nerve,
B. Type ll coll•gen. A. Thymus, 70. After administering the anesthetic, the patient
C. Type lll collagen. A. Fibrocartilage B. Thyroid. complains that her "heart feels like it's racing."
D. Type IV collagen. B, Hyaline cartilage C. Parathyroid. You explain to her that it may be from the
K Type V collagen. C. Articular cartilage D. Pineal. epinephrine in the anesthesia. J.Jlhich of the
49. Calcium bin<ls to which of the following for D, Elastic cartilage E. Suprarenal. following glands could most likely cause the
contraction in smooth muscle? E. Perichondrium 6.5. Which of the following are the most abundant same symptoms in the patient?
A Troponin C. 57. The primary sensory nellrons• nucleus of in the fovea centralis of the eyeball? A, Hypophysis.
8. Calmodulin. te,·mination involved in the jaw jerk reflex is A Rod cells. B Thyroid.
C. Myosin. the B. Cone cells. C. PlneaL
D. Actin. A. Facial nucleus C. Rod and cone cells. D. SuprarenaL
E. Desmosomes. B. Trochlear nucleus D. Amacrine cells,
50. Lymph from the mandibular incisors drain C. Mesencephaiic nucleus E. Ganglion cells. 71. All of the following are rotator cuff muscles
chiefly Into _ _. D. Spinal trigeminal nucleus 66, Which of the following bones is part of the except:
A. Submandibular nodes L Nucleus of solitary tract superior wall (roof) of the orbit? A. Supraspirrntous muscle
B. Suhrnental nodes 58. Red pulp in the spleen consb;ts of _ _. A. Zygomatic. B. infraspinatous muscle
C. Superficial parotid nodes A Fibroblasts B. Lacr!mal. C. Teres minor muscle
D. Deep cervical nodes B. T lymphocytes C. Sphenold. D. Teres major muscle
E. Occipital nodes C. B lymphocytes D. Maxilla. L SuhscapulaTis muscle
51. Which of the following muscle attaches to the D, Macrophages E. Ethmoid. 72. The brachia/ plexus of nerves arises from
anterior end of the articular disc of the E. Chromaffin cells which of the follotuing roots of the anterior
temporomandlbular joint? 59. 1'he vertebral artery meets with the basilar primary rami of spinal nerves?
A, Superficial head of the medial pterygoid artery at the lower border of the _ _. Test items 67-70 refer to the following A. All cervical roots (Cl-C8),
muscle. A. :v1ldbrain test/et. B. All thoracic roots (Tl-T!2).
B. Deep head of the medial pterygoid muscle, B, Pons C. C 8 and Tl.
C. Superior head of the lateral pterygoid muscle. C. Medulla A 30-year-old woman comes to your office for a D. C5 through C8 and TL
D. Inferior head of the lateral pterygoid muscle. D, Temporal lobe dental examination. She has not been to the den- E. C5 through C8 and Tl through T4.
52. All of the following arteries are branches of the- E. Cl tist in 2 years. The patient has type l diabetes, 73. The right subcladan artery arises from the
mandibular dir 1ision of the maxillary artery 60. Where are the cells that produce calcitonin which requires her to take insulin. She is other- and the left subclavian artery arises from
except one. Which one is the exception? located? wise in good health. On intraoral examination, you the
A Incisive artery. A. Red marrow. A. Axillary artery; aortic arch
notice that the dorsum of her tongue has a thick,
B. Submental artery. B. Adrenal gland. B. Brachiocephalic artery: aortic arch
C. Middle meningeal artery,
matted appearance and diagnose her with hairy C. Aortic arch; brachiocephallc artery
C. Parathyroid gland.
D, ~·Jylohyoid artery. tongue. You also find that the patient has deep D. Rrad1iocephnlic artery; axlllary artery
D. Thyroid gland,
E. Deep auricular artery. E. Spleen. caries in her upper second maxillary molar. E, Axillary artery; brachia! artery
53. The maxillary nerve pa.~es through which of 61. Chromosomes line up at a cell's equator during 67, Which type of papillae is affected that causes 74. Tile pulmonary vein of tile lung carries:
the following? which phase of mitosis?
the hair~like appearance of her tongue? A Unoxygenilted bJood from the lungs to the heart
A. Superior orbital fissure, A. Telophase. A Foliate. B. Oxygenated blood from the lungs to the heart
8. Internal acoustic rneatus. B. Metaphase. R Circumvallate, C. Unoxygenated blood to the lungs from the heart
C. Foramen ovale. C. lnt,rphase. C. Fungiform. D. Oxygenated blood to the lungs rrom the heart
D. Foramen rotundum. D. Filiionn, E. Oxygenated blood from the heart to the lungs
D. Anaphas~.
E. Foramen spinosurn. E. Prophase. 68. On the patient's radiograph, you notice that 75. The of the heart is also known as the
54. Injury to which of the following nen•es would 62. Which of the folfowfog types of epithelium lines the pulp chamber in the carious molar appears mitral va/t;e.
affect abduction of tire -eyeball? the oropltarynx? smaller than the surrounding teeth. This is A. Right atrioventricular valve
A. Optic nerve. most likely due to the deposition of which type 8- Left atrioventricular valve
A. Simple s.quamou~.
B. Oculomotor. of dentin? C. Pulmonary valve
B. Stratified squa.mou~.
C. Trochleac A. Secondary. D, Aortic valve
C. Simple cuboidal.
D. Trigcminal. D, Simple columnar. B. Tertiary. E. Tricuspid valve
E. Abducens. E, P5eudo5tratified columnar.
C. Mantle.
D. Sclerotic.
Sect.ion l Anatomi;: Sciem::ei y l 41

76. 77,e cricopharyngeus muscle of the esophagus 84. Which portion of uriniferous tubules contains 91. Reduction division occurs during the·--· 97. Which of the following ribs cannot be
squamous epithelial cells? A. First stage of mitosis palpated?
A. ls a parasympathetic shmulator of pcristal&is A. Proximal convoluted tubule, B. Second sta2e of mitosis A. First
B. Is a 5yrnpathetic inhibitor o! peri~ta.lsis B. Thick descending limb of Henle's loop, C. First stage of meiosis 8. Second
C. Prevents swallowing air at the pharyngeal end C. Thin segment of Henle'~ loop. D. Second stage of meiosis C. Third
D. Prevents regurgitation of stomach contents at D Thlck ascending segment of Henle's loop. E. Third stage of meiosis D, A and B
the abdominal end E. Distal convoluted tubule. 92. The embryo develops specifically from the 98. An infectfon in a mandibular incisor with an
E. Controls the gag reflex 85. The _ _ is a component of the apex he/ow the mylohyoid muscle drains into
77. The pancreas is enveloped at its head by the juxtaglomerular apparatus which functions in A. The entire blastocyst which of the following spaces?
regulation of blood pressure. B. The entire tmphoblast A. Subllngual space
A First part of the duodenum A Proximal convoluted tubule C, The embryonic disc B Submental :.ii:pace
R Second part of the duodenum B, Distal convoluted tubule D. The extraembryonic coelem C. Submandibular space
C. Third part of the duodenum C. Bowman's capsule E. The morula D. Parapharyngeal space
D. Fourth part of the duodenum D. G!omeru!us 93. Tooth enamel is derived from 99. The spread of an odontogenic iftfecti<>n to
E. First part of the jejunum E. Macu[a densa A. Endoderm which of the following spaces would MOST
78. The gallbladder arises from the _ _. 86, Urinary flltrate is most hypotonic in the _ _. B. Meso<lerm likely be considered life-threatening?
A Common hepatic duct A, Proximal convoluted tubule C. Ectoderm A. Submandibular space
B. Common bile duct B, Descending limb of Henle·s loop D. Endoderm and mesoderm B. Sublingu~l space
C. Left hepatic duct C. Thin segment of Hen!e's loop E. Ectoderm and mesoderm C. Parapharyngeal space
D. Cystic duct D. Thtck a5cending segment of Ht:nle's loop 94. The olecranon fossa is located on the _ _ D. Retropharyngeai space
E. Bile canaliculi E. Distal convo{uted tubule surface of the E. Pterygomandlbu!ar space
79. The apex of a medullary pyramid in the kidney 87. 11te differentiates into ameloblasts. A Superior; radius 100, The median pharyngeal raphe serves as the
is called the A. Stellate reticulum B. Anterior; humerus attachment site for which of the followi11g
A. Cortex B. Inner enamel epithelium in the cap stage C. Posterior; humerus muscles?
B. Medulla C. Inner enamel epithelium in the bell stage D, Anterior; radius A. Lateral pterygoid muscle
C. Renal papilla D. Outer enamel epithelium in the cap stage l}S. The latisslmus dorsi muscle is supplied by the B. Palatopharyng~u5. muscle
D, Major calyx E. Outer enamel epithelium in the bell stage _ _ nerve. C. Levator veli palatine muscle
E. Minor calyx 88. The dental lamina arises from _ _. A. Medial pectoral D. Salpingopharyngeus
80. Ureters travel inferiorly just _ _ the parietal A Somites B. Cranial nerve Xl E. Superior constrictor muscle
peritoneum of the posterior body wall. They B. Neural crest cells C. Dorsal scapular
pass _ _ to the common iliac arteries as they C. The first branchial arch D. Thoracodorsal
enter the pelvis. D. The second bn:mchlai arch 96. The middle trunk of the brachia/ plexus of
A Above; posterior E. The buccopharyngea! membrane nerves arises from:
8. Above; anterior 89. The correct order of tooth formation is _ _. A CS
C. Below; posterior A, Ameloblasts form, odontoblasts form. B. C6
D. Below; anterior ameloblasts start to form enamel, C, C7
E. Above; superior o<lontohlasts start to form dentin D. CS
81. The lumen of the gastrointestinal tract is lined B, Ameloblasts form, odontob-lasts form,
with _ _• odontoblasls start to form dentin,
A. Mucosa arneloblasts start to form enamel
B. Submuc-osa C. Odontobfasts form, odontoblasts start to form
C. Muscularis externa dentin, amelohlasis form. amclnblasts start to
D. Fibrosa form enamel
E. Adventitia D. Ameloblasts form, amelohfa'lts start to form
82. Gnt-assodated lymphoid tissue (GAL1) enamel, odontnblasts form. odontoblasts start
produces secretory _ _. to form dentin
A, !gA E. Odontoblasts form, ameloblasts form,
R lgD odontoblasts start to form dentin,
C lgE am<:loblasts start to form enamel
D, lgG 90. The auricular hillock.fl are derived from the
E. lgM
83. The muscularis externa has a third layer in the A First branchial arch
R Second branchial arch
A. Esophagus C, First and second branchial arch
B Stomach D. Lateral nasal process
C. Liver £. Medial nasal process
D. Small intestine
E. Large intestine
212 ,. Section 2 BiocheMistry and Phy,iology Section 2 Bior:heml1try and Physiology Y 21 3

(3) Increased extracellular K· alsD ( 4) Hypoglycemia.


increases aldosterone secretion by (5) Hypothermia. SAMPLE OUESTI0NS
acting directly on the adrenal ceils. 3. Mechanisms of action-although both
c. Androgens-regulated by the same epinephrine and norepinephrine are I. Nonsteroidal anti-inflammatory agents are pain- 7. The binding of epinephrine or glucagon to the
mechanisms as glucocortlcoids. secreted from the adrenal medulla, epi- relieving and anti,,inflammatory. They are corresponding membrane receptor has which
3. Actions of adrenocortical steroids (Table nephrine is the main cate.::holamine. ~- effective since they act to inhibit prostaglandin of the following effects on glycogen
2-14) adrenergic receptor stimulation tends to synthesis by: metabolism?
A. Inhibiting fatty acid llpo-oxygenMe activity. A. The net synthesis of glycogen ls increased.
C. Adrenal medullary (catecholamines) hor- be the major response to adrenal
8. Inhibiting fatty acid-«peciflc cyclo-oxygenase B. Glycog~n phosphoryla5e is activated while
mones medullary stimulation, even though glyc0gen 5yntha5e ii inactivated.
activity.
L Structure and synthesis epinephrine stimulates both a- and 13· C. Inhibiting fatty acid-specific hydroperoxidase C. Glycogen phosphorylase is inactivated while
a. Synthesized by a series of steps from adrenergic receptors. activity. glycogen synthase is activated.
tyrosine. a. a-receptor-mediated D. Inhibiting phospholipase A,. D. Both giycogen synthase and phosphory!ase are
b. Epinephrine (the methylated form of (I) Increased gluconeogenesis. 2. The synthesis of all steroid hormones involves activated.
norepinephrine) is the major secretory (2) Decreased insulin secretion. which of the following compounds? E. Both glycogen synthase and phosphorylase are
product. (3) Increased vasoconstriction (splan- A. Pregnenolone. inactivatf':d.
c. The adrenal medulla is essentially a chnic, renal, cutaneous, and geni- B. Proge,terone. 8. When an enzyme is competitfr.ely inhibited,
sympathetic ganglion, which has secre- tal). C. Aldosterone. which of the follorving changes occur?
tory cells instead of postganglionic (4) Increased sphincter contraction (GI D. Cortisone. A. The apparent Km is unchanged.
E. Testosterone. B. The apparent Km is decreased.
neurons. The catecholamines are and urinary).
3. Lipid micelles are stabilized by which of the C. Vmax ls decreased.
secreted directly into the bloodstream 1 (5) Dilation of pupils.
following? D. VmM/. is unchanged.
to play an important role in the "fight (6) Increased sweating. A. Hydrophobic interactions. 9. W'hich compound is produced in the hexose
or flight" response. b. 01-receptor-mediated B. Hydrophilic interactions. monophosphate (jx?ntose phosphate) pathway?
2. Regulation of synthesis and secretion (1 J Increased Jipolysis and ketosis. C. Interactions of lipid and water. A. ATP.
a. Catecholamines are stored in granules (2) Increased cardiac contraction and D. Interaction of hydrophobic lipid tails with B. NADH.
in chromaffin cells. rate. hydrophobic domains of proteins. C. NADPH.
b. Stimulation of the sympathetic nerve to (3) Increased glycogenolysis. 4. Which one of the following carbohydrates is a D. Fructose l,G~bisphosphate.
the adrenal gland releases acetyl- c. j3 2-receptor-mediated ketase sugar? E. Phosphoenolpyruvate.
choline at ganglionic type receptors, (1) Increased insulin secretion. A. Galactose. 10. During exercise, which of the following is
which results in the release of cate- (2) Increased glucagon secretion. B. Fructose. decreased?
cholamines. (3) Increased K• uptake by muscle.
C. Glucose. A. Oxidation of fatty acids.
D. Mannose. B. Glucagon release.
c. Examples of stress-related factors that (4) Increased arteriolar dilation in car- E. Glyceraldehydes. C. Glycogenolysls.
result In adrenal medullarv secretion diovascular musde. 5. Mucopolysaccharidoses are hereditary D. Lipogenesis.
(1) Fear, anxiety, and excitement. (5) Increased muscle relaxation in disorders tltat are characterized by the 11. Increased formation of ketone bodies during
(2) Trauma, pain. bronchial wall. Gl wall, and wall of accumulation of glycosaminoglycan.v in various fasting is a result of which of the follouling?
(3) Hypovolemla. the urinary bladder. tissues due to which of the following? A. tncreased rnddatlon of fatty adds as a source oi
A. Overproduction (synthesis) oi proteoglycans. fuel.
B. Deficiency of one of the lysosomal. hydrolytic 8. Decreased formatlon of acetyl CoA in the liver.
enzymes normally involved in the degradation of C Decreased levels of glucagon
one or more of the glycosaminoglycans. D. Increased glycogenesis in muscle.
C. The synthesis of abnormal proteoglycans. 12. Which of the following enzymes found in the
D. The synthesis of highly branched liver is int1ofoed in gluconeogenesis during the
TABLE 2-14. ACTIONS OF ADRENOCORTICAL STEROIDS glycosaminoglycan chains. postabsorptive state?
6. Hydrolysis of which of the following compounds A. Glucose 6-phosphate dehydrogcna.se.
ACTIONS OF GLUCOCORTICOIDS ACTIONS OF MINERALOCORTICOIDS ACTIONS OF ADRENAL Al\lDROGENS
yields urea? B. 6-phosphogluconate dehydrogcna~e.
Increase g:luconeogenesis Increase Na' resorption Femnlei.: pre,ence of pL1blc and a:xHl.iry A. Ornithine. C. Glucose &-phosphatase.
halr; libido B. Arglninosuccinate. D. Glucokinase.
Increase proteoly~I, (catabo!ic) Increase K+ secretion Males: same as testosteronC>
Increase lipolyl5-i& Increase H+ secretion C. Aspartate.
Decrease gluco•e utillzat!on D. Citrulline.
Decrease in•ulin sens.ltivitv E. Arginine,
Anti-inflammatory ·
Jmmunosuppression
Maintain vascular responsiveness
to catecholamines
Inhibit hone formation
lncrease GFR
De<rE'ase REM sleep
M:Ullil L I ~

Frnm Costanzo LS: Physiolo&'Y, ed 3, WB Sa,mders, Phllm:lelphla, 2004.


Section 2 8\ochcmi,;try 1;111d Physiofogy Y 2 1 5

13. fn which one of the following tissues is glucose 22. Which of the following amino acids is 29. The participation of calcium in the contraction 36. CO,,_ generated itt the tis.'iues is carried in
transport into the cell unaffected by insulin? positioned at every third residue in the of skeletal mw>cle is facilitatetl or associated n•,;r;us blood primarily in which form?
A. Skeletal muscl<:'. p,-imary structure of the helical portion of the with which of the following? ,\ CO., ln the pla~ma.
B. Uver. collagen-a chains? A. Calcium release from sarcoplasmic reticulum. B H.(O _in the p!asma
C, Adipose Hssue. Glycine, C. HCJ\' i:i the pi.sm;1.
D. Smooth muscle. B. Glutamate.
!, B. Cakium binding to the myosin hertds.
l C Active transport nf calclmn out o! k,ngltudirnd D. CO .. in the red blood cdls
14. Which of the following genetic diseases that C. ProUne. tubules.. E. Ca1~ho~yhemoglohin in the n-:d blood Ct'::!~.
res11lls from a deficiency in the liver enzyme D. Lysine. D. Uptake of calcium by !~tubules. 37. Jllfrich of tlw following is thF moHt significant
that co1werts phenylalanine to tyrosine? E. Hydroxyprnline.
A. Alh!nisrn.
B. Horr..ocystirrnria.
23. Which of the following is [!Of involved in the
process of mineralization?
r SO. Calcium that t?nters tire cell during Sf!loo_th
muscle excitation binds with which 6f the
stimulant of the ,·espir-alory center?
A Decreased blood oxygen tension

C, Porphyrla.
D. Phcny!ketonuria.
A. Matrix vesicles.
R. Amelngenins.
I fotlOwing?
A. Calrno<lu!in.
B Increased hlond hydr(lgt:n ion concentralion.
C. Decreased blnod !:ydroJ:!i."n ion concentratlon.
B. lnactive rnyosin kinase. Ll Increased b!ood carbon dioxide ten~ion
I 5. If the molar percentage of guanine in a human C. Fluoride. C. Troponin, 38. The primary factor determining the percf.•nt of
DNA is 30%, wlutt is the molar percentage of D. Phosphoryns. D. Myosin, hemoglobin saturation is:
at.fenine in that nwlecute? 24. ATP is utilized di•·!rJly for each of the following E, Actin. A. Blood PO.,.
A 101,. proces!les except: 31. Which of the follou.1illg does not affect the ll. 13100,J PC()..
H. 20'.Y,. .A.. Accumul-ati(m of C~1"" by the sarcop!asmic muscle tension produced during contraction? C. Diphospho°glycerrtte concentration .
c. 30'){,. reticulum. A. The extent of rnotor~unit recruitment. D. The tPmp~rature of the blood
D. 40%. B. Tram1port of Na" from [ntraceltular to 8. The µroport_i_qn of each single motor unit that is E. The acidity of the blood.
E. 50'.Y,. extracellular fluir:L stimulated to contract. .19. For which of the following subst<uu-es would
16. Which of the following phrases best describes C. Tnmsport of K•· from extracellular to C. The number of muscle fibers contracting. you expect the renal cieonrnce to be the lowest.
restriction enzymes? intracellular fluid. D, The frequency of stimulation under nonrwf conditions?
A. Site-specific endonudeases. D. Transport of H' from parietal cells into the 32. The pressure in a capillarj-' in skeletal muscle A Urea.
B. Enzymes that regulate RNA lumen of the stomach. is 37 mmHg at the arteriolar end and 16 mmHg H. Crentin!oe.
C. Nonspecific -endonucleases. E. Transport of glucose into muscle cells. at the venular end. The interstitial pressure C. Sodium
D. Topoisomerases. 25. Both active transport and facilitated diffusion is 0 mmHg. The colloid osmotic pressure is D. Water.
17. 11,e coenzyme that serves as an intermediate are characterized by which of the following? 26 mmHg in the capillary and 1 mmHg in the E, Glucose.
carrier of one-carbon units in the synthesis of A Transport in one direction only. interstitial fluid. The net force producing fluid 40. The process of active sodium transport in the
nucleic acids is which of the following? B. Hydrolysis of ATP. movement across the capillary wall is which of ascending limb of the loop of Henle is
A. Ascorbic acid. C. Tr.llnsport against a concentration gradient. the following ? absolutely essential for which of the following
B. Tetrahydrofo!ic acld. D. Competitive inhibition. A. 1 mmHg out of the capillary. processes?
C, Biotin. 26. Which of the fo/lou•ir,g statements regarding B. 3 mmHg out of the capillary, A. Regulation of chloride exnPtion.
D Pyrldoxine. the autonomic nervous system (ANS) is true? C. 12 mmHg out of the capiilary. B. R0gulation of pH in extracellular fllml.
18. Following the production of Okazaki A. The third cranial nerve (the oculomotor nerve) D. 3 mmHg into the capillary. C. Reguialion of aidnsternrie excretion.
fragments, which of the following is required to carries symp11.thetir fibers to the smooth .13, A patient has a heart rate of 70 bpm. Her EDV D. Rt'gulation of water excretion
close the gap between the fragments? muscles of the eye. (end-dia.,tolic ,,otume) is 140 mL. Her ESV (end· 41, f-Jyp<;rtension (long-trrm) will he '-'ompemrntnl
A. DNA ligase. 8. The fadal and the glossopharyngeal cranial systolic iiolume) is 30 mL Calculate the CO by u:hich of the follmdng renal nwc-hanisms?
B. DNA po!ymerase. nerves carry the parasympathetic prega.ngllonic (cardiac output) of this imifoidtwl. A Increased circulating ADH (vasopressin).
C. RNA polymerase. fibers for the autonomic innervation to the A. 9800 mL. R. Increased sympa!h-2tJC activity.
D. Reverse transcriptase. salivary glands. B. 2100 rnL C. DenPi\sed circul;:Jting aldosterone.
19. Which of the following is not involr:ed in the C. The parasympathetic nervous system C. 7700 mL. 0, lncr~a.sed circulating nnglolf"nsi11 I!
process of gene cloning? innervates primarily striated muscle in the D. 15.400 mL 42. Whid, of the following statements regarding
A. DNA polymeraie. body. 34. The velocity of blood flow _ _. tubular sf!cretion in the ltidney is true?
B. DNA liga•e. D. The parasympathetic nervous system is A. Is higher in the capillaries th.an the arterioles A The secrd ion of K' increases when a per~on is
C. RNA polyme,rase. organized for diffusie activation and responses, B. ts higher in the veins than in the venules
D. Reslrlction endonuc!ease. 27. Which of the following resporu;es is due to the C. ls higher in the veins than in the arteries B. The secretion of H' increases when ,1 person is
20. Vitamin K serves as a coenzyme for: stimulatfon of a--adrenergic receptors? D. Falls to zero in the descending aorta during in alkalosis.
A. The enzymatic hydroxylation of proline to A Slowing of heart rate. diastole C. H is a process thf1t transports ~ubstances from
4-hydro:xyproHne. B. Constriction of blood vessels in skin. 35. Which of the following dQi!~ not occur to the fHtrate to the caplllary blood.
B. The carboxylation of inactive prnthromhin to C Increased gastrointestinal motility, compensate for a fall in blood pressure below n !t accounts for most of the K' ln thrc urine
form active prothrombin. D. Increased renal blood flow. normal values? 4,1. Which of the folfmring statements regarding
C The synthesis of nucleic acids 28. Which of the fallowing is a property of C A. Increased cardiac output. salh:arv secretion is true?
D. Protein synthesis. fibers? B. Increased strokt"' volume. A. In ge-.neral, sallva is more hyperionic than
21. Chondroitin sulfate is a major component of A. Have the slowest conduction velodty of any C. Increased heart rate. plasnrn.
which of the following? nerve fiber type. D. De(:re~\sed total peripheral resistance, B. As sct!ivary flow lncreases. bicarhnnats:>
A. Bacteri~1! cell waits. B. Have the largest diameter of any nerve fiber type. cnncentrntlon ckcrc:ases,
B. Mucin. C. Are afferent nerves from muscle ,pindle~. ,, As salivary How i11crc◄1s(~s. ionic conccntrntion
C. D. Are afferent nerves from tendon organs.
D. E. D. regulats.'.d primarily by
E. Hair. hnrn1onal stimul;itinn,
216 v Section 2
Section 2 810,:;he,mistry and Phyw,logy T 2 l 7

44. Which of the following is the predominant $]. Which of tlu;, following proteoglycan:,,· is present 60. Which of tire following cr.:u::nzyme$ are inoolped 68, The co,wer.•..ion of information f'rom DNA into
immunoglobulin in whole safii,a? in extracellular space? in the melahoUsm ofpynwate to acetyl CoA? mRNA. is called rthich of the follmrlflg?
Secretory Hyaluronk acid. Thlamin pyrophosphat0. llpoic acid, FAD, NAD, A. Translation.
B. Secretury B. Kerat(l.n ,md coenzyme A. H. Trnnscrlption.
Secretory C. Chondroit!n suliate. B. NAD, tetr.ahydrofola!t:. lipoit acid, FAD. and C. Transduction
fl Secretory D. Dennatan sulfate r>. Transformation.
45. The pancreas produces enzymes that are Hepar1n. C. nicotinmnlde aden!r:e dlnucleo!;de 69. Which of the following minen:1Ji:::.ed tissues
responsible for the digestion of dietary 52. Porphyrins use which amino fldd in their hrwc the greatest percentage of inorganic
<"ompounds. Which of the following foods synthesis? ~). maU:ria{?
would not be digested by enzymes synthesized A. Aianlne. 61. Relative or absolute lack of insulin in humans A, Enamel.
and secreted by the pancreas? B. Pheny!alM1in<2. would result in which one of the following B Dentin,
A. Carbohydrates. C. reactions in the lhier? Berne.
B, [), A. lncreased D. Calculus.
C 53, Which af the fol/awing is an essential amino B. Increased gtuco11eoger1es:1s 70. Each of the following <le,.,cri/)(.:s collagen e.1.:cept
D. Protein. acid? C. glycngen hr. '<kdown
0
one. l:Vhich is the exception?
46. Which of the following statements reflarding A D. Increased amino ;1dd uptake. A. rv1ost almndant protein in the hody.
the hormone secrelin is true? R 62. Whlch one of the following is elevated in Modincatlon;-;: in prncn!Lagcn occur in th0
A It is res:pon1'tble for activating C plasma during the absorptive period extracellular matrix
Serine. (compat·ed to the postabsorptive state)? C
B. t!Hc: releas.e of pantreatic secrt:tion Alanlne. A Chy!omicrons.
rich in bicarbonate. 54. Aspartame contains aspartic ,acid and which B. Acetoacetate.
C. It st!mulates the release of pancreatic of the following amino acids? C Lactate. oxygen.
D. It stimult>tes the contraction of thf> Pheny!alnnine. D. Glm:agon. 71. llydroxya,,atitc _ _ ,
to release bile. Ii Leuclne. 63. Insuliri produces which of the following A. ls if fluoride is suhstituterl for scmf'
47. Plwsphotipase C is an enzyme that plays an lsoleucine. changes in mammalian cells? of the hydroxy! inns
important role in the production of second D. A. Increase in !iver productlon.
messengers~ which produce intracellular E, B. Increase in concentration, carhonate ion hr(omcs ,nore
responses. Which two second messengers are 55. Which of the following is the end product of Decrease in the transport of glucose into soluh!e
produced through the action of this enzyme? purine degradation in humans? musde. D. ls corn posed nf calcium .and phnsph<tit'.: ln ;:1 l: 1
A. cAMP and tyrosine kinase. A, Urea, D. Increase in the transrmrt of glucose into the ratio
B. AcetylchoHne and histidine. R Urlc acid. bra.in 72. The type of collagen characteristically found in
C. Adenylate cyclase and protein kinase. C Adenosine, 64, Which of the following describes the function cartilage is which of the follocdng?
D. L2-<li;icvM•vcerol and inosltol 1,4, D. Xanthlne. of RNA polymerase? A, L
56. Which of the following participates in both A. Translates DNA into protein, B, IL
48, Hormones secreted by the posterior piluitarJi fatty acid biosynthesis and f>oxidation of fatty B. Terminates transcription. C !IL
gland include which of the follrm:ing? acids? C. Removes introns trans-crlpHmL !\/.
A, Prolactin. A Malonyl CoA, D. Synthesizes RNA 73. process transports amino adds across
B. FolHcle~stirnu!ating hormone. B. FAD, 6S. Analysis of DNA fragments (probing) is possibly tlte luminal surface of the epithelia that lines
C. hormone. C:oA <lue to which of the following properties of the small intestine?
D, Va;;o1Jrc,sin. DNA? A. Simple diffusion.
of which one of the following 57. The rate-limiting enzyme in glycolysis is which B. Primarv active transport
hormones are not mediated through cAMP?
A , "''"""'"""'"n bonds.
of the follo1ving? B, com,:,1m1entary strands. Cotran;port with sodium ion
A Estrogen. A F'rw:tose blsphospl:atase, C. Protein hindlng. D. Cotransport wlth chloride inn.
R Glucagon. B, 1'1l,1sn,hoiln«:tnlzirn1se, D. \Vestern bloHmg:. 74. Cell membranes typically contain th<: f'o-llowing
C. Epinephrin<". 66. 17,e amount of cytosine will be equal to the compounds except:
D. Nor<~pinephrine. D. Glucoklnase. amount of guanine in urhich of the following A. Phospho!ip1ds
50, A scientist has di$covered a new pepiide 58. The coenzyme essential for normal amino acid molecules? B. Proteins
hormone. He thinks it acts through the second metabolism is A. DNA. Cholesterols
mes,~enger system, which utilizes c,4MP. If this A. Biotin R RNA,
;s true, which of lht? following substances B. Tetrnhydrofo!atc C. DNA and RNA.
should decrease the response of this new phosphate D, mRNA
p-eptide hormone in cells? 67. What is the correct general structure of the A. ln;ictlvrites reduced stNoici derivatives
A. A,denylate cyclase. 59. Which of the foflowing metabolic aclivitie~,; is backbone of DNA and RlVA? B. Is not with nerves
B. MonoaminP oxiflase !nhlhit0rs, increased 1 hour after a meal (during the C. lnartivates e;_1tecbninmlncs by oxid:1Uv1~
C. Phosphodiesterase absorptive demnin.:itinn
l} Aspirin. A, u1ycngenc>1ysis, D, Is !ocaled in the synapse where it inr,etivates
B. Oxidation free fatty acids. the neurotr;:insmHtt:r :u:etylchnli1w
C. Glucagon release.
D, Glycol:,:rsis.
2 18 ,r Section 2 Biodiemblry and Physiology

76. Which one of the following does not release 83, .4.ll of the following focal chemical factors will 92. Which of the follordng statements regarding 96, Which of the l'ollmring factors will not
acety{cholinr? canse vmwdilatation of the arterioles_. except: tnbular reabsorptfon is true? influence the rate at which a meal uAll leart'
A Symp.:"lthetic prcg;;rnglionic fi:)ers. A. Decreased K· A. :\1os1 talclum filtered is passively reabsorbed the stomach?
B, Symp;i_thetk ;1ost.c1mgllon!c fibers that R lnc:reased CO: and not regulated under ar:y conditions. A. AcidHk:atinn of the duodenum.
innervate the heart. C Nitric oxide B. Most urea is reahsorhed passlvely and is U. lncre(1sing the tonkity of t!w ·mtc.stiiw,
C. Parasympatlwtic pnstr41rnglinnic fibers to D. Decreased 0 unaffected by regulatory mechanisms. C. Saline in the duodenum.
effoctor organs. £. Histamine reI~a!.e C. Glucose is reabsorbecl by secondary active D. Lipid in the intestine.
D. Para.symp;thetic pregaogiinn!c fibers. 84. Increasing the radius of arterioles will trnn~port and facilitated dHfusion. 97. H'hich of the followlng forms of thyroid
77. Where are the temperature control centers increase which of the follmti~g? D. Most filtered phosphate is reabsorbed ln the hormone is most readily found in lhe
located? A. Systolic blood pregsure collecting ducts and ls unaffected by regulatory circulation?
A. Cen:;bel!um. B. DiDstnlic blood pressure. mechanisms. A Tri~iodothyroninc (T)
B. Hypothalamus. C. Viscosity of the blood 93. Which of the following processes is not a trtie B. Thyroxine (T.1).
C. Mecl111ln. D. CapiHary blood :low. component of swallowing? C Thyroglobulin.
D. Cerebra! cortex. 85. In which of the following might arterial blood A Closure of the gfottis. D. TSH.
78. The energy for skeletal muscle contraction is pressure be abnormally high? R Involuntary relnxation of the upper esophageal 98. A hormone acts lo stimulate Us neighboring
derived from which of the following processes? A. Ventrlcular fihrillation, sphincteL cell to divide. This hormone would best be
A. Calcium release from sarcoplasmic membranes R Acute heart failure, C, Movements of the tongue against the palate. descrlbed as belonging lo which category of
and bindlng t\1 troponln. C. Anaphy\actic shock D. Esophageal peristalsis. hormones?
B. Cleavage of ATP by the myosln heo.d. D. focreased intrncranial pressure. 94. Which of the following statements regarding A. Paracrlne.
C. Membrane sodium~pntassiu,n A.TPase pump. 86. A major function of surfactant i$ to increase the regulation of gastrointestinal motility is B, Autocrlne.
D. Sorhum inf1ux rlurlr,g the action pntentiaL whid1 of the following? -- true? C. Endocrine.
79. lV/uscle spindle stretching when lhe patellar A. Pulmonary compliance. A Sympathetic stimulation inhibits motility. 99. Blood levels of progesterone are highest
tendon is tapped produces which of the R Alveol.tr surface tension. B. Parasympathetic stimulation inhibits motmty. during _ _.
following responses? C. The work of breathing. C, Gastro!ntestlnal motilitv is not influenced bv A. The fn!lkuiar phase of the ovarian cycle
A. Muscle contraction within muscle where the D. The tendency of the lungs to collapse. the central nervous sy;tem (CNS). ~ B, The luteal phase of the ovarian cyclt'
:ii:plndles are located. 87. The minimum volume of air that remains in the D. Gastrointestinal motility is not influenced by C. Ovulntlon
B. Inc.:reasi:-d symp;1thetlc stimulation of the lungs after a maxlmal expiration is termed the hormones. D. Menstruation
spindles. 95. Which one of the following statements 100. Glucagon trill decrease u:hiclt of the
C. A reduction in the number of afferent lmpu!s>:s A. Tldal voiume regarding the regulation of gastrointestinal following?
entering the spinal cord. B. Funf'.tlor;al residual caracity function is true? A. Glycogenolysis.
D. An lnhib\tion of the stretch reflex. C. Residual volume A. The main sympathetic nerve supply to the B. G!uconengencsis.
80. The gamma motor neurons control which of the D. Vital capacity digestive tract is the vagus. C. Glycogenesis.
following? 88. The center that prot:ides output lo the B. fn general, sympathetic stimulation is D. Blood glucose.
A. Muscle spindles. respiratory muscles is located in the _ _. excitatory to ctig-estive activity.
ll lr!s of the eye. A. Pons C. Salivary secretion is stlmu!ated by both
C. Voluntary muscle fibers. B. Medulla branches of the autunomic nervous system,
D. Pyloric sphincter. C. Cerebral cortex although not to the same deb'Tee,
81. Which of the following would be expecll!d to D. Cerebellum D. Paraiymp.athctic stimulation of the salivary
raise blood prestmre? £. Hypothalamus gland:.'! produces a saliva rich in mucus.
A. A drug that lnhlbits tlw nnglotensin 11- 89. Aldosterone --··
converting enzyme and thus the production of A. Stimulates Na-· reabsorption in the dlstal and
angiotensin [l (ACE inhihltors). cnllecting ducts
B. A drug that inhihits the synthesis ot nitrlc R ls secret~cl by the juxtaglnmerultir appar:\tus
oxide. C. Stimu!ateg K• abeorption in the distal tubule
C. A drug that blocks V1"1~opre::.sin receptors. D. Stimulates bicarhonate reabsorption in the
D. !ncn~r,sed stlmnlnHon of the carotid proximal tubule
barorcc-c-ptor. 90, Whal happens to net fluid filtration in the
82. Which of the folfowing ions has a hlgher glomerul1u; when plm;ma protein concentration
inlracellttlnr concentration compared to the is decreased?
extrace.llutm· fluid? A Net till ration (ullrafiitration) Increases.
A. l\a'. B. Net filtrntion (ultraflltralion) decreases.
B. K'. C. !\;et filtration remains unclumg'P(t
C. ci·. D. Net fi!trn.tiori cea~es,
ll. IICO, ,91. Which of the following factors would result in
E. Ca11 , decreased glomerular filtration rate?
A. A foll in plasma protein concentration.
B. An obstruction nf the tubular system which
would increase capsuiar hydrostatic pressurP.
C. Vasodilation of th" <1fferent arterioles.
D. !nulin arlrnin!stratlon.
272 Y Section 3 Microbiolos:,' and ?ethology Section 3 Mkrobmlogy and P<1tho!ogy Y 273

16. Which of the folfou·ing microbes is the most 24. The most prominent mechanism of spread of
SAMPLE 0UESTIONS common cause of gastroenteritis in children? the hepatitis A virus is bv which of the
A. Reovlruses. following routes? V

1. All of the following cells are assocfoted with 9. All of the following conditions are commonly B. Picomaviruses. A Oral-anal.
chronic inflammation except one. Which one is associated llJith a group A, f3•hemolytic C. Toga.viruses. B Respiratory.
the exception? streptococci infection except one. Wltich one is D. Paramyxoviruses. C. Sexual contact.
A. Macrophages. the exception? £. Orthomyxovlrnses. D, Perinatal.
B. Neutrophils, A. Scarlet fever. 17. A cotton wool appearance may be used lo E. Insect vectors.
C. T lymphocytes. B. Toxic shoek syndrome. describe the radiograph of a patient with 25. Accumulation of fluid in the pericardium
D. B lymphocytes. C. Pharyngitis. A. Osteopetrosis occurs mosl often with which of the folfou·ing
E. Plasma cells, D. Endocarditis. B. Osteitis cleformans conditions?
2. Dust cells can be found in th; E. Impetigo. C. Peutz~Jeghers syndrome A. Unstable angina.
A. Brain 10. Karyotyping can be used to diagnose which of D. Seborrheic kerntosis B. Cardiomyopathy.
B. Heart the following diseases? E. Osteogenesis imperfecta C. Myoc-arditis
C Lungs A. Kltnefelter's syndrome. 18. An autoclave ste,·ilizes dental instruments by D. Acute pericarditis.
D. Liver B. Multiple rnyeloma. causing which of the following? E. Tmnponade.
E. Spleen C. Niemann~Pick dlsease. A. Coagulation of proteins. 26. The most common cause of pyeloneph-ritis is
3. Which of the following mediators aid in the D, Pemphigus. B. Denaturing of proteins.
killing of intracellufar bacteria? E, Peutz~Jegher~ syndrome. C. Precipitation of nucleic acids. A. Staphylococcus aureus
A. Histamine. 11. In pem11higm,, autoantibodies are directed D. Disruption of cell membranes. R Vibrio cholerae
B. lnterteukin-2. against which of the following structures? E, Dissolution of lipids. C. Escherichia coli
C. Catalase. A. Acetykholine receptor. 19. Ehlers~D«nlos syndrome is a disease affecting D. Helicobacter pylori
D. IgG. B. S-arcomere. L Bordett7lla pertussis
E. Lysozyme. C. Epidermi5. A. Bone 27. Polycystic kidney disease is most commonly
4. The class of antibodies that plays an important D. Thyroid follicle. B. Connective tissue associated with
role in type I hypersensitfoity reactions is E. Lysosomt:s. C. Muscle A. Renal cell carcinoma
A. lgA 12. Which of the following is a major complication D. Joints 8. Peripheral neuropathy
B. lgD of chronic bronchitis? E. Glycogen synthesis C. Urolithia~i~
C. lgE A. Myxedema. 20. An increase in alkaline phosphatase may be D. Berry aneurysm
D. lgG R Pncumothorax. seen in all of the following conditions except E. i:\fon-Hodgkin ·~ lymphoma
E. lg!>1 C. Emphysema. one. Which one is the exception? 28. Squamous cell carcinoma is the most common
5. DiGeorge's syndrome is characterized by a D. Pernicious anemia. A Hyperparathyroidi5m. oral cancer. It is a tumor of
deficiency of _ _ . E. Malignant transformation. B, 01teoporo5i5, A. Melanocytes
A. B lymphocytes 13. Which of the following cells are defecthie in C Oiteiti~ deforrnans. B. Basal cells
B. T lymphocytes chronic grannlomatous disease? D. Adenoc-arcinoma of the prostate, C. Fibroblasts
C. Both B and T lymphoq1es A. Neutrophi!s. E. Multiple myeloma. D. Keratinoc-ytes
D. Antibodies B. Lymphocytes. 21. The most common cause of death in diabetic E. Macrophages
E. Complement inhibitor C, Plasma cells. patients is _ _. 29. Tinea pedis, which is commonly known as
6. Which of the following is tile most common D. Ki!ler T cells. A. Peripheral ncuropathy athlete's foot) is a fungal infection that is
cause of subacute endocarditis? E. Macrophages. B. Pancreatic cancer caused by the fallowing dermatopl,yte(s):
A. Staphylococcus aureus. 14. Which of the following describes cells that are C. Cardiovascular disease A. Microsporum
B. Staphylococcus epidermidis. abnormal in appearance and may become D. Kidney failure B. Tridwphyton
C, Streptococcus viridans. pre.malignant? E. Opportunistk infections C. Epidt:rmophytnn
D. Streptococcus pyogenes. A. Aplasia. 22. Ncuraminidase is produced by _ _. D. Both A and B
E. Streptococcus pneumoniae. B. Dysplasia. A. Influenza virus E. Both B and C
7. Aschoff bodies are obserued in which of the C. Karyomegaly. B. Hepatitis C viruses 30. Fibrotic and thickened heart valves that result
following conditions? D. Pleornorphism, C. Human immunodeficiency virus in a reduction of blood flow through the valve
A Acute mye!ogenous leukemia. E. Metaplasia. D. Measles virus characterize which of the following?
R. Pheochromocytorna. 15. The HIV vii-us binds directly to the surface £, Rubella \lirus A. Stenosis.
C. Osteopetrosis. receptors of CD4 lymphocytes with _ _. 23. Which of tire follou·ing skin lesions is most B. Reg1:rgitation.
D. Rheumatic fever. A Reverse transcriptase likely premalignant? C. (n5uffidency.
E. Sderoderma . B. In tegrase A. Verruca vulgaris. D, Prolapse.
8. Endotoxin consists of C. Hemaggluttnin B. Keloids. E. lschemla.
A Lipopolys.accharide D. Glycoprotein 120 C. Seborrhek keratosls. 31. Cystic fibrosis is a here<fitary disorder that
b. M protein E. Protease D. Actinic keratosis. results from defective
c. Hyaluronidas.~ E. Compound nevus. A. Collagen
d. Lactic acid B. Lysosomal ent}Tie~
e. Coagula.se C. Chloride channels
0. Fibnllin
E. Myelin
274 Y Section 3 Mkrobidogy and Pathology Section 3 Microbidogy and Patho!ogy .,, 275

32. The most common mutation accounting for the 38. Upon further eraluarion, the doctor requests 43. Cafe au laif spots are seen in conjunction with 49. The clinical pre,"ientation of hemophilia Bis
pathogenesis of trisomy 21 is _ _. an HI\! and hepatitis test. The laboratory potyostotic fibrous dysplasia and endocrine indistinguishable from hemophilia A. Which of
A. Chromosome translocai.ion performed both an ELISA test and Western blot~ abnormalities in which of the following the following best describes the laboratory
B. Meiotic nonclisjunction revealing that the patient is HIV-positive. The disorders? method needed to <lh,tinguish these two
C. Mitotic nondlsjunctlon Western blot is used to identifv which of the A. McCune--Atbright's syndrome. conditions?
D. Single deletion following? ,., 8. Stevens-Johnson 5yndrome. A. Bleeding time.
E, X~link:ed inheritance fl. Antibodies. C. Marfan's syndrome, B. Assay of congulatinn factor levels
33. An endocrine disorder that causes an early B. DNA. D, Goriin-Goltz syndrome, C. Assay of von Willt>brand's factor.
loss of primary teeth and the early eruption of C. RNA. E, Peutz-Jeghers syndrome. D. Blood smear.
secondary teeth is _ _• D. Proteins. 44. The patient's radiographs could be described E. Platelet count.
A. Ylyxedema E. Plaque-forming units. ·1 as having wlrat 1:)-pe of characteristic
B. Hashimoto 's thyroiditis 3.9. Given the patient's history, if the patient was appearance? 50. An infection by which of the following bacteria
C. DlGeorge's syndrome later diagnosed with active hepatitis, which of A.. Cotton wool. may result in the formation of gummas?
D. Plummer's dl&Mse the following would most likely be the B. Ground glass. A. Myc0bactnium tuberrnfosis.
E. Dwarfism causative agent? C. Cobweb. B. Nei:1serfr1 gonnrrhore<)e.
34. Which of the followirrg is not a feature of A. Hepatitis A. D. Soap bubble. C. Trcponema pallidttm.
poststreptococcal glomerutonephriti.s? B. Hepatitis B. E. Starry sky. D. Bordete!!a pertw;sf.-.·.
A.. Hematuria. C Hepatitis C. 45. A bone biopsy was taken from the patient. £, Stn:ptocoCC'us pyngenes.
B. Hypertension. D, Hepatitis D. Which of the following would most likely be 51. Which of tlte following receptors are
C. Edema. E. Hepatitis E. obserued under the microscope? recognized by CDS lymphocytes?
D. Polyuria. 40. Which of the following would the doctor likely A. A dense inflammatory infiltrate. A. Cluss I MHC molecules.
35. The most common cause of osteomyelitis is prescribe for the patient's intraoral infection? B. f'ibrous tissue, B. Class II MHC molecules.
A. Amoxicl!Hn. C. Pleomorphic: cells, C. Surface lgE.
A. srr~ptoCOffU$ pyogenes B. Vancomycin. D, Metastatic calcifications. D. Surface lgM.
B. Staphylococcus aureus C, Ciprofloxacin. E. Giant cells, E. Histamine receptor.
C. lactobacilius casei D. Nystatin. 52. Which of the following cytokines stimulate B
D, Pseudomonos aeruginasa E. Chlorhexidine. lymphocytes to differentiate into plasma cells?
E. Escherichia coif 41. All of the following molecules may be found Test items 46-49 refer to the following A. IL-!.
36. All of the following are histopathologic within the nucleocapsid of an HIV virus except test/et. 8. IL-2.
features of malignant cells except one. Which one. Which one is the exception? C. lL-3.
one is the exception? A, Reverse transcriptase, A 6-year-old boy presents with a history of severe D. IL-4.
A Anaplasia, B. Integrase. epistaxis. For the past 3 years the patient has E. IL-5.
B. Pleomorphism, C, Neuramlnidase. experienced these nose bleeds, often without any 53. All of the following symptoms are mediated bJ•
C Aneuploidy. D, Protease. apparent cause. The patient is otherwise ln good antibodies except one. Which one is the
D. Large nuclei. E. Ribonucleic acid. health, but his mother has noticed that he exception?
E. Low nudear,cytoplasmic ratio, 42. The patient is referred to an infectious disease "brnises easily," Laboratory tests are ordered. A. Arthus reaction.
37. Which of the following be.st describes specialist and placed on "triple therapy." Two B, Tuberculin reaction,
46. The laboratory test results show a normal PT
anapfastic cells that have not invaded the years later. the patient is admitted to the hut a prolonged PTT. A prolonged PTT test C. Asthma.
basement membrane and are confined within emergency room with a dry cough and suggests that the patient has an abnormality D. Erythroblastosis fetalis.
their epithelium of origin? shortness of breath, His temperature is 101 affecting t.l'hich component of the coagulation E. Serum slckness.
A. Dysp!asia. degrees F. The most likely cause of the patient's cascade? 54. Which of the following is re/pased by mast cells
B. Hyperpla:sia, pneumonia is A. Activation oi platelets. after antigen binding?
C. Mcti,_pln.3ia, A. Staphylococcus aureus B. Activation oI thrombopla.stin. A. lgE.
D. Sarcoma. B. ffaemophilus influenzae C. Activation of plasminogen. B. Lysozyme.
E, Carcinoma in situ, C, Pneumoeysti.'i jiroveci (carinii_) D, intrinsic pathway C. IL-4.
D. Kleh.1:fr,lfa pneumoniae E. £.xtrinsic pathway. D. Leukotriene.
E. .S'treptococcus pneumoniae 47. The diagnosis of hemophilia A is made. This E. Interferon.
disease is caused by a defidency of _ _. 55. Symptoms of a myocardial infarction include
Test items 38-42 refer to the following all of the following except one. Which one is the
test/et. A. FactorV
8 FactorV!l. exception?
Test items 43-45 refer to the following A. Angina.
A 43-year-old man presents for an emergency den- C. Factor VIII.
test/et. B. Diaphoresis.
tal appointment complaining of a burning sensa• D. Factor IX.
tion in his mouth. Upon examination, white E. F'actor X. C. Fever.
A mother brings her &.year-old daughter In for an
48. Which of lite following describes the hereditary D, Vomiting.
plaques are observed along the oral mucosa. The examination because she noticed hrown macules E. Oyspnea,
patient otherwise appears healthy. There is no his- on her daughter·s leg. The macnles have jagged transmission of this disease?
A Autosomal dominant.
56. A positive quelling reaction can be observed in
tory of systemic illness. but the patient did state edges but do not appear raised. The mother is bacteria with a _ _.
that he had a blood transfusion more than l O years B. Autosomal recessive.
worried that her daughter may have a malignancy. A. Thick peptidoglyo1n layer
C. X-linked.
ago following a car accident. The doctor referred After further evaluation and tests, the macules are B. Capsule
D. It is not genetically transmitted.
the patient to emergency room for further tests, identified as cafe au lait spots, C. Flagella
D. Cell wall that contains teichoic acid
E. Glycocalyx coating
276 T Section 3 Mv:robtoiosy ,.,,d Pilthology Section 3 Microbiolcsy Md P<1tholo5y • 277

.57. Which of the following groups of micro- 65. Rheumatoid arthritis is characterized by 73. Which of the following structures is the most 81. A p,·oductive cough may be .-,een in all of the
organisms produce dipicoli.nic acid? inflammation of the _ _. common .site for oral cancer? J'ollo,dng conditions except one. Which one is
A. Actinornycetes. A Articular capsule A Soft palate. the exception?
R Histop/asma. B. Articular cartilage B. Lat~ra.l border of the tongue, A. P1wlrn1onk1s
C, Streptococcus. C Cortical bone C. Lower llp. R Lung abscc.ss.
D. Staphyiococcus. D. Perichondrium D. Floor of mouth. C. RronchiectHsls
E. l1o,'<;f1'idillm. E. Synovium E. 13uccal mucosa. D. Asthma.
58. Which of the following consists of glucose 66. The most common form of breast cancer is 74. The presence of Auer rods in a peripheral E. Brnncbogf:nlc carcinoma.
molecules linked together that act as the blood smear suggests which of the Following 82. Antinuclear antibodh:s are seen (n the se,·um
structural component of plaque? A Adenocarcinoma conditions? samples from 1>atients with
A. Fructose. B. Teratoma A Acute lymphocytic leukemia. A Hypngamn1ag_inhnlin~mi11
B. Sucrose. C. Fol!icular lymphoma 8. An11f' lymphob!aslic leukemia. B Chronic.' gram;lomatous: disease
C. Levans. D. Sarcoma C. Anll e myelogenous leukemia. C. Sy'.;tt"mic lupus erytht•nrntosns
D. Dextrans. £. Carcinoma D. Chronic lymphocytic leukemia- D. Multipll" mydnma
E. Fructans. 67. Which of the following antimicrobials is £. Hodgkin's lymphoma E. J~heochrn111nt'.V1on1a
59. 11te most common cause of bacterial meningitis bacteriostatic and inhibits protein synthesis in 7.5. Complications of Barrett'$ eso/Jhagus include 83. Nephrolithiasis ,itt most likely to be associated
in newborns is _ _. bacteria? all of the follorring except one. lt'hich one is the with which of the foffouiing t:onditions?
A. Staphylococcus aureus A. Streptomycin. exception? A. Hyperparathyn,idi5rr1
B, Streptococcu5 pnf!.umonio~ B. Penicillin V. A. Varlces. B Myxedema.
C. Escherichia coli C. Ciprofloxacin. B. Stricture. C. Pydonephritis.
D. Haemophilus influenzae D. Cephalexin. C. Hemorrhage. D. VVllson's diseas~
E. Listeria monocytogenes K Tetracycline. D. Adenocarclnoma E. Tl1r0mh0cytopC'nla_
60. All of the following can be found in the cell 68. Which of t/1e following may be obserued in a E. Ulceration. 84. A.n infant diagnotoed with ost(copetrosis has
wall of a gram-negative bacterium except one. child diagnosed with rickets? 76. The presence of Al-protein antibodies suggests dysfunctfonal _ _.
Which one is the exception? A. Dark pigmentation on the oral mucosa. immunity to infection by which type of A. Chnndrocytes
A. Endotoxin. B. Early eruption of teeth. bacteria? 8, Osieoblasts
B. A thin peptldoglycan layer. C. Hutchinson 'is incisors. A. Stn-ptococcus pyog,:,ri1s C. Ostenclasts
C. l.lpopolysaccharide. D. Abnormal dentin. 13. Streptococcus l1irufans. D. Fibroblasts
D. Teicholc acid. E. ~acroglossia. C. Streptococcus sanguis. E. Lymphocytes
E. 0 antigen. 69. In 2% glutaraldehyde, which of the following D. Staphylococcus aureus. 85. All of the following factors play a role in the
61. What type of vaccine is used for Clostridium times is minimally sufficient for achieving E. Lactobaci!fus easer virulence of the mkrobe that cau!ws w!wo11ing
tetani? sterilization? 77. Hemorrhagic infarction and tissue necrosis cough except one. Which one is the exception?
A. Capsular polysaccharides, A l 5 minutes. suggest which of the following? A. igA protease.
B. Toxoids. B. I-2 hours, A. Aspergillosls. B. Hemagg'httinin.
C Kiiled bacteria. C. 6 hours. B. Blastomycosls. C. Exotoxin.
D. lmmunoglohulins. D. 12 hours. C, Histopl7,1smosis. D. Capsule.
E. No vaccine is available. 70. An 8-J·ear-otd boy presents with macr-oglossia D. Mucormycosls. E Pili
62. The class of antibodies that plays an important and delayed eruption of his primary teeth. E. Toxoplasmosis. 86. T--cell lymplwm<r is most likely to ot:cur in a
role in mucosa[ immunity is _ _. Of the following choices, which one is most 78. Which of the following is usually least patient tL'ilh which of the folloming conditions?
A. lgA likely? malignant? A. Chninic granuiornatous disease.
B. lgD A. Plummer's disease. A. ACl!te lymphoblastlc leukemia. Ft Myastheuia gnwi!'I.
C. lgE B. Osteochondroses. B. Acute lymphocytic leukemia. C. Osteochondroma.
D. lgG C. Cretinism. C. Acute myeloge11ot1!i leukemia. 0. Wilson's disease
E. lgM D, \Vilson's disease, D. Chro11k lym1Jhocytic leukemia. E. Celiac sprue.
63. The presence of which of the follou:ing in a E, Mallory-Weiss syndrome. E.. Chronic myelogcnous leukemia.
patient's serum indicates that the patient is a 71. Which of the following bacteria would be 79. Bronchogenic carcinoma is a complication
highly infectious hepatitis B carrier? expected to fir&t colonize onto plaque? most characteristic of which of the following Test items 87-92 refer to the following
A. HBsAg. A. Streptococci. conditions?
B. HBsAb.
test/et.
B. Bacteroides. A. SiHcosis.
C. HBcAg. C. Fusobacterium. B. Asbestusis. A 60-vear-old homeless man who lives in a com-
D. HBeAg. D. Actinomyces. C. Anthracosis. r:nmtly shelter presents with history of coughing
E HBeAb. E. p,.,,votel/a. D. Sarcoidosis, !or th~ past 6 months. He has a slight lever.
64. All of the following microbes listed are 72. The hereditary transmission of Peutz.Jeghers E, Bronrhiectasis. hemoptysis, and productive cough with a yellow-
associated with infections secondary lo an HIV syndrome is _ _• 80. Which of the following disorder& is least likely ish sputum discharge. After further examination
infection except one. Which one is the A Autosomal dominant to be included in the differential diagnosis of a
-exception? and tests. the patient is diagnosed \Vith actlve
B. Autosoma! recessive patient wilhjaundice?
A, Pneumocystis jiroveci (carinii). C. Sex-linked dominant tuberculosis.
A Hepatitis.
B. Epstein~Barr virus. D, Sex-linked recessive B. Hemolytic fl.nemia.
C. Coxsackievirus. E. Not genetically transmitted C. Cholelithiasis.
D. Mi-·coi)acterlum tuberculosis. D. Glomer·u1onep11ritis.
E, albicans. E. thr- pancreas.
Section 3 Micr,,biolosy and Pathology T 279

87. When the sputum samples were taken to the Test items 93-97 refer to the following Test items 98-100 refer to the following 99. The palhologJ· report reveals o characteristic
laboratory, what test did the doctor order to be test/et. test/et. patl.ern of tumo1' cells that is dassically
performed lo help make the diagnosis?
A 55-year-okl man presents with malaise and dys- associaJed with Burkitt ':s lymphoma. Which of
A Gram stain, ' A 3•year-old African girl presents in the emergency the foflowing describes this liistopathologic
B. Add~fast Main. pnea. He has a low-grade fever and reports that room with a palpable mass in her lower right pattern?
C, Spor~ stain. his shortness of breath has increased steadily mandible. She is currently in the United States vis- /!,., Honeyco1:1b
D. PPD test (tuberculin test). over the past week and a haif. He has a history of iting relatives with her parents. Her mom claims B Cobweb
E. Voges-Proskauer test rheumatic fever and denies ever using recre- that a few days ago she noticed a growing mass in C. Cotton wool,
88. Alter 2 weeks, tile bacterfol cultures came back ational drugs. He is currently being treated by a her daughter·s jaw. There appears to be slight D. Sun ray.
from the lab confirming the initial diagnosis, dentist for full mouth reconstruction, swelling around the area, although it is painless E. Starry sky.
positively identifying the organism 93. Upon further examination, a heart murmur 100. Tire African form of' Burkitt's lymphoma has
Mycobacterium tuberculosis. M. tuberculosis is and not tender to the touch, After further exami-
was detected. Given the patient's past medical been linked to the £1,stein-Rarr cirus. This
known to infect which oftlte follmving cells? history, u·hich heart valve is most likely nation. a biopsy was taken. and the diagnosis of virus is also responsihlr: for which nf the
A. Fibroblasts. affected? Burkitfs lymphoma was made. following diseases?
B. Basal cells. 98. Burkitrs lymphoma is a malignancy that
A Mitral valve. A Mononucleosis.
C. Type! pneumocytes. B. Tricuspld valve. affects which of the fallowing cells?
R Shingles.
D. i\'Jacrophages. C. Aortic valve. A. Macrophages. C. Chicl.:.~n pox.
E. Erythrocytes. R T lymphocyte-~.
D. Pulmonary valve D. Kaposi's sarcoma
89. Which of' the following is a glycolipid found on C. B lymphocytes
94. Be/'ore the patient's development of rheumatic E. Herpcmgina.
the surface of M. tuberculosis that plays a role fever, he likely suffered from which of the D. Neutrophil5.
in its pathogenesis? following conditions? E. Keratinocytes.
A. Cord factoL A Cystltis.
B. 0 antigen, B. Pharyngitis,
C. Protein A. C. Food poi5oning,
D. Exotoxin A. D. Thrornbocytopcni<l.
E. Leclthinase.
£. Meninigltis.
90, Since the patient was lfoing in a homeless 9.5. After further evaluation and tests, the patient
shelter, the tuberculin test was administered to i,'i diagnosed with snbacute endocardltis. JI' the
all of' the staff and residents living at the in/'ecting microbe was cultured in the
shelter. This test is based on a delayed type laboratory, the results would most likely show
hypersensitivity reaction that is mediated by that this microbe is po.sitive for _ _.
A o:-hemolysis
A. Only IgG 8, [>-h•moly•ls
B. lgG and IgM
C. "themolysi~
C. lgE
D. Coc1gulase
D. T cells and macrophage!: E. LecithinAse
E, Mast cells and bn.sophil~
96. Which of' the following is the most likely
91. Which of the following is the most appropriate complication that may occur from the
drug used in combination therapy for vegetations forming on the patient's defective
tuberculosis to treat the patient? heart valve?
A, Amoxkillin.
A. Myocardial infarction.
B, Clindamycin. 13. Hemorrhage.
C. Cephalosporin.
C. Petechiae.
D. Tetrncycllne. D. Cor pulmonale.
£. RHampin. E. Embolus.
92. After 3 weeks, the patient was feeling ''much 97. After the diagnosis is made, the patient is
better" and was discharged from the hospital, immediately placed on high-dose, IV antibiotics.
although he remained 01i his drug therapy for One of the antibiotics that is administered to
another 6 months. Which of the follon•ing best the patient is streptomycin, an aminoglycoside.
describes the calcified scar that later formed in The antimicrobial effect of streptomycin is to
the affected lung parenchyma and hUar lymph inhibit the .,vntltesis of _ _•
node? A. The bacte~rial cell wall
A. Gumma.
R Folate
R Chancre. C. Proteins
C. Metasti:ttir calcifications.
D Nucleic acids
ll. Tubercle. E. J1·lactamase
E. Ghan complex.
308 " Section 4 Dent,il Anatomy :md Occlusion SectiQn 4 Dente! AN1tomy tind Ocdusion • 309

Widmalm S, Lillie J, Ash M: Anatomical and elect;omvo- Zsigmondy A: Ghrundz(ige einer praktischen Methode
graphic studies of the lateral pterygoid mu,~le. zur raschen und genauen, Vonnerkung der zahnart~ SAMPLE 0UESTIONS
J Oral Rehab/I 14:429, 1987. zlichen Bf'!obachtungen' und Operationen. Dtsch
Woelfel JB. Scheid RC: Dental Anatomy: Its Releeance to V;schr Zahnhk 1:209. !861. J. There are several tooth numbering systems, 6. The primary mandibular first molar has tchich
Dentistry. Philadelphia: Lippincott Williams & Zsigmondy A: A practical method for n1pidly noting den-
some used more than others, and some used by of the following characteristics?
Wilkins. 2002. tal ob5ervation5 and operntions. Br 1 Dent Science dental specialties or by svecial organizations, A, Resembles other primary and permanenl te-eth.
World Health Organization: Oral Health Survev!: Basic 17:580, 1874.
The so-called universal system consists of: B. From the occlusal perspective. has a heart-
Methods (3rd edition). Geneva: The World Health
A. 1\vo--diglt sets of numbers for each tooth ln each shaped outllne.
Organizl\tion, 1987.
arch quadrant (e.g., 18 to 1 !). C. The mesiobuccal cusp is smaller than the
B. Single sequential number for teeth repeated ln <llstohuccal cusp.
each quadrant (e.g., 8 to l). D. No develop:nental groove ls evident between
C. A .sequential alphab~t letter for each tooth in an the buccal cusps.
entire dentition (e.g., A to T). 7. A comparison occlusally between the primary
D. Different symbols for each numbered tooth in mandibular second molar and the permanent
each quadrant (e.g., !ii to 11). mandibular fir.,;t molar shows which of the
2. From the occlusal perspectfoe, which tooth in following differences?
the primary dentition oaries the most in form of A The mestoM, distobucca!, and distal cusps of the
which tooth in the permanent dentition varies primary molar are almost equal in size; the
the most? distal cusp of the permanent molar is smaller
A. Maxillary first primary molar. than the other two cusps.
B. ~axillary second primary molar. B. The primary molar crown is wider
C. Mandibular first primary molar. buccolinguelly (in comparison with its
D, Mandibular second primary molar. mesiodistal measurement) than is the
3. The primary maxillary first molar has which of ptrmanent molar.
the following characteristics? C. The primary molar outlfne is somewhat
A. lt is larger in all dimensions than the primary hexagonal: the permanent molar ls rhomhoidal.
maxiUary second molar. D. The ratio of the crown/root length of both
8. All three roots can be seen from mesial molars is the same.
perspective. 8. In comparing permanent and primary teeth.
C. Bifurcation of roots begins almost immediately which of the following differences are noted?
at the site of the cervical line (CFJ). A. Crowns of anterior primary teeth are narrower
D. The mesial root is considerably ,horter than the mesiodistal!y (in comparison to their crown
distal one. length) than the permanent teeth.
4. From a mesial perspective, the crown of the R Comparatively, the roots of primary anterior
primary maxillary first molar has which of the teeth are narrower and longer.
following characteristics? C. Cervical ridge5 of enamel of the primary
A. Pronounced convexi1y on the buccal outline of .anterior teeth are les5- prominent.
the cervical third. D. Buccal and lingual ~urfaceE of primary molars
B, The cervical fioe mesially shows some curvature are less Hat above the cervical curvature than
in an aplcal direction. those of permanent molars
C. The dimension at the occlusal third is the same 9. The overall lengtlt of the primary teeth that are
as at the cervical third. given hf!re are the average range of dimen.sions
D. The mesiobuccal cusp Is longer and sharper with one exception. Which range for what tooth
than the me.1iolingual cusp. is not correct?
5. From the lingual perspecti£ie, the crown of the A. Ma-xillary central incisor, 16 to 17 mm.
primary maxillary second molar $lwws wftich of B. Mandibular central incisor, 16 to 17 mm.
the following? C, Maxillary lateral incisor, 16 to 17 mm.
A. Small, well-developed mesiolingual cusp. D. Mandihulbr lateral incisor, 15 to 17 mm,
B. Distolingua! (DL) cusp smaller than the maxlllary 10. For each type of tooth, the primary teeth
primary fir,t molar DL cusp. consistently show which of the Following
C. There is no supplemental cusp apical to the characteristics?
mesiolingual cusp, A. Greater mcslodi.stal diameter relative to crown
D. Developmental groove separating the height than permanent teeth.
me:5iolingual and <llstolingual cusps. R An elongat~d appearance of the primary
crowns and roots.
C. Crowns that are translucent white in color.
D, Root trunk length is one~ha!f of the crown
height.
31 0 T Secti~n 4 De:ital Analamy and Ocdusion Section 4 Dental Anatomy and Ocdusfon • 31 i

11. Which primary tooth is generally accepted as 18. Which of the Following are not type traits of 24. If jaw opening is divided into phases, and it is 3(). The ncclusa/ surface of a primary mandibular
the first to erupt, and at about what mean age? permanent maxillary molars? assumed that the surfaces of the articulating first molar often has a promine11t fadolingtwl
A. Maxillary central incisor, 8 to 12 months. first Second Third bones and disc are associated throughout jaw ridge. This trdnsverse ridge connects U'hich two
B. central lnc-lsor, 7 to 9 months. Molar Molar Molar openin& what is the relationship of the disc
C central incisor, 6 to 10 months. A. Buccal Widest Intermediate Smallest and cnndyle in the follot.t:ing phases? and distolingual.
D. Mandibular central incisor, to lO months. Vlew: molar width molar A. In the earliest phase, the condyle moves B. !\ lesio!ing11a! and <:listohucc.-1!
1

12. ,4 most favorable sequence of eruption far the B. DL cusp; S.amesize Same size Smallest forward the disc. C. lV:esiobuccnl and
permanent dentition is which of the following as as M 1 size B. ln the early phase, the disc and condyle move D. Dlstobur-cal and d1stnllngual
(right side)? (Eruption sequence given by C Occlusal More anteriorly in concert. 31. ·which premolar would be the most likely tt>
numbers in parentheses.) view: rhomboid rttomboidal or C. In an intermediate phase, the condy!e moves have a single valp horn?
A 3. 30; (2) 8, 25; (3) 7, 26; (4) 5; (5) 28, shaped forward at a slower rate. first
6; (7) 31, 2, D. MB root In Hne-with ln line with Roots D. In the flna! phase. the disc moves forward at first.
3, 30; (2) 8, (3) 7; ( 4) 27, 6; (5) 28, 5; apex: cusp tip crown dis- faster rate.
29, 4; (7) 31, center placed 25. Occlusal interferences can be defined by all of MaxilJary second.
30, 3; (2) 25, 8; (3) 26, 7: (4) 27, (5) 4; the following except· 32. ft'hlch one of the following is not a normal
29, 6; (7) 31, 2, 19. Which of the following is not an arch trait of
the maxillary canine? A. Oc:dusal contact relations that Interfere with anatomical feature of mandibular incisors?
30, 3; (2) 25, 8; (3) 25, 7: (4) 27, 6; 28, 5; function. A. Bifurcated roots.
A In the same dentition, the crown is larger than
29, 4: (7) 2, 3L B. Interference to jaw closure into the intercuspal B
the mandibular canine.
13. primary tooth generally erupts last? C Four rle'v~l,omnc,,tal
B. The indsa! margin of the crown
A. Mandibular second molar. C. 1nt,er1<2renc,,s to laterotrusive movements. D. Inds.al edges placed .:-lightly lingnally.
least one third to one half of (TO½-'fl
B. Maxillary second molar. D. lnterferences to jaw opening. 33. The heights of contour of the distal surfaces of
C. canine. Labial aspect: mesial and distal ridges
toward cervix, 26. If posterior teeth on the left side contact permanent mandibular central incisors are
D. M,mo10111m canlne. occlusally during a right lateral excursion of located in which coronal third?
D. symmetry of mesial/distal halves when
14. Comparing the overall length of primary the mandible., the left side occlusal contact Middle.
viewed from inclsal.
central incisors (Ei f) with permanent would be referred to as: B. CervicaL
maxillary central incisors (Bl 9), which is the 20. Which one of the following morphological A. Laterotrusive contact. C. OcdusaL
correct ratio expressed as a percentage? characteristic is representatfoe of all posterior B. Protrusive contact. D. lncisaL
A. About 50'.Y., maxillary teeth? C. Mediotrusive contact. 34. On avera~, approximately what is the
B. About 60%, A. Marked mesial concavity on crowns and roots. D. Centric relation. dimension of the permam:nt maxillary canine at
C. About 70%. R Tips of cusps are well within the confines of the 27. Which one of the following is considered a the ttJldest mesiodistal diameter of the crown?
D. About Sm,,. root trunks. primary ligament of the TA-fJ? A mm.
15. At what time is the crown completed for the C. From me-sial/distal aspect, crowns are A Stylomandlbular, B. 6.5mm.
tooth indicated? rhomboidal in shape. B. Sphenornandibular. C. 7.5 mm,
A Primary maxillary centra! incisor, 3 weeks. D. From mesial/distal aspect, all maxillary C. Stylohyokl. D_ 8.5 mm.
B. Permanent maxillary central incisor, 2 to posterior crowns are trapezoidal with shortest D. Temporomandibular. 35. Which one of the follmdng is found on the
3 years. uneven sl<le to\.vard ocdusal surface. 28. }J,fand1'bular movement resulting from occlusal crown of permanf'nt mandifmlar first mol<irs
C. Primary maxillary lateral incisor, 2 to 3 months, 21. In terms of vertical dimension, where is the contacts of the teeth from retruded contact but is not found on the crowns of mandibular
D, Permanent maxillary lateral incisor, 2 to 3 mental foramen found most frequently? positiort (CRC) to intercuspal position (slide in second molars?
years. A. At the of the premolars. centric) ma)' show all except one of the MB cusp.
16. When are the crowns of the primary maxillary B. to the apices. following directional components when viewed B. Distobucca) groove.
second molars completed? Below the only in the horizontal plane. Which is the C. groove.
A 11 months. 0. No location predominates. exception? D. cusp.
B. 10 months. 22. A major anatomi<:al variant of the two-rooted A. Vertical component. 36. The Y..shaped central derelopmental groove is
9 months. B. Horizontal component. most likely found on which of the following
mandibular molar is a tooth with an
D. 8 months, L-:1teral component. premolars?
additional distolingual and third root. What
17. Whic/1 of the fol/ou,ing is not a type trait of the is the prevalence of these three•rooted 0, Protrusive component. A.
peYmanent maxillary second premolar? mandibular first molars? 29. Where is the height of contour located relative B. 'lfanr!;hnl:ir first.
A Buccal view: narrow shoulders of A. May exceed 10'.){, in Caucasians. to the following teeth (viewed from the mesia/)? C
crown; mesio-- and w,,cruucow,u, B. Less than 1% in Eurasian and Asian A. Facial surfaces of aH molars, middle third.
8. Ocdusal table outline: ovoid. B. Lingual surfaces of all premolars and molars, 37. In a cusp-embrtu.mre relationship, the
cervical third. maxillary first premolar is most likely to
C. Lingual s:urfac:es of molars and premolars, articulate with which of the follmDing
profile is not visible. cervical or mlddle third. mandibular teeth?
D. Anterior teeth, cervical or mldd!e third. First premolar only.
23. Which jaw activity does not involve one of the
B. Fir.st molar only.
following muscles?
C. Canine and first premolar.
A, Clenching, superior heads of lateral pterygoid
D. First :1nd second premolars.
muscles (LPM).
B. inferior heads of LPM.
C. ja\v movements, inferior heads of LPM.
D. jaw opening, deep masseter muscle.
312 • 54::<.tion 4 Dental Anatomy and Ocduslon Section 4 D1:ntal AMtomy and Ocdufion -,, 313

38. Equal contracture of the lateral pterygoid 42. Exacerbation of bruxism has been reported to 49. Extraction of tooth 32 r<'vealed ,,nached soft 56, From the occluHa/ asp<.:ct the primary maxillary
muscle bilaterally produces which of the occur 1i:ith all the following agents except one? tissue. Which of the following is most important second tno1ar has which of the fo-1/orring
fvllo111ing mandibular mrn,ements? Which Js the exception? for a presumptive diagnosis of tuberculosis? characteristics?
A. Retrus\ve. A Pn:rm:etinc (Paxil\ A. Cll6t'-ous n<:>crntic areas. A. Somnvhz1t rhombolrla! in form.
B. Elevation. B. Selective serotdnin reuptake inhibitors. B. Acld~fast bacilli. B. Three well d{;'vdoped cusps.
C. Protrusive C Naproxen_ ( Napro3yn ), C Epithelioid his.tiocytes. C. Two 51;pp!em,,ntal cusps. including tubercle ol
D. LaternL D, Amphet.11lline clt'iivativ~ ("Ecs~asy"). D. L,ngcrhans giant ce!ls. Carabelli.
43. Migraine is a form of headache that is 50. Which one of the following disorder.,; is the D. Poody defined mesia! triangular fossa.
currenrly thought to be best understood on the least likely to be a differential diagnostic factor 57. From the occlusal aspect, the primary
basis of all the following with one exception. in the patient's limited jaw opening? mandibular second molar has which of the
Test items 39-51 relate to the following
The ~xception is which of the following? A. Exacerbation of TMJ and TMD. following characteristics?
case presentation.
A As a dysfunction of braJnstem pathways or B. Tris mus secondary to TMJ pain. A Somewhat rectangular in form.
A 35-year-old woman presents with a painful lim- diem::ephallc nuclel. C. Myalgia secondary to TJVlD. B. The outline of the crov,Ti converges meslally,
itation of jaw opening (28 mm), a painful tooth 13. As a primary disorder of the brain. D. Myositis secondary to bruxing. C. Three buccal cusps are dissirnil.:rr in siz~:.
on the right side, and swelling at the angle ol the C. As similar in mechanism to tension headaches, 51. The patient is on an anticoagulant drug D. Cusps do not have well defined triangular
D. As. a neurovascu!ar hec1rlache. (e.g., warfarin /Coumadin}) as u;ell as ridges.
jaw. She has a history of temporomandibular dis-
44. Three key factors in the pathogenesis of pain in rifampin. What is the effect of rifampin on the 58. A comparison of the pulp chambers and
ease (TMD) and conservative treatment. :Vledica! anticoagulation effect ofwarfarin (Coumadill)?
migraine are usually considered. Which of the root canals of maxillary primary and
history reveals that the patient is being treated A. Increases the anticoagutant -effect of warfarln,
following is not considered to be a key factor? permanent s<>cond molars show; which of
for tuberculosis with combination antituberculo- B. Increases the cyclic conversion of vitamin K the following?
A. Cranial blood vessels.
sis drugs including rifampin (Rifadin). She is also B. P,-.~111yloid-containing plaques in the brain. e.poxirle reductase. A. Enamel cap of primary tnoth is relatively thick
being treated with an anticoagulant. warfarin C, Trlgeminal innervation of the vessels. C Antkoagulation effect Is inhihited. but less consistent in d~1)th.
(Coumadin), a low level of aspirin, and paroxe- D. Reflex connection of trige-rninal system with D. Decn:asP,S its metabollc clearance by inducing B. ComparnUvely le1.s thirkness of dentin at the-
tine (Paxil) for depression. The intraoral exami- crnnia l parasympathetic outflow. activity of hepatic oxidases. occlusal fossa of primary molars,
nation shows extensive teeth wear from 45. The treatment of the patient with low levels of C. Pulp chambers are proportionally larger in
bruxism, diagnosed by a sleep specialist as sleep aspirin is done for which of the following 52. A transverse ridge is: primary molars.
bruxism. Tooth 32 has a deep carious lesion and, reasons? A The CDmbination name for jolning oblique and D. Pulp lioms are lower in primary molars.
A. To reduce the likelihootl of platelet aggregi,,tion. trlangular ridges especially distal horns.
on radiographic examination, a periapical radi-
B. To stimulate cyclo-oxygentlse in the platelet5. B. A combination name for jointng buccal and 59. In a comparison of maxillary, primarv and
olucency.
39. The incidence of tuberculosis is increasing as a C. To increase the formation of thromboxane. lingual cusp triangular riclge.i permaneat second molars, which of tire
D. To cause p!ateJets to regenerate cyclo- C. Chnracteristically found on all primary and following <liffercnc-es are noted?
result of an association with AIDS. Oral
oxygenase. permanent molars A. Enamel rods at the cervix slope gingiv;-1Hy in the
infections of tuberculosis (TBC) do occur but
46. When there is a pulpal-periodontal infection of D. Found occasionally on maxlllary primary first primary molar.
are uncommon. Diagnosis of oral lesions may
a mandibular third molar, which of the molar B. En~niel rods at th~ cervix slope occlusallv in
present several challenges, as set forth in all of
following listed facial and c,:,rvkal spaces is 53. From the occlusal aspect, the primary the permanent molar. "
the following statements except one. Which one
most likely to have become infected when there maxillary first molar has which of the following C. Buccal cervical ridges are less pronounced in
of thl? follmring statements is false?
is swelling at the angle of the jaw? characteristics? the primary molar.
A. Lesions secondary to HIV may be present.
A Retromolar space. A Crown outline di'V~rge:s llngunlly and di5t<llly. D. Roots of prlmary teeth ate longer and more
B. Isolation of M. tulH:rculosis by culture requires 4
B. Submaxi!!ary space. B. Small traverse ridge freqllently pn•sent <:allecl slender ln comparison wlth crown size than
to 6 weeks or longer.
C. Submasseteric space. an obllqtt<:' rldge. those of permanent teeth.
C. Mycobacteria can be demonstrated by special
D. Parotici space. C. Four tusps are present. 60. Based on average MD diamettirs of the crowns
stains in only 27'J:, to GO'Xt of cases.
47. Lymphatic drainage from tooth 32 will first D. Mesia! marglnal ridge ls thin and poorly of primary teeth. the range for ave,-age overall
D. Molecular tests ( e.g., polymerase chain
in1Jolve which of the following node groups? f!eveloped. length of the primary maxillary arch is about
reactlon) show slow turnaround times.
A. Lateral upper deep cervical node. 54. From the lingual perspective the primary what dimensfon?
40. All the following side effects hm;e been
13. Medial upper deep cervict,11 node. maxiJlary first molar has which of tlte following A. 60-68 mm.
reported to be related to the use of rifampin
C. Lateral lower deep cervical node. characteristics? B. 68-76 111m.
except one. Which is the exception?
D. Submaxil!ary node. A. Distolint,n,rn! cut.pis the most prominent cusp. C. 76-84 mm.
A Green bocmy fluids-sweat, tears, urine,
48. All but one the following are considerations B. iV1e:siolingual cusp poorly dt>fined D. 84-H2 mm.
B. liepatotoxicity.
relevant to the diagnosis and treatment of C Distoburcal cusp cannot be !!een fr-om lingual 61. What is the average height of curvature of the
C. Thrombocytopenla.
tuberculosis. Which of the following statements aspect. cervical line (CE.I) on the mesial and distal of
D. Rashes.
is not true? D. Crown converges -conslderably in a Hngua! the permanent maxillary and mandibular
41, Which of the following modes of action does
A. Increase in the prevalence nf TB. direction. incisors?
not relate to rifampin?
B. Orn! TB lesions occur most frequently on the 55. The primary maxillary second molar has what A ;\bout 35 mm nn the mesial of the muxi!!arv
A. lnhlbits Ri'\A synthesis.
glngivaL characteristics? central incisor.
B. Binds tightly to eukaryotlc RNA polymerase.
C. F.merg('"nce oi rnultidrug-resistant strains. A Does not have a well-defmed rnesial trlan~ular B. About l.5 mm on the cHstnl oi the rnaxillary
C. Tuhcrculoeida.l to intracellular and extrace!iu!ar fossa. ~
orgnnisms. 0. Hi~h risk of M. tubernrlo,'>i! infection in patients central incisor.
infected with human immunorleficlencv virus B. Oblique ridge ab:c;e-nt or not well developed. C. Ahou: 2.0 mm on the nwsird nf tlw m<HHiihul;lr
D. Reduces activity of hepati<: mixed-function
(HIV). , C, Devl!':lopment ( centrn!) groove ls well defined. central lncis:or,
oxidases.
D, A tubercle of Carflb~•m (Supplemt>ntary cusp) is D About I.() mm on the di~tnl of tht'. rn,,mdllmlar
well dcvl?lnped. central incisor.
Section 4 Dental An.itomy and Ocd:.ision • 315

62. Considering the period of 2i1 months to about 6 69. W'h{ch of the following a,-e NOT arch trails of 75. Recent focus on causatii,e factors in bruxism 82. Rest position (RP) is defined:
years of age which of the following is true? tile canines? include ALL of the following Except? A. As any position of tile nrnncllh!e that lacks
A. Not all the primary teeth have attaif!ed their :\-!axillary Mandibular A Occlusal interferences. contact of the teeth.
occlusal ;evel. Canine (6) Canine (27) B, Part of a sieep arousal response. 8, As the centric relation position of the fOVi.dyles
B. Parts of hoth Jaws contain\ng primary teeth A. Crown size ,Large,- (same Smaller (same C. Palhophysiological factors. with the teeth apart.
change noticeably in size. dentition) dentition) D N<>urotransmitters in the centrai nervous C. As a man<libulnr position with mastlcatory
C A significant increase in intercanine width B. Lingual Common None system. muscles at complete rest
occurs shortly before and during the time the pits/grooves 76. AU of the follou:ing are supporting cusps D. A.s a dinical mandlbu!nr position in relation to
primary teeth are lost an<l the permanent teeth C. L.1biolingual Near cervlx Near cervix EXCEPT? the lnterocclusal space.
emerge. diameter same as 27 same as 6 A. Buccal cusp of Tooth #29. 83. Name the point angle which represents the
D. Denta! arch form is more or less constant and D. lnclsal view, Symmetrical Marked B. Lingual cusp of tooth# 4. junction of the cutting edge of an indsor with
prnctically no dimensional changes take place M;D halves asymmetry C. ML cusp of tooth #3. the surface that is toward the tongue and the
in depth or width after 9 month~ of age. 70. Which of the following are NOT type traits of D. ML cusp of tooth# 19. surface that is away from the midline.
63. Which sequence of eruption of permanent teeth the permanent maxillary central and lateral 77. Measurement of horizontal overlap (over jet) of A. Distoproximoincl~aL
occurs most onen? (S.7-6,.54-3•2·1 ,tl3•M:rM,·P2 • incisors? the teeth is easily done by which of the B. Distolabioincisal.
P1..C•Ll·CIJ. First #, # 's in each series is following methods? C. DistolinguolncisaL
Central Lateral
considered to be the first to erupt. A Measure from the fadal surface of a mandibular D. Labioincisol\nguo.
Incisor Incisor
A 6-1-2-4-3-5-7-8 (Maxilla} incisor to the facial surface of a maxlllary 84. Root bifurcation would be a more likely finding
A. Labial view: lncisat 3rd Jnct. inclsal/
B. 6-1-2-3-4-5-7-8 (Maxilla). incisor with the subject in lntercuspal position. in which of the following permanent teeth?
Meslal Contact mid 3rd
C. (&-1)-2-4-3-5-7-8 (Mandible). B. With the 5ubject intercuspal position, mark the A. Maxillary canine.
B. Labial view: Jnct. incisal/ Middle third
D. l-6-2...j-5-3-7-8 [Mandible). position of the maxillary incisal edie on the B. P.fondibular canine.
Distal Contact mid 3rd
64. Tile firSt evidence of calcification (weeks in facial surface of the mandibular inch,or with a C Mnxillary central incisor_
C. Mesia! view: Within inclsal Jnct. incisal/
utero) in the primary dentition occurs in which pencil. D. Mandibular laterai incisor.
contacts third mid 3rd
of the following teeth at about what age? Then have the subject open the mouth and 85. What is the correct schematic outline of the
D. Lahial: Slightly Sharp rt.angle
A Maxillary central incisor-14 (13-16) weeks. measure from the mark that you made to the following teeth?
mesioinclsal angle rounded
B. Mandibular central incisor-12 (10-13) weeks. indsal edge of the mandibuhu incisor. A. Mandibular premo!<1rs, vlewed from occlusal.
71. Which posUion of the mental foramen relatitie
C. Maxillary lateral incisor-14 (13-16) weeks, C. Measure from the miclline between the rhomboidal.
to the mandibular premota,·s and ffrst molar
D. Mandibular lateral incisor-14 (13-15) weeks, maxillary central lncisors to the mid!ine of the B. l\.faxill:iry central incisors, viewed from f.:iciaL
occurs most frequently?
65. A most characteristic feature of the primary mandibular central incisors. triangles.
A. Between the first and second premolars.
maxillary central incisor is which of the D. Measure from the incisal edgt of a maxillary C. Moxilla.ry lateral in<:isors, viewed from mesial.
B. In line with the second premolar.
following? incisor to the incisal edge of a mandibular tnipezoidai.
C Distal to the second premolar.
A. Faclolinguat breadth of the crown, incisor with the mandible in the maximum open D, All mandibular posterior teeth. dlstal aspect,
D, In line with the mesial root of the first molar,
B. Mesiodistal width of the crown. position. rhomboidal.
72. The maxillary sinus overlies the alveolar
C. Mesial and distal margin outlines in line with 78. In a cusp•fossa occlusal relationship, the 86. A distinct central developmental grooue,
processes in particular what tee/Jt?
profiles of root. maxillary second premolar is most likely to prominent buccal triangular ddge, two cusps
A. First and second maxillary molars.
D. Root/crown ratio. articulate with tvhich of the fallowing and distinct mesial and distal occlusa( pits
B. All maxillary molars.
66. When is the crown of the permanent mandibular teeth? would be mo!~t characteristic of:
C, First and second premolars.
mantlibular second molar completed? A. First premolar only. A. Mandibular Hrst premolars.
D. First and second premolars and first and
A. About 7-8 years, B. Second premolar only, B. Primary mandibular first molars.
second molars.
ll. About 8-9 years. C. Canine and first premolar. C. Primary m;J;ndibu!ar second molars.
73, The masseter muscle, which has a complex of
C. About 9--10 years. D. First and second premolars. D. Mandibular .second premolars.
internal components, includes all the following
D. About 10-11 years. 79. Which of the following contributes primary 87. From the incisal view. a greater mesiodistal
£XCEP77
67. Which of the following is NOT a type trait of sensory innervation to the temporomandibular measurement than faciolingual measurement
A. Pennation.
the permanent maxillary first premolar? joint? can be seen in idiich of the following
B. Structural composition permitting regional
A Ocdusal table outline, trapezoldaI. A. Auricu!otemporal nerve. permanent anterior crowns?
activation.
B. Generally two roots - mesiai and distal. R Infra.orbital nerve. A. Maxi!Iary central incisor
C. Mnltlp!e internal aponeuroses.
C. Central groove is long. D. Internal aponeuroses that do not move or C. Branch of the lingual nerve. B. Mllxillary canine,
D. Supplementary grooves are rare. D. Facial nerve. C. M;ilmtibular canine.
deform.
68. Which of the following are NOT type traits of 80. The smallest cusp on the crown of a 5 lobed D. \fandlbulnr centrnt in<:'isor,
74. Sleep bruxism can be characterized by which
permanent mandibular first and second mandibular second premolar is tile: 88. ,ffaximum rotation and translation of both
of the following?
premolars? A. Buccai cusp. cond;yles takes place at:
A. Episode~ of massive, bilateral clenching.
first Sf:cond B. Tooth grinding that may last for uµ to 20 B. Di~tohuccat cusp. A. :Vfoximum opening.
Premolar Premolar minutes. C. Distal cusp. B. Maximum prntrusive.
A. Buccal vlew: Crown bilate-rally Bilaterally D. Di5toling:ual cusp, C Right nm! left lutf'rnl t:xcurs:ive movf'nlf'l!ts.
C. Often coincides with passage from lighter to
A5ymmetrical symmetrical deeper sleep.
81. A typical primary mandibular first molar has D. Hinge movemf:'nL
B. Lingual aspect· Entire bucca! Buccal profile D. Occurs ;t_pproxlm21tely every 20 minutes in the how many pulp horns?
profile vis:lble not seen sleep cycle. A. l.
C. Lingual aspect: Most of ocdusai Little, if any 8. 2.
surface visible seen C.
D. Lingual aspect: Contour height: Cervical third D. 4.
Middle third
316 v Section ff o~t,1 Anatomy and Ocd,.rnion Section 4 Dental Anatomy and Ocdusion T 31 7

Questions 89-1 00 relate to the foffowing 91?. The differential diagnosis of while ''spots'' of 97. Aggravation of bruxism has been suggested to 99. During muscle contraction what physical
history and examination the enamel of primary and permanent teeth occur secondarily to all of the following change DOES NOT occur relative to muscle
should include disorders that have a occlusal relationships except one. f+'hicfl one fiber contraction?
A mother brings her 12-year-o!d boy to the dentist substantiated cause. 'Which of the following DO is the EXCEPTION? A. Sarcomeres-shorten,
to ask about her son grinding his teeth and about NOT have an evidence•based causal A Occlusal lnterferences in centric relation. R Thick and thin filaments-shorten,
some white "spots" located on the smooth-sur- relationship u:irlt enamcil hypoplasia? B. Occlusal Interferences in the intercuspa! C. 1 Band-shortens.
faced enamel of several of his anterior teeth and A. Rickets. po~ition, D. H zone-shortens.
premolars. Among other questions about the 8. Congenital syphilis. C. Iatrogenic occlusal relations that imerfere with 100. Regarding the superior and inferior heads of
cause of the defects, the mother then asks when a C. Measles. bruxism. the lateral pterygoid muscle (SHLP and IHlP),
systemic disturbance occurred that may have D. Fiuorosis. D, Angle Class Ill malocclusion (prognathism). which of the following statements is NOT
93. Clenching and grinding of teeth involves 98. The differential diagnosis of enamel TRUE?
caused the "spots." The patient has excessive
contraction of skeletal type muscles. Several hypoplasia should lake into account A. Hypothetically, SHLP and IHLP can be
tooth wear from bruxing and clenching. The den- types of myofilaments are present in the suggested differences between non.fluoride considered to be parts of one musde.
tist measures the cervical-incisal length of the per- contractile efement,'i of skeletal muscles. Which and fluoride opacities. Which of the following B. Distributions of SHLP and IHLP activities are
manent maxillary central incisor (10.5 mm). Also statement abo.ut muscle filaments IS NOT true? statements does not suggest a basis for a shaded according to biochemical demands of
measured ls the distance from the CEJ to the mid- A Mvosin forms the thick filament of muscle. ,liagnosis of non-nu.oride (NF) enamel tasks.
point of the defect (5.5 mm). Given that the crown B. A~tin is a major protein of thln filaments. hypoplasia? C, SHLP stabilizes the condyle and disk against
is completed over a period of 4 to 5 years it is pos- C. Tltin ls a protein of elastic filament. A. Levels of Fin drinking water that range from the artlcular eminence in a wide range of jaw
sible to estimate the age at which the hypoplasia D. Connectin is a protein of intermediate filament. 0.2 to 0.34 ppm have been reported to be movements and forces,
occurred using 6 months or yearly periods in the 94. 711e clinical examination of the patient reveals associated with prevalences of NF enamel D. tHLP plays a m1.jor role in the generation and
following formula: extensive wear of the right canines, and opacities ranging from 22% to 35 %. fine control of horizontal forces.
somewhat less wear of the lateral incisors. B. At a level of l to l.5 ppm of f in drinking
ADF • ACF - (yrs. of formation/crown height x Also there is tenderness of the jaw closing water, only few F opacities occur.
distance of defect from CEJ) muscles, particularly on the right side. The C. Most NF enamel opacities appear as white,
muscle(s) that would be invoked primarily in opaque !.pots in smooth surface enamel; areas
89. Using an average age of crown formation (ACf) providing most of the force for anterior tooth of mild dental nuorosis are lusterless, opaque
of both 4 and 5 years, the age of defect
clenching include which of the following white patches.
formation (ADF) is estimated to be about what
masticatory muscles? D. Fluorosis and NF opacities are clinkally
time? A Inferior lateral pterygoid muscle, significantly different.
A. 7-9 months in utero.
B. Superior lateral pterygoid muscle.
B. 0-1 years of age.
C. Anterior ternporali~ musde.
C. 1-2 yearo.
D. Masseter musc!e.
D. 2-3 years,
95. Sleep bruxism (SB) is defined by many bat not
90. The increase of lluorosis of permanent teeth in all of the following characteristics. Which is
both nonfluoridated and optimally fluoridated
the EXCEPTION?
populations points to the need for dentists to A. Stereotypical movement disorder.
caution parents with children about potential B. Grinding and clenching of the teeth during
causes of f1uorosis in children. Which of the
sleep.
following cautions about fluoride is correct,
C. More frequent in the younger gener2'-tion.
but the age or implied age is NOT Correct? D. lndivlduals who brux during the daytime
A Excess (> 1 ppm) fluoride in the drinking water
inevitably brux at nlght.
during enamel formation,
96. Recent physiologic evidence suggests that
B. Exceasive (> pea-fiized amount) use of fluoride
central and/or autonomic nervous systems
toothpaste under 6 ye~rs of age.
rather than peripheral sensory factors play a
C. Use of fluoride toothpa!le !or children under 4
dominant role in the genesis of sleep bruxism
years o[ age.
(SB). Which statement about the central genesis
D. Use oi a 1.1 % sodium fluoride toothpaste or gel
of SB is NOT true?
by pediatric patients only when 6 years of age A. During sleep the mouth is usually open due to
and older.
motor repression.
91. Systemic etiologic factors that are said to be R Tooth contact most Hkely occurs in association
a.uociated with enamel defects such as wlth sleep arousal.
hypoplasia occur generally in what period of C. Some peripheral sensory factors may exert an
time? lnfluence on SB through their interaction with
A. Before birth.
sleep a,;,vake mechanisms.
4

B. Generally after birth and before the age of 6


D, Sequential change from autonomic
years. ( cardiac)/brain cortical activities follows
C. During the first year postpartum for SB~related jaw motor activity.
Hutchin~mn'e; incisors.
D. During birth.
1. AU of the following are supporting cusps 5. Which of the follou•ing statements regarding
EXCEP17 tubular ,-eabsorption is true?
A. Buccal cusp of Tooth #29. A. l'vlost calcium filtered is pa$sively rca bsorbed
B. Lingual cu,p oi tooth !14. and not regula.tecl under any c-orn1itiong,
C. ML cusp of tooth #3. B. Most urea i, reabsorhed passively and is
D. ML cusp of tooth #19. unaffected by regulatory mechanisms.
2. The lateral pterygoid muscle attaches to which C. Glucose is reabsorbed by secondary <lcUve
of the following? tran5port and facilitated diffusion.
A Lateral surface of the lateral pterygoid plate. D. Most filtered phosphate is reabsorbed in the
8. Medial surface of the lateral pteryg_oid plate. collecting ducts and l~ unaffected by regulatory
C, Lateral surface of the medial pterygoid plate. mechanisms.
D. Medial surface of the medial pterygo!<I plate. 6. The class of antihodie,,; that plays an important
E. Pyrarnldal process of palatine bonc:. role in type I hypcrsensitit>lty reaction ..; is _ _.
3. A transverse ridge is: A. lgA
A. The cornbinatlon name for joining oblique and B. lgD
triangular ridges C. lgE
B. A combination name for joining huccal and D. IgG
lingual cusp triangular ridges £. lgM
C. Charact eri,tically found on an primary and 7. The amount of cytosine tdll be eqtu1l to the
permanent mo!ars amount of guanine in which of the following
D. Found occasionally on maxillary primary first molecules?
molar A. DNA.
4. Bronchogenic carcinoma is a complication most B. Rf\A.
characteristic of which of the following C. D:-IA and RNA.
conditions? D. mRNA.
A. Silicosis. 8. The hrartchial arche:~ disappf.'ar when the
B. Asbestosis. branchial arch grou:s down to contact the
C. Anthracosis.
D, Sarcoldosis. A se-eon<l; third hranchial arch
E. Bronchiectasis. B. second; flfth brnnchiai arch
C. thlr<l; fifth brand1iaf arch
D. first; first bnmchial groove
E. first: sixth hranchial groove

319
Sample Exam .,.- 32 J
3 2 O -. Samp-le exam

15, Measurement of horizontal ot:erlap (over jet) of 20. Which of the follou:ing structures is the most 28. Both active transport an<l faci(itated diffusion
9. Glucagon will decrease which of the
the teeth is easily done by uthich of the common site for oral cancer? are characterized by which of the following?
following? A. Soft palate.
following methods? .. A. Transport in one direction only,
A. Glycogenolysis B. Lateral border of the tongue.
A Measure from the facial surface of a mandmuiar H. Hydrolysis ol ATP.
B. Gluconeogenesis. \,,, Lower Hp.
C, Glycogenesis. Incisor to the facial surface of a maxillary C. Transport against a co;1centration gradient.
inclsor with the subject in !ntercuspal position, D. Floor of mouth. D. Competitive lnhihH\on.
D. Blood glucose. E. Buccal mucosa.
JO. When an enzyme ts competitively inhibited, B. With the sub\ect intercuspal position. mark the 29. Which of the following bacteria would be
position of the maxillary indsaI edge on the 21. :Wandibular movement resulting from occlusa{ expected to first colonize onto plaque?
which of the following changes occur?
facial surface of the mandibular incisor with a contacts of the teeth from retruded contact A, Streptococci.
A. The apparent Km is unchanged.
pencil. Then have the subject open the mouth position (CR(] to intercuspal position (slide in B. Bacteroides.
B. The apparent Km is decreased.
and measure from the mark that you made to centric) may show all except one of the following C. Fusobacterium.
C. Vmax is decreased.
the indsal edge of the mandibular incisor. directional components when viewed only in the D. Attinomyce.~.
D. Vmax is unchanged.
C. Measure. from the midline between the horizontal plane. Which t's the exception? E. PrePotd!o.
J 1. A typical primary mandibular first molar has
maxillary central incisors to the mldline of the A. Vertical cornponenL 30. Cell membrane typic-ally contain the following
how many pulp horns?
mandibular central incisors. B. Horizontal component. compounds except:
A 1. C Lateral component
D. Measure from the indsal edge of a maxillary A. Phosphotipids
B. 2. D. Protrusive component.
incisor to the ine:isal edge of a mandibular B. Proteins
c. 3,
incisor with the mandible in the maximum open 22. Which of the following muscles is responsible for C. Cholesterols
D. 4. the formation of the posterior tonsillar pillar?
position. D. Tria<:ylglycerols
16. Increasing the radius of arterioles will A. Stylopharyngeus. E. Sphingollpids
increase which of the following? R Tensor veli palatine. 31. Which of the following muscles adducts the
Test items 12-14 refer to the following A. Sv,tolic blood pres~rnre. C. P.alatoglossus. vocal cords?
test/et. B. Diastolic blood preMure. D. Palatopharyngeus. A. Lateral cricoarytenoid.
C. Viscosity of the blood. E. Levator veli palatine. B. Posterior cricoarytenoid.
A 3-year-old African girl presents in the emergency 23. Which of the following does not occur to C Cricothyroid.
D. Capillary blood flow.
room with a palpable mass in her lower right 17. Which of the following nerves supplies taste compensate for a fall in blood pressure below D. Vocalis.
mandible. She is currently in the United States vis- sensation to the anterior two-thirds of the normal values? E. Tensor veli palatine.
iting relatives with her parents. Her ~om daiI~s tongue? A. lncreaserl cardiac output. 32. Which of the following best describes
that a few days ago she noticed a growmg mass m A. Hypoglossal. B. Increased stroke volume. anaplastic cells that have not invaded tile
her daughter's jaw. There appears to be slight B. Glossopharyngeal. c_ Increased heart rate, basement membrane and are confined within
swelling around the area, although it is painless C. Lingual. D. Decreased total peripheral resistance. their epithelium of origin?
and not tender to the touch. After further exami- D. facial. 24. Relative or absolute lack of insulin in humans A. Dysplasia,
nation, a biopsy was taken, and the diagnosis of E. Mental. would result in which one of the fallowing B. Hyperplasia.
18. Which of the following are NOT arch traits of reactions in the lit;er? C. MetapJasia.
Burkitt's lymphoma was made. A Increased glycogen synthesis.
J2. Burkitt's lymphoma is a malignancy that the canines? D. Sarcoma.
Mandibular B. Increased gluconeogene:si,;. E. Carcinoma in situ.
affects which of the following cells? Maxillary
C. DetreAsed glycogen breakdown. .13. Which muscle of the anrerolateral abdominal
A Macrophages. Canine (6) Canine (2.7)
D. Increased amino acid uptake. wall is described as being belt• or strap,like?
B. T lymphocytes. A. Crown size Larger (same Smaller (same
25. In 2% glutaraldeh_yde, which of the following A. External oblique muscle.
C. B lymphocytes, denhtion) dentition)
times is min,lmally sufficient for achieving R internal oblique muscle.
D. Neutrophils. R Lingual Common None
sterilization? C. Transversus ahdomlnis muscle.
E. Keratinocytes. pitsigrooves A. 15 minutes. I} Rectus abd0mi11is muscle.
13. The pathology report reveals a characten'stic C. Labiolingual Near cervix Near cervix B. 1-2 hours. E. Quarlratus lumborum muscle.
pattern of tumor cells that is classically diameter same as 27 same as 6 C. 6 hours. 34. In a comparison of maxillary, primary and
associated with Burkitt's lymphoma. Which of D, h1clsal view, Symmetrical Marked D. 12 hours. permanent second molars, which of the
the following describes this histopathologic M/D halves asymmetry 26. Which of the following bones is part of the following differences are noted?
pattern? 19. ff'hich of the follawing statements regarding superior wall (roof) of the orbit? A. Enamel rods at the cervix slope gingivally in the
A. Honeycomb. the regulation of gastrointestinal motility is A. Zygomatk, primary molar.
B. Cobweb. true? B. LacrlmaL B. £name! rods at the cervlx slope occlusally in
C. Cotton wool. A. Sympathetic stimulation inhi~its _motlHty. C. Spbenoid, the permanent molar.
D. Sun ray. B. Parasympathetic stimulation mhtbits motility,
D, Maxilla. C. Buccal cervical ridges <1re less pronounced In
E. Starry sky. C. Gastrointestinal motillty is not influenced by E. Ethmold. the primary molar.
J4. The African form of Burkitt's lymphoma has the centnil nervoos system (CNS). 27. On average, approximately what is the D. Roots of primary teeth nre longer r:md more
been linked to the Epstein-Barr vfrus. This virus D. Gastrointestinal motility is not influenced by
dimension of the permanent maxillary canine slender in comparison with cro\-vo size than
is also responsible for which of the following hormones. at the widest mesiodistal diameter of the those of permanent teeth.
diseases? crown?
A. Mononucleosis.
A. 5.5 mm.
B. Shingles. B. 6.5 mm.
C. Chicken pox. C 7.5mm.
D. Kaposi's sarcoma. D. 8,5mm.
E. Herpangina,
Sample exam • 323
322 • Sample Exam

41. All of the following local chemical factors will 49. There are seu:,-al tooth numbering systems, 54. During the examination, the patient observes
Test items 35-37 refer to the following
cause vasodilatation of the arterioles, except: some used more than others, and some used by that he cannot feel it when you touch part of
test/et. dental specialties or by special organizations. his cheek and his upper lip. Which of the
A. Decreased K
A mother brings her 6-year-old daughter in for an R increased CO,, The so.called unicersal system consists of: following nerve.,; was prohahly damaged during
examinatlon because she noticed brown macules C. Nitric oxlde - A. Two-digit sets of numbers for each tooth in the accident?
on her daughter's leg, The macules have jagged D. Decreased 0,, each arch quadrant (e.g., IS to 11). A Lingual.
E. Histamine release B. Slngle sequential number for teeth repeated in B. Maxillary.
edges but do not appear raised. The mother is
42. The inferior aspect of the diaphragm is supplied each quadrant (e.g.. 8 to 1). C. Long buccaL
worried that her daughter may have a malignancy.
with blood by which of the following arteries? C. A sequential ~lphabet letter for each tooth in an D. Superlor alveolar.
After further evaluation and tests, the macules are entire dentitlon (e.g., A to T). E. Inferior alveolar.
A. Median sacral artery.
identified as cafe au lait spots. B. Lumbar arteries. D. Dlfierent symbols for each numbered tooth in 55. You decide to restore the missing cusp on the
35. Cafe au lait spots are seen in conjunction with each quadrant (e.g,, Q '. to 1.i ), patient's molar. During the administration of
C. lnferior phrenic arteries.
polyostotlc fibrous dysplasia and endocrine the inferior alveolar nerve block, which of the
D. Celiac trunk.
abnormalities in which of the following following ligaments is most likely damaged?
E, Superior rnesenterk artery.
disorders? 43. The coenzyme essential for normal amino acid A. Sph~nornandibnlar.
A. McCune-Albright's syndrome. metabolism is _ _. Test items 50-55 refer to the B. Stylom,c,ndibult1r.
B. Stevens~Johnson syndrome. following testlet. C. Tc•mporomandibular.
A. Biotin
C. Marfan·s syndrome. D. lnterck~ntal
B. Tetrahydrofolate
D, Gorlin~Goltz syndrome. A 24-year-old man presents to your office for an
C. Pyrldoxal phosphate
E. Peutz-Jeghers syndrome, emergency visit, after being hit on the left side of 56. Polycystic kidney disease is most commonly
D. Nlacin
36. The patient's radiographs could be described his face with a soccer ball. He complains that his associated with
44. Equal contracture of the lateral pterygoid
as having what type of characteristic muscle bilaterally produces which of the "tooth got knocked out" and that his jaw feels "out A. Renal cell cardnoma
appearance? following mandibular movements? of place." He has no other medical conditions. R Peripheral nenropathy
A Cotton wooL 50. During introora/ examination, you find that C. llrolithtasis
A Retrusive.
B. Ground glass, the patient;s lower second premolar is missing. D. Berry aneurysm
B. Elevation.
C. Cobweb. C. Protrusive, Which type of alceolodental fibers was least E. Non-Hodgtin's lympboina
D. Soap bubble. intio/ved in resisting the force that pulled this 57. Which of the following types of epithelium lines
D, Lateral.
E. Starry sky. 45. Which of the following cells forms the myelin patient's tooth out of its socket? the oropharynx?
37♦ A bone biopsy was taken from the patient. sheath around myelinated nerves in the central A. Apical. A. Simple squamous.
Which of the following would most likely be B. Oblique, B. Strnt!Hed 5quamom;,
nen-ous system?
observed under the microscope? C. Alveolar crest. C. Simple cuboidul.
A. Schwann cells.
A. A dense inflammatory infHtrate, D, lnterradkular. D. Simple columnar.
B, Astrocytes.
R Fibrous tissue, 51. You also notice that a cusp of his mandibular E. Pseudostratified columnar,
C, Microglia.
C, Pleomorphic cells. second molar has fractured off and that dentin 58. A scientist has discovered a new pevtide
D. Oligodendrocytes.
D. Metastatic calcifications. is exposed. Jf this patient were to drink hormone. He thinks it acts through the second
E. Amphicytes.
E. Giant cells, 46. Which of the following mineralized tissues have something cold, what will he sense? messenger system, which utilizes cA1lfP. Jf this
the greatest percentage of inorganic material? A. Pain. is true, which of the follor.l)ing .i,ubstances
38. The overall length of the primary teeth that are 13. Pressure. should deer-ease the res1wnse of this new
A. Enamel.
given here are the average range of dimensions C. Vibration. peptide hormone in cells?
B. Dentin.
with one e.'l:ception. Which one is the D. Temperature_ A. Aden:y!ate cyclase.
C. Bone.
exception? D, Calculus. 52. You decide to take a radfograph of the frac- B. Monoaminc oxidase inhibitors.
A Maxillary central incisor, 16 to 17 mm. 47. Which of the following is not a feature of tut'ed tooth. On the first film you miss the apex C, Phosphodiestcrnse.
B. Mandibular central incisor, 16 to l7 mm. of the tooth, so you decide to take another 0, Aspirin.
poststreptococeal glomerulonephritis?
C. Maxillary lateral incisor. 16 to 17 mm, radiograph. Relaxation of which of the 59. From the ocrlusal perspective1 which tooth in
A. Hematuria.
D. Mandibular lateral lncisor, 15 to l 7 mm. patient's muscles would help yon in taking the the vrimary dentition varies the most in form
B. Hypertension.
39. Fibrotic and thickened heart valves that result C. Edema, second film? from that of any tooth in the permanent
in a reduction of blood now through the valve A, Cieniohyoid. denlition?
D. Polyurla.
characterize which of the following? 48. In a cusp-embrasure relationship, the B. Stylohyoid. A. !VlaxiHary first primary rnoli1r.
A Stenosis. maxillary first premolar is most likely to C. Mylohyoid B. Maxil!nry second primary molar.
B. Regurgltaiion, articulate with which of the following D. Levator veli palatine, C. Mandlbular first primary rnoiar.
C. lnsufficiency, E. Palatopharyngeus, D. Mandibular priniary second molar.
mandibular teeth?
D. Prolapse, 53. On further examination, you determine that the 60. The most common cause of ost<>omyelitis is
A First premolar only.
E. Ischemia. B. First molar only. articular disc of the patient's
40. From the occlusal aspect the primary maxillary C. Canine and first premolar. temporomandibu/ar joint has been displaced. A Streptococtus pyogerws
second molar has which of the following D. First and second premolars. If the fJatient contracts his lateral ptery,goid B. Staphy/ococcu!'; aureu.,
characteristics? muscle, the disc will move C. Lartobacilfus casei
A... Somewhat rhomboidal in form. A Posteriorly and medially D Pseudorrw:ws aeruginnsn
B. Three wen devt>loped <;usps. R Anteriorly and medially £, Escherichia colt
C Two supplemental cusps, including tubercle of C. Post~riorly and lateralJy
Carabe.l!L D..Anteriorly and laterally
D. Poorly defined rnesial triangular fossa.
324 ., Sample Exam Sample Exam .,. 325

61. 1'/Je muscle that is found in the walls of the 70. The superior and inferior ophthalmic veins 76. The median pharyngeal raphe serves as the 83. Hassall's corpuscles are found in tht' medulla
heart is characU:rized by _ _ . dl'ain into the _ _ . attachment site for which of the following of which of the following glands?
A. A peripherally placed nucleus A. Internal jugular vein muscles? A Thymu~,
B. Multiple nuclei B. P:erygoid plexus A. Lateral pterygoid muscle B. Thyroid.
C. Intercalated discs C. Frontal vein B. Palatopharyngeus muscle C. P.:m.tthyn)1d.
D. Fibers with spindle-shaped cells D. lnfraorbital vein C. Levalor veli palatine muscle D. PineaL
62. The smallest cusp on the crown of a 5 lobed E. Facial vein D. Salpingopharyngeus E. Suprarenal.
mandibular second premolar is lite: E. Superior constrictor muscle 84. The masseter originates from the--··
A. Buccal cusp. 77. A most favorable sequence of eruption for the A. Co,idyle of the m.:1ndibk 0

B. [)Jstnbuccai cusp. Test items 71-74 refer to the following permanent dentition is which of the following H. infratempornJ crest of the sphenoid bone.
C. Distal cusp. test/et. (right side)? (Eruption sequence given by C. Inferior bonier of the 2ygom.1tic Hrch.
D. DistolingL1al cusp numbers in parentheses.) D, Pyramidal process of the palatine hone.
A 6--year-old boy presents with a history of severe A. (I) 3, 30; (2) 8, 25; (;l) 7, 26; (4) 27, 5; (5) 28, 4;
63, Which of th~ following is n<>t irwolt:ed in the E:. Mastuid proct:ss uf ternrora! bone
epistaxis. For the past 3 years the patient has (fi) 29, 6; (7) 31. 2. 85. Considering the period of 24-monlhs to about
proce.~s of mineralization?
A, Matrix vesicles.
experienced these nose bleeds, often without any B. (1) 3, 30; (2) 8, 25: (3) 26. 7; (4) 27. 6; (5) 28, 5: 6 years of age which of the folfowing is true?
lt Arnelogen!n:-. apparent cause. The patient ls otherwise in good (6) 29, 4; (7) 31, 2. A. Not all the primary teeth hove att.-ilned their
C. Fluoride. health. but his mother has noticed that he C. (l) 30, 3; (2) 25, 8: (3) 26, 7; ( 4) 27, 5: (5) 28, 4: ocdusal leveL
D- Phosphoryns. "bruises easilv." Laboratory tests are ordered. (6) 29, 6; (TJ :n, 2. B. Parts of both jaw!. containing prlmary teeth
64. T-cell lymphoma is most likely to occur in a 71. The labar:iton• test results show a normal PT D. (l) 30, 3: (2) 25, 8; (3) 26, 7; (4) 27, 6; (5) 28, 5; cha1~ge notie:eah!y in r;i2e,
patient with which of the follou:ing conditions? but a prolong;d PIT. A prolonged PIT test (6) 29, 4; (7J 2, 31. C. A !lignificant lm:reas.c ln lnten'an\n('. width
A. Chronic granulomatous disease. suggests t/Jot the patient has an abnormality 78. Which of the following metabolic activities is occurs shortly before and during the time the
B. Myasthenia gr<'lvis. affecting which component of the coagulation increased 1 hour after a meal (during the primary teeth are lost and tile permanent teeth
C. Osteochondromn. cascade? absorptive state)? emerge.
D Wilson's disease. A Activation of platelets. A. Glycogeno!ysls. D. Dental arch form i! more or less constant and
E. Celiac sprue. B. Activation of thrombopiastln. B. Oxidation of free fatty acids. pr<ictically no dimensional changes take place
65. For which of the following substances would C. Activation of plas1ninogen. C. Glucagon release. in depth or width after 9 montl!s of age.
you expect the renal clearance to be the lowest, D. Intrinsic pathway. D. Glyc-nlysis. 86. The primm-y factor determining the percent of
under normal conditions? E. Extrinsic pathway. 79. The most prominent mechanism of spread of hemoglobin saturation is:
A. Urea, 72. The diagnosis of hemophilia A is made. This the hepatitis A t>irus is by which of the A. Blood PO.,.
B, Creatinlne. disease is caused by a deficiency of _ _• following route,f? B Blood PC().,.
C. Sodium. A. Factor V. A Ora\..anal. C. Diphnsphn'gJycerote concentrntion.
D. Water. B. Factor VIL B. Respiratory. D. The temp.,.rature of th<::! blond.
E. Clucose. C. Factor VIII. C. Sexual contact. E. The acidity of the blood.
66. Di George's syndrome is characterized by a D. Factor IX. D. Perinatal. 87. What is tlte tlfterage height of cur,.iatnre of the
deficiency of _ _. E. Factor X. E, fnsect vectors. cervical line (CE.I) on the mesial and distal of
,.._. B lymphocytes 73, Which of the following describes the he,·editary 80. If the molar percentage of guanine in a human the permanent maxillary and mantlihulat·
B. T lymphocytes transmission of this disease? DNA is 30%~ what is the molar percentage of incisors? ·
C. Both B and T lymphocytes A. Auto-smnal dominanL adenine in that molecu{e? A. About 3.5 mm on the rnesla! of th<" 1:iaxillary
D. Antibodif>s B Autosomal recessive. A. !OW.. centred inds.or.
E. Complement inhlhltor C. X-linked. B. 20'.V.. B. About 1.5 mm on the distal of the max!Hary
67. Which of the folloudng phrases best describes D. lt is not genetically transmitted. C. 30¼,. central incisor.
restriction enzymes? 74. The clinical presentation of hemophilia 8 is D. 40~,. C. About 2.0 mm on the mesial of the m<1ndlhnlat
n. Site"specHlc endonudeases. indistinguishable from hemophilia A. ff'1llc/1 of E. 50%. central incis0r.
B Enzymes. that regulate 1-{;"iA. the following best describes the fq/mralory 81. The muscularis externa has a third layer in the D. About 1.0 mm on th(~ distal of thr- mandibular
C. Nonspecific e-ndonudea.ses method needed to distinguish these two central incisor.
D. Topoisomernses. conditio,u;? A Esoph;,gus 88. Which of the folloudng de,sr:ribes cells that are
68. The gallbladder arises from the _ _• A Bleeding time. B. Stomach abnormal in appearance and may become
A. Common hepatic duct 8. Assay of coagulation factor levels, C. Liver premalignant?
B. Common bile duct C. A5say of von Willebrand's factor. D. Small intestine A. Aplasi;L
C. Left hepatic duct D. Blood smear. E. Large intestine B. Dysp!asia.
D. Cystic duct E. Platelet count. 82. .4n infection by which of the following bacteria C. Karyomcgaly,
K Bile canalicu!i may result in the formation of gummas? D. Pleomorphi~m
69. Uthich one of the following disorders is the 7.5. Hydrolysis of which of the folloti:ing A. /1.{ycobacterinm tuberculosis E. MC'taplnsia
least likely to be a differential diagnostic factor compounds yields urea? B. Neisseria grinorr/weac. 89. Which jaw activity does not involve one of the
in the patient's limited jaw opening? A. Ornithine, C. Treponr:ma pol/idum. following musdes?
A. Exacerbation of TMJ and TMD. B. Arglninosuccinate. D. Bordeteita pertussis, A. C!end1ing, superior heads of lateral pterygnid
B. Trismus secomtary to TMJ pain. C. Aspartate. E. Streptococcus pyogenes. rnu:::;cles (LPM).
C. ~tyalgia secondary to TMD. D. Citrulline. B. Clenehing. inferlor he,1ds of LPVL
D. Mymliti5 secondary to bruxing. C. lp'.':iilaternl jaw movements, inferior heacls uf LPM.
D. Simpie jaw opening. cleE.•p massc-ter muscle.
326 "" Sample exam Sample Lxam "' 3 2 7

90. Hemorrhagic infarction and tissue necrosis 97. The effects of which one of thl:' foltou.Jing 102. The patient is referred to an infectious 109. In comparing permanent and primary teeth,
s1,_ggۥt which of the following? hormones are not mediated through cAAfP? disease speciaUst and placed on ' 1triple which of the follorl'ing differences are noted?
A. Asperglllosis. A. E:!>trogen. therapy. '1 Two years later, the patient is A. Crowns of anterior primary teeth are narrower
B. Blastomycosis. B. Gh..:cagon. admitte,l to the emergency room with a dry mcsiorlistally (;n comparison to their crown
C. 1-llstoplc,smnsls. C. Epinephrlne. cough and ,,;hortness of breath. His length) than the permanent teeth.
D, Mucormycnsis. D. Norepinephrinc· temperature is 101° F. The most likely cause of B. Comparntively, the roots of primary anterlm
£, Toxoplas1nosis. the patient's pneumonia is _ _ . teeth are narrower and longer.
91, lVhich of the following ribs cannot be A . ..,)'taphylococcus aureus C. Cervical ridges of enamel of the primary ante-
palpated? 8, Hciemophilvs influen::ae rior teeth are less prominent.
Test items 98-102 refer to the following C. Prreumocys:i.1 jiroveci (carinif) D. Buccal and lingual surfaces of primary molars
A. First
8. Second test/et. D. K{Pbsiefla pneumoniac .are iess flat above the cervical curvature than
C. Third E. Streptococcus pneumoniae those of permanent molars
A 43-year-old man presents for an emergency den-
D. A and B 110. Cystic f'ibrosis is a hereditary disorder that
tal appointment complaining of a burning sensa- 103, The primary maxillary first molar has which
92. Which of the following is relea!;ed by mast cells results from defective
tion in his mouth. Upon examination, white of the following characteristics? A. Collagen
after antigen binding?
plaques are observed along the oral mucosa. The A It is larger in all dimensions than the primary B. Lyios.ornal enzymes
A. lgE.
B. Ly.ozyme. patient otherwise appears healthy. There is no his- maxillary second molar. C Chloride channels
C. llA. tory of systemic illness, but the patient did state B. AU three roots can be seen from mesial per• D. Flbri!!in
D. Le-ukotriene. that he had a blood transfusion more than IO years spective. E. Myehn
E. Interferon. ago following a car accident. The doctor referred C. Bifurcation of roots begins almost immediately 111, Which of the following strata of oral
93. lJthere are the temperature control centers the patient to emergency room for further tests. at the slte of the cervical line (CE.I), epithelium is engaged in mitosis?
located? •98. U1,on further evaluation, the doctor requests D. The mesial mot is com1iderab!y shorter than A. Basale .
A. Cerebellum. cw HIV and hepatitis test~ The laboratory the distal one. B, Granulosum.
B. Hypothalamus. performed both an El/SA test and Western 104. J.flhich of the following is the end product of C. Comeum.
C. Medulla. blot1 re()ealing that the patient is HIV-positive. purine degradation in humans? D, Spioosnm,
D. Cerebral cortex, The Western blot is used to identify which of A. Urea. 112. The class of antilmdies that plays an
94. Which of the following is a major complication the following? B. Uric acid. important role in mru:osal immunity is _ _ .
of chronic bronchitis? A Antibodies. C. Adenosine, A. lgA
A. Myxedema. B. DNA. D. Xanthlne, B. lgD
B. Pneumothornx. C. RNA. 105. Which of the following is an essential amino C. lgE
C. Emphysema. D. Proteins, acid? D. lgG
0, Pernicious anemia. £, Plaque-formlng units. A. Tyrosine. E. lgM
E. Malignant transformation. 99. Given the patienrs history, if the patient was B. Tryptophan. 113. lVhich of the following cells are defective in
95. A comparison occlusatly between the primary later diagnosed with actiue hepatiti.vJ which of C. Proline. chn1nic grarwlomatous disease?
mandibular second molar and the permanent the following would most llkeiy be the 0. Serine, A, Neutrophils,
mandibular first molar shows which of the causative agent? E, Alanine. B. Lymph<,cytes.
following differences? A. Hepatitis A. 106. Which of the following are the most abundant C. Plasma ce!is..
A. The meslo-, tlistobuC"Cai, and distal cusps of the B. Hepatitis B. in the fovea cenlralis of the eyeball? D. Killer T cells.
primary molar are almost equal in size: the C. Hepatitis C. A. Rod ceils, E. Macrorhagcs.
distal cusp of the permanent molar ls smaller D. Hepatitis D. B. Cone cells, 114. The coenzyme that serves as an interme<liate
lhan the other two cusps. E. Hepatitis E. C. Rod and cone cells, carrier of one-carbon units in the synthesis of
B. The primary moiar crown ls wider 100. Which of the Following would the doctor likely D, Amacrine cells. nucleic acids is which of the following?
buccolingually (in comparison with Its prescribe for the patient's inlraoral irrfectfon? 107. The middle trunk of the brachia! plexus of A. Ascorbic add,
mesiodistal measurement) than is the A. Amoxicillin. nerves arises from: B. Tetrahyctrofolic acid.
permanent molar. B. Vancomycin. A. C5 C. BioOn.
C. The primary molar outline is somewhat C. CiproOoxacln. B. C6 D. Pyridoxine.
hexagonal; lhe permanent nwlar is D. Nys.tatin C. C7 115. Which of the following genetic diseases that
rhomboidal. E. Chlorhexi<lin'"-. D. CS results from a deficiency in the liver enzyme
D. The ratio of the crown/root length of both 101. All of the following molecules may be found 108. Tineu pedis. which b~ commonlv known as that cmwerts phenylalanine ta tyrm;;ne?
molars ls the same, within the nucleocapsid of an HIV virus athlete's foot, is a fungal infeciion that is A. Albinism.
.96. Calcium binds to which of the following for except one. Which one is the exception? caused by the following dermatophyte(s): B. Homocys.tinurl<L
contraction in smooth muscle? A. Reverse transcriptase A Microsporum C. Porphyria.
A. Troponin C. B. lntegrase. B. Tm:hnphyton D. Phenylketonuriu.
8. Calmodulin. C, Neuramlnidnse. C Epidermophyton
C. Myosin. D. Protease. D. Both A and B
D. Actin. E. Ribonucleic add. E. Both B and C
E. Desmos(}mes.
328 v Sample Exam Sample Exam • 329

Test items 116-127 relate to the following 120. Migraine is a form of headache that is 126. Extraction of tooth 32 reveafed attached soft 133, Which of the following factors would result in
case presentation. currently thought to be best understood on the tissue. Which of the following is most decreased glomerular filtration rate?
basis of all the foflowing with one exception. important for a presumptive diagnosis of A. A fall \n plnsmn protein concentration.
A 35-year-old woman presents with a painful lim- The excl!ption is which of the fallowing? tuberculosis? B. An obstruction of the tt;bular system whlch
itation of jaw opening (28 mm), a painful tooth A. As a dysfunction of brainstem pathways or A. Caseous necrotic areas. wnulrl increase capsular hydrostatic pressure.
on the right side, and swelling at the angle ol the dier:.cephaHc nucleL B. Acid~fast bacillL C Vasodilatmn of the afferent arterioles.
jaw. She has a history of temporomandibular dis- R As a primary di!order of the brain. C. Epithellold histlocytes, D lnulln admini:',tration.
ease (TMD) and conservative treatment. Medical C As ~imii.ar in mechanism to tension D. Langerhans giant cells. 134. The most common form of breast cancer is
history reveals that the patient is being treated headaches. 127. The patient Is on an anticoagulant drug (e.g.,
D. As a neurovascnlar headaC'he. u:arfarin [Coumadinj) as well as rifampin. A. Adenocar<'inoma
for tuberculosis with combination antituberculo-
121. Three key factors in the pathogenesis of pain lv'hat is the effect of rifampin on the B. Teratoma
sis drugs including rifampin (Rifadin). She is also
in migraine are usually com•idered. Which of anticoagulation effect of warfarin (Coumadirt)? C. follk:ulnr lymphoma
being treated with an anticoagulant, warfarin the follou.Jing is not considered to be a key A. Increases tte anticoagulant effect of warfarin. D. Sarcoma
(Coumadin), a low level of aspirin, and paroxe- factor? B. Increases the cyclic converl'\ion of vitamin K E. Carcinoma
tine (Paxil) for depression. The intraoral exami- A. Cranial blood vessels. epoxlde reductase. 135. Which of the following are NOT type traits of
nation shows extensive teeth wear from B. ~-amyloid-containing plaques in the brain. C. Anticoagulation effect is inhibited. the permanent maxillary central and lateral
bruxism, diagnosed by a sleep specialist as sleep C. Trigeminal lnnervation of the vessels. D. Decreases its metabolic clearance by inducing ittdson'i?
bruxism. Tooth 32 has a deep carious lesion and, D, Reflex connection of trigeminal system with activity of hepatic oxidases. Central Lateral
on radiographic examination, a periapical radi- cranlal parasympathetic outflow. Incisor Incisor
olucency. 122. The treatment of the patient with low levels of 128. Gut-associated lymphoid tissue (GAL1J A. Labial view: lncisal 3rd Jnct. incisali
116. The incidence of tuberculosis is increasing a,,; aspirin is done for which of the following produces secretory _ _. Mesia! C'oniact mid 3rd
a result of an association with AIDS. Oral reasons? A. lgA B. Labial view: JncL inclsal/ :V1iddle third
infections of tuberculm;is (TBC) do occur- but A. To reduce the llkelihood of platelet aggrega- B. lgD Distal Contact mid 3rd
are uncommon. Diagnosis of oral lesions may tion, C. lgE C. Mesla! view: Within incisa! JncL incisal/
present several challenges, as set forth in all B. To stlmulate cydo-oxygenase in the platelets. D. lgG contacts thir<l mid 3rd
of the following statements except one, Which C. To increase the formation of thromhoxane. E. lgM D. Labial: Slightly Sharp rt. angle
one of the following statements is false? D. To cause platelets to regenerate cydom.-ygc- 129. Antinuclear antibodies are seen in the serum mesloincisal rounded
A Lesions secondary to HIV may be present. nase. samples from patients with _ _. angle
B. Isolation of M tu~rculosis by culture requlres 123. When there is a pulpal"periodontal infection A. Hypogammaglobulinemia 136. 17te apex of a medullary pyramid in tlle
4 to 6 weeks or longer. of a mandibular third molar, which of the B. Chronic granulomatous dlsease kidney is called the _ _.
C. Mycobacteria can be demonstrated by special following listed facial and cervical spaces is C, Sy,temic lupus erythematosus A Cortex
5t.ain.s ln only 27% to 60% of cases. most likelv to have become infected when D. Multiple myeloma B. Medulla
D. Molecular tests ( e.g,, polymerase chain reac- there is sl~;elling at the angle of the jaw? E. Pheochromocytoma C. Renal papH!a
tion) show slow turnaround times, A. Retromolar space. 130. What is the correct general structure of the D. Major calyx
117. All the following side effects have been B, Submaxillary space. backbone of DNA and RNA? E. Minor calyx
reported to be related to the use ofrifampin C. Submasseterlc space. A Sugar-hasf':-6ugar. 137. All of the following conditions are commonly
except one. Which is the exception? D. Parotid space. B. Bases Jinked through phosphodiester linkages. associated with a group A, /3,hemolytfr
A Green bodily fluids-sweat, tears, urine, 124. Lymphatic drainage from tooth 32 will C. Bases linked through hydrogen bonds. streptococci infcctfon except one. Whic/1 one is
R Hepatitis. first involoe which of the following node D, Sugars linked through phosphodiester link- the exception?
C. Leucopenia, group,f? ages. A. Scarlet fever.
D. Nephrotoxlcity. A. Lateral upper deep cervical node. 131. All of the following are found in the posterior B. Toxic shuck syndrome.
118. Which of the following modes of action does B. Medial upper deep cervical node. triangle of the neck except one. Which one is C. Pharyngitis.
not relate to rifampin? C. Lateral lower dieep cervical node, the exception? D. Endocard!Hs.
A. Inhibits RNA synthesis. D. Submaxillary node. A. External jugular vein. £. lmpE:-tigo.
B. Binds tightly to eukaryotic RNA polymerase. 125. Alt but one the following are consideratiom, B. Subclavian vein. 138. The auriculotemporal nerve encircles which
C. Tuberculocidll.l to intracellular and extracellu~ relevant to the diagnosis and treatment of C. Hypoglossnt nerve, of the following vessels?
lar organlsms. tuberculosis. Which of the following D. Phrenic nerve. A. Maxi!k1ry artery,
D. Reduces activity of hepatic mlxed-functlon oxiM statements is not true? E. Brachia{ plexus. B. Superficial temporal artery.
dases. A. Increase in the prev;:1lence of TB. 132. H"hich of the following amino acids is C. Deep auricular artery.
119. Exacerbation of bruxism has been reported to B. Oral TB lesions occur most frequc-ntly on the positioned at every third residue in the D. Mirldle mening<.:al artery,
occur with all the following agents except gingival. primary structure of the helical portion of the E. Anterior tympanic artery.
one? Which is the exception? C, Emergence of multldrug-resi3tant strains, collagen-a-chains?
A. Paroxetine (Paxll). D. High risk of A{ turxm:ufo.<Jis infection in patient~ A. Glycine.
B, Selective serotonin reuptake inhibitors. infected with human immunodeficiency B, Glutamate,
C, Naproxen (Naprosyn). virus (HIV). C. Proline.
I} Amphetamine derivativie ("Ecstasy"), D. Ly5ine.
K Hydroxyproline.
330 T Sample Exam
Sample Exam • 331

Test items 139-143 refer to the following 144. The conwrsion of information from DNA into 152. The pancreas is enveloped at its head by the 159, 17re cricopharyngeus muscle of the <'sophagus
test/et. mRNA is called which of the following?
A Translation. A. first part of the duodenum A, Is a para.sympathetic stimulator of pe-ristalsi!:
A 55-year-old man presents with malaise and dys- R Transcription. B. Second part of the duodenum B. Is a ~ymp.:1thetic inhibitor or peristaltib
pnea. He has a low-grade fever and reports that C, Transduction, C. TI1ird !)art of the duodenum C. Prevents S-\•.:allowing i,ir at th-e pharyngeal
his shortness of breath has increased steadily D, Transformation. D. Fourth part of the duodenum end
over the past week and a half. He has a history of 145. Maximum rotation and translation of both E. first part of the jejunum D. Prevents regurgitation oJ stomach contents at
rheumatic fever and denles ever using recre- condyles takes place at: 153. Afucopolysaccharidoses are hereditary• the ahrlomina! end
ational drugs. He is currently being treated by a A. Maximum opening. disorders that are characterized by the E. Controls th~ gag reflex
dentist for lull mouth reconstruction. B. Maximum protrusive. accumulation of glycosaminoglycans in 160. The most common cause of bacterial
139. Upon further examination1 a heart murmur C. Right and left lateral excursive movements. various tissues due to which of the following? meningitis in neu:borns is _ _.
was detected. Given the patient's past medical D. Hinge movement A. Overproduction (.e:ynth~is) of proteoglycans, A. Stt1phylococc11s aureus
history, which heart valve is most likely 146. The right subclavian artery arises from the B. Deficiency of one of the lysosomal, hydroiytic B, Streptncoccus prre11mnnioe
affected? _ _ and the left subclavian artery a.rises enzymes normally involved in the C Escherichia coli
from the _ _. degradation of one or more of the
A. Mitra! valve. D. Haemophilus influenzae
B. Tricuspid valve. A Axillary artery; aortic arch glycosaminoglycan.s, £. Listeria monorytogenes
C. Aortic valve. B. Brachiocephalic artery; aortic arch C. The synthesis of abnormal proteoglyctins. 161. ln which one of the following tissues is
D. Pulmonary valve, C. Aortic arch: brachtocephaHc artery D, The synthesis of highly branched glucose transport into the cell urwffocted by
140. Before the patienrs development of rheumatic D. Brnchiocephallc artery: axillary artery glycos.1minoglycan chains. insulin?
fever, he likely suffered from which of the E. Axillary artery; brachial artery E. Arginin~. A. Skeletal muscle.
following conditions? 147. Rest position (RP) Is defined: 154. A hormone acts to stimulate its neighboring B. Liver
A Cystitis. A ..4.s any position of the mandible that lacks cell to divide. This hormone would best be C. Adipose tlssue.
B. Pharyngitis. contact of the teeth. described as belonging to which category of D, Smooth musde.
C. Food poisoning. B. As the centric relation position of the hormones? 162. Deoxygenated blood from the tram,·ot~rse sinus
D. Thrombocytopenia. condy!es with the teeth apart. A Paracrine. drains into the
E. Menlnigitis. C. As a mandibular position with masticatory B. Autocrine. A. Inferior sagittal sinus
141. After further evaluation and tests, the patient muscles at complete rest. C. Endocrine. B. Confluence of sinuses
is diagnosed with subacute endocarditis. If D. As a clinical mandibular positJon in relation to 155. The most common cause of pyelonephritis is C. Sigmoid sinus
the infecting microbe was cultured in the the interocd~sal space. D. Straight sinus
laboratory, the re,,ults would most likely shmv 148. Which of the following ions has a higher A. Staphylococcus aureus K Internal jugular vein
that this microbe is positive for _ _. intracellular c.oncentratiott compared to the B. Vibn~ cholerae 163. The primary maxillary second molar has
A. a-hemolysis extracellular fluid? C. Escherichia coli what characteristics?
B. !3-hemolysis A. Na·. D. Helicobacter pylon.' A. Does not have a well~deflned mes!al tri.1ngular
C. )'-hemolysis B. K•. E, Bordetetla pertussis fossa.
D. Coagulase C. CI·. 156. Which of the following types of epithelium B. Oblique ridge absent or not well developed.
E. Lecithinase D. Hco,- acimar units of tines salivary glands? C. Development (central) groove is well defined.
142. Which of the following is the most likely E. Ca2•. A. Simple squamous. D. A tuhercle of C'arnbelli {Supplementory cusp)
complication that may occur from the 149. All of the following cells are associated with B. Stratified squamou!I. is well develope(!.
vegetations forming on the patient's defectire chronic inflammation except one. Which one C. Simple cuboidal. 164. Facial nerves are derived from the
heart valve? is the exception? D. Simple columnar. branchial arch.
A.Myocardial infarction, A. Macrophages. E. Pseudostratified columnar. A. First
B.Hemorrhage. B. Neutrophils. 1§7. The _ _ is a component of the B. Second
C.Petechiae. C. T lymphoc,1es. juxtaglomerular apparatus which functions C. Third
D.Cor pulmonale. D. B lymphocytes. in regulation of blood pressure. D. Fourth
E. Embolus. E. Plasma cells. A. Proximal convoluted tubule E. Fifth and sixth
143. After the diagnosis is made, the patient is 150. When is the crown of the permanent B. Distal convoluted tubule 165. Analysis of DNA fragments (probing) is
immediately placed on high-dose, IV mandibular second molar completed? C. Bowman's capsule possibly due to which of the following
antibiotics~ One of the antibiotics that is A. About 7-8 years. D. Glomeruius properties of DNA?
admint..~tered to the patient is streptomycin, B. About &-9 years. E. Macula densa A. Pbosphodiester bonds.
an aminoglycoside. The antimicrobial effect of C. About !,-!O years. 158. Tire masseter muscle, which has a complex of B. Compllmentary strands.
streptomycin is to inhibit the synthesis of D. About 10-11 years. internal components, includes all the C. Protein binding,
151. Which of the following proteoglycans is following EXCEPT? D. Western blotting,
A. The bacterial cell wall present in extracellular space? A. Permation. 166. From the occlusal aspect, the primary
B. Folale A. Hyaluronic add. B. Structural composition permitting regional mandibular second molar has which of the
C. Protelns B. Keratan sulfate. activation. following characteristics?
D. Nucleic acids C Chondroitin sulfate. C. Multiple internaf aponeuroses. A Somewhat rectangular in form.
E. jl-Iactamase D. Dermatan sulfate. D. Internal aponeuroses that do not move or B. The outline of the crown crnn'erge, meslally
E. Heparin. deform. C. Three buccal cusps are dissimilar in size.
D. Cusps do not have well define(] Lriangular
ridges.
332 ,,, Sample Exam Sample Exam Y 333

167. Whiclt of the follnwing are not type traits of' 174. Which one of the following is considered a 179. Which of the following is the most 185. From the incisal deu:, a greater mesiodistal
permanent maxillary molars? primaty ligament of the 17'rfJ? appropriate drug used in combinotion measurement than fadolinguaf measureuwnl
A. Sty!omandibular. therapy for tuberculosis to treat the patient? can be seen in which of the foffowinM
First Second 17Jfrd
8 Sphenomandihular. A. Amoxidllin. permanent anterior crowns?
Molar Molar Molar
C. Stylohyoid B. Clinclamycin. A. \-1,;xlllary central ir.dsor.
A. Fluccal Widest Inter~ Smallest
D_ T~rnpornmandH:rnlilr. C Ceph&losporin. B. l\foxillary canine,
vlev:: molar mediate molar
D. Tetracycline. C :\fondlhular canine.
\Vidth
E. Rifampin. D Mandibular central lndsoc
B. DL cusp: Same ,ize Same size Sma!lP-s1
Test items I 75-180 refer to the following 180. After 3 weeks, the patient was feeling ''much 186. The ,,estigial cleft of Rathke's pouch in the
as MJ as M, size
More Triangle
test/et. better" and was discharged from the hospital, hypopf1ysis is located between the
C. Occlusal Squar~/
although he remained on his drug therapy for A. Anterior an<! po:aerior lobes
view: rhom- rhom- or heart- A 60-year-old homeless man who lives in a com- another 6 months. Which of tlu~ following best R Anterior lobe ctnd hypothal:1mu:-
Doiel boidal shaped munity shelter presents with history of coughing describes the calcified scar that later formed C. Poste!'lor lobe and hypotlrnlamus
D. MB root ln line ln line wiih Roots for the past 6 months. He has a slight fever. in the affected lung parenchyma and hilar D. !'v1edian eminence 11.nd the optk s.'hiaslll
apex: wlth crown dis- hemoptvsis. and productive cough with a yellow- lymph node? 187, A comparison of the pulp chambers and ro<Jt
cusp tlp center placed
ish sputum discharge. After further examination A. Gumma. canals of maxillary primary and permanent
168. A cotton wool appearance may be used to
and lests, the patient is diagnosed with active B. Chancre. ,,r;econd molars shows which of the follmdng?
describe the radiograph of a patient with - ~ ·
tuberculosis. C. Metastatic caklfkaUons. A. Enamel cap of primary tooth is relatively th!ck
A Ostcopetrosis D. Tuhercle.
175. When the sputum samples were taken to the but less cnns!stent in depth.
B. O~teitis dcformans
laboratoryt what test did the doctor order to E. Ghon complex B. ComparaHvely less thkkness of dentin at th<?
C. Peutz-Jeghers syndrome
be performed to help make the diagnosis? occlusal foss;ci of primary molars.
D, Seborrheic keratosi.s
A. Gram staln. 181. An infection in a mandibular incisor with an C. Pu!:, chambf'rs are proportionally larger in pri ➔
E. Osteogenesis imperfecta
B. AcicHast stain, apex below the mylohyoid muscle drains into rnarv molars
169. The energy for skelet<il muscle contraction is
C. Spore stain. which of the following spaces? D. Puli; horn" are lower in primary molar5, espe-
,lerfoed from which of the following processes?
D PPD test (tuberculin test) A. Sublingual space ciaHy distr1I horns.
A. Calcium release from sarcoplasm!c mem-
E. Voges-Proskauer tes:L B. Submental :space 188. The center that provides outpnt to the
branes antl binding to troponin.
176. Alter 2 weeks. the bacterial cultures came C, Suhmandlbular space respiratory muscles is located in the _ _ .
B. Cleavage of ATP by the rnyosin head.
back from th~ lab confirming the initial D. Parapharyngeal space A. Pons
C Membrane &odiurn-potas~lum ATPase pump,
diagnosis, positive[)-' identifying the organis,:n 182. Phaspholipase C is an enzyme that play:; an R fVfedullo
D. Sodium influx during the action potential.
Mycobacterfum tuberculosis. 1U. tuberculm;rs important role in the production of second C. Cerebral cortex
170. Which of the following is a prope,·ty of C mf?sse.ngers, which produce intracellular
is known to infect which of the following D. Cerebellum
fibers?
cells? responses. Which two second messengers are E. Hypothalamus
A. Have the slowest conduction velocity of any
A. Fibroblasts. produced through the action of this enzyme? 189. 11ie brachia( plexus of nerves arises from
nerve fiber type.
B. Basal cells. A. cAMP and tyrosine kinase. ttrhich of the following roots of the anterior
B Have the largest diameter of any nerve fiber
C, Type l pneumocytes. 8- Acetylcholine and histidine. primary rami of spinal nen;es?
type. C. Adenylatc cycla~e and protein kinase.
D. Macrophages. A. All cervical roots \ C l-C8),
C. Are afferent nerves from muscle spindles.
E. Eryl hrocytes. D. l,2-diacylglycerol and inositol 1,4,5-triphos- B. All thoraclc roots (Tl-T12).
D. Are afferent nerves from Golgi tendon organs. phate.
J 77. Which of the folfowlng is a glycolipid found C. C 8 and Tl
E. Are preganglionk autonomic fibere:.
on the surface of M. tuberculosis that plays a 183. Which of the following cytokines ,,;timulate B I.l C5 through CS and TL
171. To which of the following bones is the tensor
role in its pathogenesis? lymphocytes to differentiate into plasma cells? £. C5 throuf{h C8 a:1d Tl through T4,
tympani attached? A. IL-1.
A. Cord factor. 190. Aschoff bodies are observed in which of the
A Incus. B. IL-2.
B. 0 antigen. following conditiom;?
B. Malleus. c. n~3.
C. Proteln A. A. Acute mye!ngenous leukemia.
C. Stapes. D. IL-4.
D. Exotoxin A. B. Phcochromocytoma.
D. Hyoid.
E. Lecithinase. £. IL-5. C. Osteopetrnsis.
E. Mandible. 184. Which of tire following ,"itatemenls regarding D, Rheumatic !ever.
172. An increase in alkaline phosf)hatase may be 178. Since the patient was living in a homeless
shelte,~ the tuberculin test was administered the parasympathetic nervous system is true? E Sclerndcrma
seen in all of the following conditions except
to all of the tilaff anti residents liuing at the A. The third cranial nerve (the oculomotor 191. Nonsterofrlal anti-inflammatory agents are
one. Which one is the exception? nerve) carries ~ymp.nthetic fibers to the
shelter. 11ris test is base<J on a delayed type poinrellering and anti-innommntory. Tirey are
A. Hyperp~rathyroidi!'lm,
frypersen.o;itieity reaction that is mediated by ~month muscles of the eye. effective since they act to inhibit
B, Osteoporosi!';.
B. The facial and the glossopharyngeal crau!al prostaglandin s;vnthn;is by:
C. Osteitis deformans. nerves carry the parnJympath~tk pregan-
A OnlylgG A. inhlbitlng fatty acid llpo-0::.ygen.:1.sc activity,
D. Adenocarcinoma of the prostate. gliontc fibers for the antonom!c innervation to
B. lgG and lgM B. lnhlhiting fatly acid-specific cyclo-oxygP1:as<e~
E. Multiple myeloma. the salivary glan<ls.
C. lg£ actlvity.
173. The hereditary transmission of Peutz.Jeghers C. The parasympathetic nervous system Inner-
D, T cells and macrophnge~ C. Inhibiting fatty aC!d~spe,lfic hydrr,pcroxld;1se
syndrome is _ _.
E. Mast cells and basophil~ vates prirnarHy striated muscle ln the body. activity.
A. Autosomal domimmt
D. The parasympathetic nervous system is D. lnhthiting pho~pho!ip,:ise A2.
B. Autosomal recessive organized for diffuse activation and responses.
C Sex~linke<l (lorninanr
D. Sex-Hnkecl recessive
E_ Not genetically transmitted
334 Y Sample Exam Sample Exam Y 335

192. Terminal bronchioles are characterized by Test items 199-202 refer to the following 205. llreters travel inferiorly just _ _ the parietal Questions 212-223 relate to the following
cells. test/et. peritoneum of the posterior body wall. They history and examination
A. Goblet pass _ _ to the common iliac arteries as
R Cll!ated cuboidal A 30-year-old woman comes to your office for a they enter the pelvis. A mother brings her l2,year-old boy to the dentist
C. Nonci!lated cuboidai dental examination. She has not been to the den- A. Above; posterior to ask about her son grinding his teeth and about
D. Ciliated squamous tist in 2 years. The patient has type l diabetes. B. Above; anterior some white ··spots·· located on the smooth-sur-
E. Noncitiated squamous which requires her to take insulin. She is other- C. Below; posterior faced enamel of several of his anterior teeth and
193. A major anatomical variant of the two-rooted wise in good health. On intraoral examination, you D. Below; anterior premolars. Among other questions about the
mandibular molar is a tooth with an notice that the dorsum of her tongue has a thick, E. Above; superior cause of the defects, the mother then asks when a
additional distolingual and third root. What is matted appearance and diagnose her with hairy 206. Aspartame contains aspartic acid and which systemic disturbance occurred that may have
the preullence of these three-rooted of the following amino acids?
tongue. You also find that the patient has deep caused the "spots". The patient has excessive
mandibular first molars? A Phenylalanine.
A. May e.xceed l O'¾, in Caucasians,
caries in her upper second maxillary molar.
B. Leucine. tooth wear from bruxing and clenching. The den-
B, Less than lW; in Eurnsian and Asian populations.
199. Which type of papillae is affected that causes C. lsoleucine. tist measures the cervical-incisal length of the per•
C. Greater than S'K, ( even up to 40':¾i) in popula- the hair-like ap11earance of her tongue? D. Lysine. manent maxillary central incisor (10.5 mm). Also
tions with Mongoli~ln traits. A. FoHate. E. Praline. measured is the distance from the CEJ to the mid-
B. Circumval\nte. 2()7. Injury to which of the fol/owil!g nerves would point of the defect (5.5 mm). Given that the crown
D. Greater than 8'.t in African populations.
C. Fungiform. affect abduction of the eyeball?
194. Which one of the following statemeats is completed over a period of 4 to 5 years it is
D. Flliform.
regarding the regulation of gastrointestinal A. Optic nerve. possible to estimate the age at which the hypopla-
function is true? 200. On the patient's radiograph, you notice that B. Oculornotor.
the pulp chamber in the carious molar sia occurred using 6 months or yearly periods in
A. The main sympathetic nerve supply to the C. Trochlear. the following formula:
appears smaller than the surroun,ling teeth.
dlgestiv-e tract i~ the vagus, D. TrigeminaL
This is most likely due to the deposition of
B. In general, sympathetic stimulation is excita- E. Abducens. ADF ACF - (yrs. of lormationicrm,m height x
which type of dentin?
tory to dlgestive activity. 208. Which of the following statements regarding distance of defect from CEJ).
A. Secondary.
C. Salivary secretion is stimulated by both tubular secretion in the kidney is true?
branches of the autonomic nervous system, B. Tertlary. A. The secretion of K· increases when a person is 212. llsing an average age of crown formation
although not to the same degree. C. Mantle. in acidosis. (ACF) of both 4 and 5 years. the age of defect
D. Sclerotic.
D. Parasympathetic stimulation of the salivary B. The secretion of H+ increases when a person is fonnation (ADF) is estimated to be about what
glands produces a saliva rlch in mucus.
201. You decide to remoue the caries and prepare in alkalosis. time?
the patient for anesthesia. Which nerve must C. It ls a process that transports substances from
195. The occlusal surface of a primary mandibular A. 7-9 months in utero.
you anesthetize to ensure adequate
first molar often has a prominent faciolingual the filtrate to the capillary blood. B. 0-1 yean of age.
anesthesia for the patient? D. lt accounts for most of the K+ in the urine.
ridge. This transverse ridge connects which C. 1-2 ye.ars.
A. Nasopalatlne nerve. 2()9. _ _ marks the end of growth in length of
two cusps? D. 2-3 years.
B. Greater palatine nerve.
A. Buccal and distolingual. long bones. 213. The increase offluorosis of permanent teeth
B. Mesiolingual and di5tobuccaL
C. Anterlor superior alveolar nerve. A. Diaphyseal closure in both nonfluoridated and optimally
D, Middle superior alveolar nerve.
C. Mesiobuccal and mesiollngual. B. Epiphyseal closure fluoridated populations points to the need for
E. Posterior superior alveolar nerve.
D. Distobuccal and dlsto!ingnaL C, Ossification dentists to caution parents with children
202. After administering the anesthetic, the patient
196. Which of the following disorders i.t; least likely D. Formation of periosteum about potential causes of fluorosis in
complains that her "heart feels like it's E. Cessation of bone remodeling
to be included in the differential diagn<>sis of children. Which of the following cautions
a patient with jaundice? racing." You explain to her that it may be 210. The presence of M-proteln antibodies suggests about fl.uoride is correct, but the age or
from the epinephrine in the anesthesia. Which
A, Hepatitls, immunity to infection by which type of Implied age is NOT Correct?
of the following glands could most likely bacteria?
B. Hemolytic anemia. A. Excess(, 1 ppm) fluoride in the drinking
C. Cholelithiasi.-;. cause the same symptoms in the patient? A. Streptococcus pyogenes. water during enamel formation,
A. Hypophyoi,.
D. Glomerulonephritis B. Streptococcus viridans. B. Excessive(> pea-sized amount) use of fluoride
B. Thyroid.
E. Carcinoma of the pancreas. C. Streptococcus sanguis. toothpaste under 6 years of age.
C. Pineal. D. Staphylococcus aureus,
197. Arteriovenous anastomoses in deeper skin are C Use ot fluoride toothpa~te only for cbHdren
important in _ _. D. Suprarcnal. E, lactobadflus casei. under 4 years of age.
A. lmmunity 211. The synthesis of all steroid hormones involves D. Use of a LI% sodium fluoride toothpaste or
203. Which of the followiag microbes is the most
B. Thennoregulation which of the following compounds? gel by pediatric patients only when 6 years oi
common cause of gastroenteritis in children? A. Pregnenolone.
C. Controlling the arrect.or pili muscle age and older.
A Reovlruses. B. Progesterone.
D. Pigmentation 214. Systemic etiologic factors that are said to be
B. Picornaviruses.
E. Pain sensation C. Aldosterone. associated with enamel defects such as
198. Complications of Barrett's esopltagus include
C. Togaviruses. D. Cortisone. hypoplasia occur generally in what period of
D. Pararnyxoviruses. £. Testosterone.
alt of the following except one. Which one is time?
204. Name the point angle which represents the A. Before birth,
tlte exception?
junction of the- cutting edge of an incisor with
A. Varices. B. GenetaHy after hlrth and before the age of 6
the surface that is toward the tongue and the
B. Stricture. years.
surface that is aa,ay from the midline.
C. Hemorrhage. C, During the first year postpartum for
A. Distopro:.cimoindsal.
D. Adenocarcinorna. Hutchinson's incisors.
B. Dlstolabwlnci:rnl.
E. Ulceration. D. During birth.
C. Distolinguolndsal.
D Labioincisnlinguo.
336 -rr Sample Exam Sample Exam T 337

215. The differential diagnosis of white "spots" of 220. Aggravation of bruxism has been suggested to 225. Hypertension (long~term) UJill be compensated 233. The first eriidence of calcification (weeks in
the enamel of primary and permanent teeth occur secondarily to all of the following by which of the following renal mechanisms? utero) in the primary dentition occurs in
should include disorders that haf.Je a occlusal relationships except one. Which one A. Increased circulating ADH (vasopressin), which of tl,e foltou·t'ng teeth at about what
sulu;tantiated cause. Hlhich of the following is the EXCEPTION? R Increased 5ympathetic activity, age?
DO NOT have an euidence-based causal A. Occlusal interferences in centric relation. C. Decreased clrculating: aldosterone. A. Maxillarv central incisor-14 (13-16) weeks,
relationship with enamel hypoplasia? B. Ocdusal interferences in the intercuspal posl- D. Increased circulating anglotensin H B. Mandib1;l1"1r central lncisor-12 (10-13) weeks;,
A. Rickets. tion. 226. The most superficial layer of the epidermis is C Maxillary lateral lndsor-14 (13-lG) weeks.
B. Congenttal syphilis. C, latrogenlc occlusa! relations that interfere with the stratum o_ Mandihuiar lateral incisor-•14 (13-1:3)
C. Measles, bruxism. A Spinosum weeks,
D. F!uorosis. D. Angle Class HI malocclusion (prognathism), 8. BaMle 234. From the lingual perspective, the crown of the
216. Clenching and grinding of teeth involves 221. The differential diagrwsi.-, of enamel C, Granulosum p,•imary maxilfory second molar shows which
contraction of skeletal type muscles. Several hypopla:,ia should take into account D. Lucidum of the following?
types of myofilaments are present in the suggested differences between non•fluoride E. Corneum A. Small, we!l-developN! m"'~iolingual cusp.
contractile elements of skeletal muscles. and fluoride opacities. Which of the following 227. Where are the cells that produce calcltonin B. Distollngual (DL) cusp smaller thnn the maxi!~
Which statement about muscle filaments IS statements does not suggest a basis for a located? lary primary first molar DL cusp.
NOT true? diagnosis of non..fluoride (lVFJ enamel A. Red marrow. C. There is no supplemental CUS[) apical to the
A. Myosin forms the thick filament of muscle. hypoplasia? B. Adrenal gland. mesiolingual cusp.
B, Actin is a major protein of thin filaments. A. Levels of Fin drinking water that range from C. Parathyroid gland. D. o~velopmental groove separating the mesl-
C. Titin is a protein of elastic fllament 0.2 to 0.34 ppm have been reported to be D, Thyroid gland. olingual and distolingual cusps.
D. Connectin is a protein of intermediate associated with prevalences of NF enamet E. Spleen, 235. An B•year-old boy presents with macroglossia
filament. opacities ranging from 22'¾, to 35%, 228. In mature dentin, the ratio of inorganic to and delayed eruption of his primary teeth. Of
217. The clinical examination of the patient B. At a level of l to 1.5 ppm of F in drinking organic matter is approximately _ _. the following choices, which one is most
reveals extensive wear of the right canines, water, only few f opacities occur. A 94:6 likely?
and somewhat less wear of lhe lateral C. Most NF enamel opacities appear as white, B. 50:50 A. Plurnmer·s disease.
incisors. Also there is tenderness of the jaw opaque spot5 in smooth surface enamel; areas C. 70:30 B. Osteochonctroses.
closing muscles, particularly on the right side. of mild dentnl fluorosis are lusterless, opaque D. 80:20 C. Cretinism,
The muscle(s) that would be involved white pa.tche~. E. 60:40 D. Wilson's disease.
primarily in providing most of the force for D. f]uorosi! and NF opacities are dinically slgnH- 229, All of the following symptoms are mediated by E. MaHory~\Veiss syndrome.
anterior tooth clenching include which of lhe icantly different. antibodies except one. Which one is the 236. A major function of surfactant is to inc1-·ease
following masticatory muscles? 222. During muscle contraction what physical exception? which of the following?
A. Inferior lateral pterygoid muscle. change DOES NOT occur relative to muscle A Arthus reaction.. A. Pulmonary compllance.
B. Superior lateral pterygoid muscle. fiber contraction? B. Tuberculin reaction. B. Alveolar surface tension.
C. Anterior temporalis muscle, A. Sarcomeres-shorten. C. Asthma. C. The work of breathing.
D. Masseter musde. B. Thick and thin filaments-shorten. D. Erythroblastosis fetalis. D. The tendl"ncy of the lungs to collapse,
218. Sleep bruxism (SB) is defined by many but not C. I Band-shortens, E. Serum sickness. 237. Red pulp in th~ spleen consists of _ _.
all of the following characteristics. Which is D. H zone-shortens. 230. The pancreas produces enzymes that are A. Fibroblasts
the EXCEPTTON? 223. Regarding the superior and inferior heads of responsible for the digestion of dietary B. T lymphocytes
A. Stereotypical movement disorder. the lateral plerygoid muscle (SHLP and lf!LP), compounds. Which of the following foods C. B lymphocytes
B. Grinding and c!ench!ng of the teeth during: which of the following statements is NOT would not be digested by enzymes synthesized D, Macrophages
sleep. TRUE? and secreted b.,v the pancreas? E. Chromaffin cells
C More frequent in the younger generation. A. Hypothetically, SHLP and IHLP can be consid- A. Carbohydrates. 238. The t'elocity of blood flow _ _.
D. lndtviduals who brux during the daytime ered to be parts of one muscle B. Lipid,. A. Is higher in the capillaries than the arterioles
inevitably brux at night. B. Dlstrlbutioos of SHLP and IHLP activities are C. Vitamins. B. 11 higher in the vPins than in the venules
219. Recent physiologic evidence suggests that shaded according to biochemical demands of D. Protein. C. Is higher in the veins than in the arterles
central and/or autonomic nervous systems tasks, 231. The primary sensory neurons' nucleus of 0. Falls to zero in the descending aorta during
rather than peripheral sensory factors play a C, SHLP stabilizes the condyle and disk against termination invo-lve.d in the jaw jerk reflex is diastole
dominant role in the genesis of sleep bruxism the articular eminence in a wide range of jaw the 239. Neuraminldase is prodnced by
(SB). Which statement about the central movements and iorces, A. Facial nucleus A. Tnf!uenra virus
genesis of SB is NOT true? D, IHLP plays a major role in the generation and B. Trochlear nucleus B. Hepatitis C viruses
A. During sleep the mouth is usually open due to fine control of horizontal forces. C Mesencephalic nucleus C. Human immunodeficiency virus
motor repression. ll Spinal trigeminal m1c!eus D. Measles virus
R Tooth contacl most likely occurs in associa- 224. The process of active sodium transport in the E. Nucleus of solitary tract E. Rubella virus
tion with sleep arousat ascending limb of the loop of Henle is 232. Altlosterone 240. IVhich sequence of eruption of permanent
C Some peripheral sensory factors may exert an absolutely essential for which of the following A. Stimulates N reabsorptlon in the distal and teeth occurs most often? (8-7-6-54-3-2•1 = M3-llf2~
Influence on SB through their intera~tion with processes? collecting duct! llfJ.P2•Pl-C-U-Cl). First#, #'sin eaclr se.-ies is
sleep-awake m~chani~ms, A. Regulation of chloride excretion. R Is 5ecreted by the juxtaglnmerular apparatus considered to be the first to erupt.
D. change from autonomic B. Regulation of pH ln extracellular fluld, C. Stlmulates K absorption in the distal tubule A. 6-l-2+3-5--7-8 (Maxilla).
cortical activities foHows SB~ C. Regulation of aldosterone excretion, D. Stimulates bicarbonate reabsorption in the B. 6-l-2-3--4-5-7-8
reiated jaw motor activity, D. Regulation of water excretion. proximal tubule C.
D. H>·Z·4·CrCH-iO
338 Y Sample Exam
Sample Exam T 339

241. All of the following are histopalhologic 249. The auricular hillocks are derived from the 256, Nephrolithiasis is most likely to be associated 264. The presence of Auer rods in a peripheral
features of malignant cells except one. J,Vhicl1
with which of the following conditions? blood smear suggests which of' the following
one is the exception? A. Fir'st branchlal arch A. Hyperparl'l.thyroidism. conditions?
A. Anaplasia. B Second branchial arch R. Myxed'°ma. A. Acute lymphocytk lt:\1kernl;1.
B. Pleomorphis:m. C. First and second branchial arch
C, An•uploidy.
C. Pyeloneph:ritis. B. Acute \yrnphnhl,,stk lz•ukt'!nla
D. Lateral nasal process D, Wilson's diseaie. C. Acute rnyrloge-ncn1s leukPmi.:1.
D. Large nuclei. E. Medial nasal process E. Thrombocytopeni..:L D. Chronic !ymptweytic !eukrmla.
E, Low nudear-cytoplasmic ratio. 250. Comparing the overall length of primary 257. The rate-limiting enzyme in glycolysis is which E. Hu,:lgkin's lymphnrna.
242. The vertebral artery meets with the basilar central incisors (E!F) with permanent of the following? 26S. Hormones secreted by the posterior pituitary
artery at the lower border of the _ _. maxillary central incisors (8! 9), which is the A. Fructose bisphosphatase. gland incltlfle which of the following?
A. Mldbrain correct ratio expressed as a percentage?
B. Pons
B. Phosphofructokinase, A. Prolactin.
A. About 50%. C. Phosphoglucose isomerase. B. Follide*stimulating honnone
C. Medulla B. About 60%. D. Glucokinase. C. Luteinizing hormone.
D. Temporal lobe C. About 70%.
E. Cl 258. The type of collagen characteristically found D. V,~sopre1sln.
D. About 80%. in cartilage is which of the following? 266. Which of tht! following is the predominant
243. Which of the following is not invofred in the 251. Ehle,-s•Danlos syndrome is a disease affecting A. Typ• I. immnnoglobulin in whole s,lliva?
process of gene cloning?
B. Type II. A. Secretory lgA
A. DNA polymerase. A Bone
B, RNA ligase.
C. Type ill. B. Secretory lgG.
B, Connective tissue D, Type IV. C. Se\:retory lg\1.
C. RNA polymerase. C, Muscle 259. A most characterb;tic feature of the primary D. Secretory lgB.
D, Re111triction endonudease. D. Joints maxillary central incisor is which of the 267. Which of the following is not an arch trait of
244. Which of the following is the most significant E. Glycogen synthesis follouiing? the maxillary canine?
stimulant of the respiratory center? 252. Which of the following participate In both A. Faciolingual breadth of the crown. A. tn the sJme dentition, lhe crown is larger than
A Decreased blood oxygen tension. fatty acid biosynthesls and b-oxidatlon of fatty B, Mesiodistal width of the crown. the mandibular canine.
B. Increased blood hydrogen ion concentratlon. acids? C. '.\1esial and distal margin ontHnes in iine with B. The incisal margin of the crown occupies at
C, Decreased blood hydrogen ion concentration. A. Malooyl CoA. profile5 of root. least one thlrcl to one hali of crown height
D. Increased blood carbon dioxide tension, B. fl\D. D. Root/crown ratio. C. Labia! aspect: mesiai and distal mMginal
245. The infraorbital nerve l$ a branch of the C. Acetyl CoA. 260. The spread of an odontogenic infection to ridge! converge toward cervix.
D. NAD. which of the following spaces would MOST D. Marked symmetry of mesial/dlstal halves
A. Optic nerve 253. Insulin produces which of the following likely be crmsidered life-threatening? when viewed from indsal.
B. Ocuiomotor nerve changes in mammalian cells? A. Submandibular space 268. During exercise, which oF the fo{lott1-ing is
C. Ophthalmic nerve
A. Increase in liver glycogen production. B. Sublingual space decreased?
D. Maxillary nerve B. Jnc.rease in blood glucose concentration.
E. Mandibular nerve C. Par.pharyngeal space A. Oxidation of fatty acids.
C. Decrease in the transport of glucose into D. Retropharyngeal space B. Glucagon release.
246. Root bifurcation would be a more likely finding muscle, E. Pterygonumdibular S(Jace C. Glycogenolysls,
in which of the following permanent teeth? D. Increase in the transport of glucose into the 261. The most common mutation accounting for fl Lipogenesls.
A. MaxiHary canine. brain. the pathogenesis oftrisomy 21 is _ _ . 269, Involution of the thymus would occur
B. Mandibular canine. 254. Osteoeytes are found in _ _ in mature hone~ A. Chromosome translotation following which ·year in a healthy
C, Maxillary central incisor. A. Trabeculae B, Meiotic nondlsjunction indittidual?
D, Mandibular lateral incisor. B. Lacunae C. Mitotic nondisjunction /\, 0 years (at birth),
247. Which one of the following carbohydrates is a C. The central canal D. Single deletion B. 12th year.
ketose sugar? D. Canaliculi E. X-linked inheritance C. 20th year.
A. Galactose. E. Spicules 262. What type of collagen is found in cementum? IJ 60th ye.er.
B. Fructose. 255. Which one of the following morphological A. Type I collagen. 270. Monoamine oxidase (J1AO) _ _.
C. Glucose, characteristic is representative of all B. Type II collagen, A, lna<:tlvates reduced steroid deriv.:1tives
D, Mannose. posterior maxillary teeth? C. Type Ill collagen. [L Is activnted by MAO inhibitors
E. Glyceraldehydes. A Marked mesiaI concavity on crowns and D. Type IV collagen. C, Inactivates catf'cholamincs l>y oxklMive
248. Which of the following antimicrobials is roots, E. Type V collagen. deamln;tt!{)n
bacteriostatic and inhibits protein synthesis B, Tips of cusps are weH within the confines of D, ls located ln the syrwps1: where lt lnacllvatt'.S
263. What happens to net filtration in the
in bacteria? the root trunks. glomerulus when plasma protein the neurotransmitter acetylcho!ine
A. Streptomycin. C. From mesial/dfstal aspect, crowns are rhom- concentration is decremwd? 271. Sleep bruxism can be characterized by which
B. Penicillin V boidal In shape. A. Net filtration (ultraiiltration) incr~ases. of the Following?
C. Ciprofloxacln, D. From mesial/distal aspect, all maxillary poste- B. Net ftltraHon (ultrafiltration) decreases. A Episodes of ma~sive. hil.::1!('.ra! clenching.
D. Cephalexin.
rior crowns are trapezoidal with shortest C. Net filtrntion remains unchanged. 8. Tooth grtnding thJI may la:-t for up to 20 min-
£. Tetracydine. uneven side toward occlusa! surface. D, Net filtration ceases. utes.
C Often co!nddes ,villi passag-e from lighter to
deeper sleep_
D, Occurs upprnxlmately every 20 mlr:utes in the
sleep cycle,
340 "' Sample Exam Sample Exam ,,. 341

272. An endocrine disorder that causes an early 280. In addition to the esophagus itself, which of the 288. Which one of the following is not a normal 296, Porphyn'ns use which amino acid in their
loss of primary teeth and the early eruption of follmtting structures also passes through the anatomical feature of mandibular synthesis?
secondary teeth is _ _. diaphragm through the esophageal opening? incisors? A Alanine,
A. Myxedemn A. The aorta. A Bifurcated roots. B. Phenyla!aninr-
ll Hashimoto·s thyrolctit!s B The Inferior vena cava B. lnconsplcuous cingula. C. Cysteine.
C. DiGeorge's syndrome C. The azygo.s vein. C. Four develo[)mental lobes, D. Glycine.
D. Plummer's disease D, The posterior and anterior vag:al trunks, D. lncisal edges placed slightly !ingua!ly. 297. The dental lamina arises from _ _ .
E. Dwarfism E, The splanchnlc nerves. 289. The differentiates into ameloblasts. A. Somites
273. Which of the following factors will not 281. Which one of the following does not release A. Stellate reticulurr. B, Neural crest cells
influence the rate at which a meal will leave acetylcholine? 8. Inner enamel epithelium in the cap stage C. The first branchia! arch
the stomach? A, Sympathetic pregangHonic fibers. C. Inner enamel epltheHum in th€ bell stage D, The second branchia! arch
A. Acidification of the duodenum. B Sympathetic pos.tgangllonic fibers that inner~ D, Outer enamel epithelium in the cap stage E. The buccopharyn~eal membrane
B. Increasing thE.' tonicity of the inte~tine. vate the heart. E. Outer enamel epltheHum in the bell stage 298. Karyotyping can b~ used to diagnose which of
C. Saline in the duodenum, C. Parasympathetic postgangtionic fibers to 290. Which of the following statements is true of the following diseases?
D. Lipid in the intestine. effector Ort.Jans. the hi..;tology of the trachea? A. Klinefelter's syndrome.
274. Occlusal interferences can be defined by all of D. Parasympathetic preganglionic fibers, A. The mucosa is covered with oral epithellum, B. MultiplE' rnyeloma.
the following except: 282. An infant diagnosed with osteopetrosis has B. Elastic cartilage rings lie deep to the submuco5e.. C. Niemann-Pick disease.
A. Occlusal contact relations that interfere with dysfunctional _ _. C, The cartHage is ring-shaped; the open end of D, Pemphlgus,
function. A Chondrocytes the ring faces ,mterior. £. Peutz-Jeghers syndrome.
B. Interference to jaw closure Into the intercus~ B. Osteoblasts D. The cartilage is covered by a perichondrium. 299..'lfuscle spindle stretching when the patellar
pal position. C Osteoclasts E. Skeletal muscle extends across the open end tendon is tapped produces which of the
C Interferences to laterotnisive movements. D. Fibroblasts of each cartilage. following responses?
D. interferences to jaw opening. E. Lymphoc,1es 291. In pemphigus, autoantibodies are directed A. M11scle contraction within muscle where the
275. The olec:ranon fossa is located on the 283. Langerhans' cells are located primarily in against which of the following structures? splndles are located.
surface of the _ _. stratum _ _. A. Acetykholine receptor. B. lncreased sympathetic stimulation of the spin-
A Superior; radius A. Corneum B. Sarcomere. dles.
8. Anterior; humerus B. Lucidum C. Epidermis. C A reduction ln the m1mber of aff0rN1t impulse-s
C Posterior; humerus C. Granu!osum D. Thyroid follicle, entering the spinal cord,
D. Anterior: radius D. Spino.sum E. Lysosomi"..s. D. An inhibition of the stretch reflex.
276. Ur1,ich of the following is usually least E. BMale 292. Which one of the following is elevated in 300. In terms of vertical dimension, where is the
malignant? 284, Which of the follouring woul,J be expected to plasma during the absorptive period mental foramen found most frequently?
A. Acute lymphoblastic leukemia. raise blood pressure? (compared to the postabsorptii:e state)? A At the apices of the premolars.
B. Acute lymphocytic leukemia. A A drug that inhibits the angiotensin A. Chylomicrons. B. Coronal to the af}ices.
C. Acute myetogenous leukemia. lkonverting enzyme and thus the productlon B. Acetoacetate. C. Below the apices,
D. Chronic lymphocytic leukemia. of anglotensln ll (ACE lnhlbitors). C. Lactate. D. No particttl.t1r location predominates,
£. Chronic myelogenous leukemia. R A drug that inhihiti the synthesis of nitric D. Glucagon. 301~ Urinary filtrate is most hypotnnic in the _ _.
277. Each of the following describes collagen oxide. 293. Recent focus on causative factors in bruxism A. Pnn.:imal convoluted tubule
except one. Which is the exception? C. A drug that btocks va!!:opressin receptors, include AU, of the following Except? B. Descending limb of Henle'::> loop
A. Most abundant protein ln the body. D, increased stimulatlon of the carotid baro- A. Ocdusal interferences. C. Thin segment of Henle's loop
8. Modifications to procollagen occur in the receptor. B. Part of a .sleep arousal response. D. Thick ascending segment o[ Henle·s loop
extracellular matrix. 285. lHwn are the crowns of the primary maxillary C. Piithophy.~iologlc.al factors, E. Dista! convoluted tubule
C. Incorporates hydroxyproline ?nto the molecule second molars completed? D, Neurotransmitters in the central nervous sys~ 302. There are _ _ pairs of true ribs.
byTrna. A, 11 months. tern. A Four
D. Hydroxylation of proline requires "itamin C B. JO months. 294. A distinet central developmental groove, B. Five
and molecular oxygen. C. 9 months. prominent buccal triangular ridge, two cusps C. Seven
278. Wllich position of th~ mental for-amen relative D. 8 months. and distinct mesial and distal occlusal pits D, Eleven
to the mandibular premolars and first molar 286. Which of the following processes is not a true would be most characteristic of: E. Twelve
occurs most frequently? component of swallowing? A. Mandibular first premolars. 303. Based on a11erage MD diameters of the croums
A, Between the first and second premolars. A. Closure of the glotti:a. B. Primary mandibular first molars. of primary teeth, the range for average oceralt
B. ln tine with the Sf'cond premolar. B. Involuntary rt>laxation of the upper C, Primary mandibular second molars. length of the primary maxillary arch is about
C. Distal to the second premolar. e~ophageal sphincter. D. Mandibular second premolars. what dimens;on?
D. In line with the mesi<\I root of the first molar. C, Involuntary movements of the tongue against 295. An autoclave sterilizes dental instruments by A. 60-68 mm.
279. From the lingual perspective the primary the palate. causing which of the following? B. 68-76 mm.
maxillary first molar ltas which of the D. Esoplv1g0al peristalsis. A. Coagulation of proteins. C. 76-84 mm.
following characteristics? 287. Which of the following receptors are B. Denaturing of proteins. D. 8·1-92 mm.
A. Distollngual cusp is the most prominent cusp, recognized by CDS lymphocytes? C. Precipitation of nucleic adds.
B. l\fosiolingual cusp poorly defined. A. Class I MHC molecules. D. Disruption of cell membranes,
C. Dlstobuccal cusp cannot be seen from lingual B. Class 1l MHC motecu!es. E. Dissolution of lipids.
aspect. C. Surface
D. Crown converges consirlerab:y in a lingual D, Surface
direction. Histamine receptor.
342 v Sample Exam Sample Exam "' 343

304. Symptoms of a myocardial infarclion include 1


312. ll hich primary tooth generally erupts last? 320. A positive quelling reaction can be observed 327. ATP is utilfaed direct(y for t::.ac!t of the
all of the following except one, Which one is A. Mandibular second molar, in bacteria with a follou•ing processes except:
the exception? B. Maxillary second molar. A. Thick peptidog!ycan layer A. Acnmrnla~Jnn of Ca~· by the ltarcopl:t~rnk
A lrngina, C. Maxillary canine. B. Capsule reticulum.
R Diaphorcsis. D, Mandibular canine. C. FlaKella B. Transport of Na· from intracellular to extni.cel•
C. Fever. 313. The latissimus dorsi muscle is supplied by tlte D. C~ll wall that contains teichoic acid Jular fluid.
D. Vomiting. _ _ nerve. E, Giycoca!yx coating C. Tran~port of K· from cxtrncellul~ir to intrncel•
E. Dyspnea. A. Medial pectoral 321. In which of the following might arterial blood lular fluld.
305. Ur'hich process transports amino acids across B, Cranial nerve Xl pressure be abnormally high? D. Transport of H· from parietal ce!ls into the
the luminal surface of the epithelia that lines C. Dorsal scapular A. Ventricular fibrmation, lumen of the !>tomach
the small intestine? D. Thoracodorsal B. Heart failure. E. Transport of glucose into musde cells.
A. Simple diffusion. 314. Which of the following responses is due to the C Anaphylactic shock 328. All of th<' follmriag microb<>s listed are
B. Primary active transport. stimulation of a-adrenergic receptors? D. Increased intrac.ranial pressure. assodated with infections secondary to an
C. Cotransport wlth sodium ion. A. Slowing of heart rate. 322. All of the following can be found in the cell HIV infection except one. lVhich one is the
D. Cotransport with chloride !on. B. Constriction of blood vessels in skin. wall of a gram"ftegative bacterium except one. ex~eption?
306. Which of the following cells are capable of C. Increased gastrointestinal motility. Which one is the exception? A. Pneumocysiis Jitoved (carinii)
mitosis? D. increased renal blood Dow. l\.. Endotoxin. B. Epstein,Barr virus.
A. Smooth muscle. 315. Which of the following contributes primary B. A thin peptidoglycan layer. C. Cox~acklevirus.
B. Skelelal muscle, sensory innervation to the C. Lipopolys.accharide. D. Myco/xrcteriwn tubern:losis.
C. Cardiac muscle. temporomandibu/ar}oint? D. Teichoic acid. E. Condida albicans.
D. Type I pneumocytes. A, Aurlculotempornl nerve. E, 0 antigen. 329. Which of the following is not a type trait of
E. Neuron~. B. Infra.orbital nerve. 323. The articulating surfaces of the the permanent maxillary second premolar?
307. Vitamin K serves as a coenzyme for: C. Branch of the lingual nerve. temporomandibular joint are covered with A. Buccal view: narrow shoulders (margins of
A. The enzymatic hydroxylation of proline to 4~ D. Facial nerve. crown; mesio- and disto-occlusal angles).
hydroryproline, 316. All of the following arteries are branches of A. Fibrocartilage B. Occlusal table outline: ovoid.
B. The c.arboxylation of lnactive prothrombln to the mandibular division of the maxillary B, Hyaline cartilage C. Mesiom;irginal groove interrupts mesial mar-
form active prothrombin, artery except one. Which one is the exception? C. Artlcular cartilage ginal ndge,
C. The synthesis of nucleic acids. A. Incisive artery. D. Elastic cartilage D. Lingual view: buccal profile is not visible.
D. Protein synthesis. B. Suhmental artery. E. Perichondrlum 330. What is the correcl schematic outline of the
308. Squamous cell carcinoma is the most common C. Middle meningeal artery. 324. The binding of epinephrine or glucagon to the follmcing teeth?
oral cancer. It is a tumor of D. Mylohyoid artery. corresponding membrane receptor ha,s u:ltich A Mandibular premolnrs, viewed from occlusal,
A. Melanocytes E. Deep auricular artery. of the following effects on glycogen rhomboidal.
B. Basal cello 317. Rheumatoid arthritis is characterized by metabolism? B. Maxlllary central lndsors, viewed from fodal.
C. Fibroblasts innammatian of the _ _. A. The net synthesis of glycogen is increased. triangles.
D. Kerntinocytes A. Articular capsule B. Glycogen phosphoryla~e iJ activated while C. Maxillary lateral incisors, viewed from mesial,
E. Macrorhages B. Articular cartilage glycogen synthas.e is inactivated. trapezoidal.
309. Which of the following is NOT a type trait of C. Cortical bone C. Glycogen pho$phorylase is inactivated \•vhHe D, Al! mandibular posterior teeth, distal aspect,
the permanent maxillary first premolar? D. Perichondrium glycogen synthase is activated. rhomboidal.
A. Occlusal table outline, trapezoidaL E. Synoviurn D, Both glycogen. synthase and phosphorylase 331. Which of the follou1ing consists of glucose
B. Generally two roots-mesial and distal. 318. Which of the following are NOT type traits of are l'lCtiva.ted. molecules linked together that act as the
C. Central groove is long. permanent mandibular first and second E. Both glycogen synthase and phosphorylase structural component of plaque?
D, Supplementary groo~es are rare, premolars? are lnac.tivate<l, A Fructose.
310. Which of the following skin lesions is most First Second 325. If posterior teeth on the left side contact B. Sucro-s:e.
likely premalignant? Premolar Premolar occlusally during a right lateral excursion of C Levans.
A Verruca vulgaris, A. Buccal Crown bilaterally Bilaterally the mandible, the left side occlusal contact D. Dextrans.
B. Keloids. view: Asymmetrical symmetrical WQufd be referred to as: E. Fructans
C Seborrheic keratosis. B. Lingual Entire buccal Buccal profile A. Laterotrusive contact_ 332. Hydroxyapatite _ _.
D. Actinic keratosis. aspect: profile visible not seen B. Protrusive contact. A. ls weakened if fluoride is substltuted for some
E. Compound nevus. C. Lingual Mo~t of occlusal Little, if any C. Mediotrusive contact of the hydm::-.yl ions
311. Which of the following statements regarding aspect: surface visible seen D. Centri<" relation. R Is a noncrystalline structure
the hormone secretin is true? D, Lingual Contour height: Cervical third 326. Bfood from the internal carotid artery C. If containing carlmnat!? ion becomes more sol¥
A It is responsible for activating ch)"-motrypsinogen, aspect: Middle third reaches the posterior cerebral artery by the uble
B. lt stimulates the release of pancreatic secreM 319. Which of the folfowing muscles of the back is D. Is composed of cakinm ;rnd phosphatr: in .a
tlon rich in bicarbonate. supplied by the CN XI? A, Anterior cerebral artery 1:1 ratio
C. It stimulates the release of pancreatic A. Levator scapulae. B, Anterior comrnunicnting artery 333. Tooth enamel is <lerfred from _ _.
enzyme!. B. Latisslrnus dorsL C. Posterior communicating artery A End0df'rm
D. It .'!ltimulates the contraction of the galibl;1cider C. Trapezius. D. Po.sierior superior cerebellar artery B, Mesoderm
to release blle, D. Major rhomboid E. Basilar artery C. Ectoderm
E. Minor rhomboid. D. Fndo<lerm and mesuderm
E Ectoderm and mt:soderm
344 .,. Samptc Exam Sample Exam T 345

334. Which portion of uriniferous tubules contains 341. The minimum volume of air that remains in /149. The Y.shaped central developmental groove is 356. lVhich of the fnllou:lng coenzymes are
squamous epithelial cells? the lungs after a maximal expiration is most likely found on whtC'h of the following incolr:ed in the metabolism of pyruvate to
A.Proximal convoluted tubde. termed the _ _. premolars? acetyl CoA?
FtThick descending limb of Henle's loop. A Tidal volume A Maxiitary first. A. Th1arnio pyrophospharn, Upoic Jcid. FAD.
C.Thin segment ot Henie ·s loop. B. Functional residua! capacity B. Mandibular first. NAD, and coenzymt A
D.Thick a~cending segment of Henle's loop. C. Residual volume C. Maxillary second. R NAD, tetrnhydrnfol:ne, lipok acid, FAD, and
E.Distal convoluted tubule. D, Vital capacity D. Manctihul~r second. vitamin 13 12 .
335. Which of the following enzymes found in the 342. Fram a mesial perspective, the primary 350. The sternal angle between the manubrium C. Mg.,, !:AD. nicntinamide. adenine rHrmc!eotide.
liver is involved in gluconeogenesis during the maxillary ffrst molar has wltich of the and the sternum marks the position of the and btotin.
postabsorptive state? following characteristics? rib. D. Coenzyme A. niacin. FAD, and ascorbic acid,
A. Glucose 6~phosphate dehydrogenase. A. Pronounced convexity on the buccal outline of A. First 357. The primary mandibular first molar has
B. 6~phosphogluconate dehydrogenase. the eervical third. B. Second which of th<> following characteristics?
C, Glucose &--pho!lphata~~- B. The cervical line mesially shows some curva~ C. Third A. Resembles other primary and permanent
D. Glucokinase, ture in an apical direction. D. Fourth teeth.
336. The heights of contour of the distal surfaces C. The dimension at the occlusal third is the E. Fifth B. From the occlusa! perspective. has n
of permanent mandibular central incisors are same as at the cervical third. 351. Which compound is produced in the hexose heartshaped outline
located in wltich coronal third? D. The mesiohuccal CU!!p is longer and sharper monophosphate (pentose phosphate) C The rnesiobucral cusp is smal!er than the dis-
A. ~1idclle. than the mesiolingual cusp. pathway? tobuccn! cusp.
B. CervicaL 343. The embryo develops fr'om the _ _• A. ATP. D. No developmental groove is evident ])etween
C Occ!usaL A, The entire blastocyst B. NADH. the bucc.:~I cusps.
D. Incisal. B The entire trophoblast C. NADPH. 358. Which of the following muscle attaches to the
337. Accumulation of fluid in the pericardium C. The embryonic dlsc D. Fructose 1,6-bisphosphate. anteriur end of the articular disc of the
occurs most often with which of the following D. The extraembryonlc coe!em E. f'llosphoeno!pyruvate. temporomatulibular joint?
conditions? E. The moruJa 352. Which of the following is the most common A. Superficii:1l head of the medial pterygold mus-
A, Unstable angina. 344. Which primary tooth is generally accepted as cause of subacutc endocarditis? cle.
R C~trdiomyopathy. the first to erupt, and at about what mean A Staphylococcu!I aureus. B. Deep head of the mcdlal pterygoid muscle.
C. Myocarditis, age? B. 5itaphylococcus epidermidis.. C. Superior head of the lateral pterygoid
D. Acute perlcarditis. A Maxmary central incisor, 8 to l 2 months. C. Streptococcus viridans. musde,
E. Tamponade, B. ~axlllary central incisor, 7 to 9 months. D. Streptococcus pyogene.s. D. Inferior head of the lateral pterygold muscle.
338. The correct order of tooth formation is C. Mandibular central inclsor, 6 to 10 months. E. Streptococcus pneumoniae. 359. In a ctrsp-.fossa occlusal relationship, the
A. Amelobiasts form. odontoh!asts form, D. Mandibular central incisor, 8 to 10 months, 353. From the occlusal aspect, the primary maxitlary second premolar is most likely to
ameloblasts start to form enamel, odonto- 345. Blood levels of progesterone are highest maxillary first molar has which of the articulate with which of the following
blasts start to form dentin during _ _. follou:;ing characteristics? mandibular teeth?
B. Ameloblasts form, odontobiasts form, odonto- A. The follicular phase of the ovarian cycle A. Crown outline diverges lingually and diAtally. A. First premolar only.
blasts start to form dentin, amelohlasts start R The IuteaJ phase of the ovarian cycle B. Sman traverse ridge frequently present called B. Second premolar only.
to form enamel C. Ovulation an oblique ridge. C Canine and first !Jrernolar.
C. Orlontoblasts form, odontoblasts start to form D. Menstruation C, Four cusps are pre!lent. D. First .and second premol;)rs.
dentin, ame!oblasts form, ame!oblasts start to .146. The of the heart ,'s also known as the D. Mesla1 marginal ridge is thin and poorly develM B60. The presence of which of the following itt a
form enamel mitral valve. oped. patient's serum indicates that the patient is a
D. Ametoblasts form, ameloblasts start to form A. Right atrloventricular valve 354. Lymph from the mandibular incisors drain highly infectious hepatitis B carrier?
enamel. odontob!asts form. odontoblasts start R Left atrioventricular valve chiefly into _ _. A. HBsAg.
to form dentin C. Pulmonary valve A. Submandibular nodes B. HBsAb.
£. Odontoblasts form, ameloblasts form, odonto- D. Aortic valve B. Suhmental nodes C:. flBcAg
b!asts start to form dentin, amelobi;,sts start E. Tricuspid valve C. Superficial parnti<l nodes D. HBeAg.
to form enameI 347. The most common cause of death in diabetic D. Deep cervical nodes E. f!lleAb.
339. Reduction division occurs during the _ _ . patients is _ _. E. Occipital nodes 361. Which of the following statement$ regarding
A. First stage of mitosis A. Peripheral neuropathy 355. Ur'hich of the following may be observed in a salivary secrrtion is true?
B, Second stage of mitosis B. Pancreatic cancer child diagnosPd w{th rickets? A. ln general, saliva is rnore hypertonie than
C. First stage of meiosis C. Car<liovascular disease A Dark pigmentation on the oral mucosa. plasma,
D. Second stage of meiosis D. Kidney fa!tme B. Early eruption of teeth. B. As .~mlivary flow increases, biccwbnn.ate conM
E, Third stage of meiosis L Opportunistic infections C. Hutchinson's incisors. centration decreases.
340. All of rite following factors play a role in the 348. At what time is the crown completed for the D, Abnormal dentin. C. As salivary flow incrc<1Ses. lonk concentration
virulence of the microbe that causes whooping tooth indicated? E. Macroglossia, increases.
cough except one. Which one is the exception? A. Primary maxll!ary central incisor, ~1 weeks. D. Salivary secretion i:s regulated primarily by
A lgA protease. 8, Permanent maxillary central incisor, 2 to 3 hormonal ;;tinrnlatinn.
B. Hemagglutinln. years.
C Exoto:xin. C. Primary maxillary lateral incisor, 2 to 3
D. Capsule. months.
£. Pili. D. Permanent maxillary lateral Incisor, 2 to 3
years.
346 T Sample Exam Sample Exam Y 34 7

362. The pulmonary vein of the lung carries: 369. The trochlea of the humerus bone articulates 377. Calcium that enters the cell during smooth 385. For each type of tooth, the primary teeth
A. Unoxygenated blood from the lungs to the with the muscle excitation binds with which of the conttistently show which of the follo1l'ing
heart A. Ulna of the forearm following? characteristks?
B, Oxygenatf'd blood from the lungs to the heart B. Radius of the forearm A. Cttlmodulin. A. Greater rnesiodista! diameter relative to crown
C. Cnoxygenated blood to the lu.ngs from the C. Coronoid process of the ulna of the forearm B. Inactive myosin kinase. heigb t than permanent teeth.
heart D. Olecranon of the ulna of the forearm C, Troponin. B. An elongated appearance ot th~ primary
D. Oxygenated blood to the lungs from the heart E. Mediai epicon<lyle D, Myosln. crowns and roots.
E. O:;icygenated blond from the heart to the lungs 370. Increased formation of ketone bodies E, Actin. C. Crowns that are translucent white in color.
363. _ _ vertebrae are characterized by a during fa.sting is a result of which of the 378. The lumen of the gastrointestinal tract is D. Root trunk one~half that of crown helght.
heartshaped body. following? lined with 386. All of the following are rotator cuff muscles
A Cervical A, increased oxidation of fatty acids as a source A. ri.focosa except:
B. Thoracic of fuel. B. Submucosa A. Supraspinatous muscle
C. Lumbar B. Decreased formation of acety! CoA ln the liver. C. Muscularis externa B. lnfra.spinatous muscle
D. Sacral C. Decreased levels of glucagon. D. Fibrosa C. Teres minor musde
£. Coccygeal D, Increased glycogenesis in muscle. E. Adventitia D. Teres major muscle
364. Lipid micelles are stabilized by which of the 371. Nucleus ambiguus contain.v the cell bodies of 379. Which one of the following is found on the E. Suhscapularis muscle
following? which of lite following cranial nerves? crown of permanent mandlbular firSt molars 387. Which of the folfou:ing does not affect the
A. Hydrophobic internctions. A. Ill. IV, and V. but is not found on the crowns of mandibular mus.de tension produced daring
B. Hydrophilic interactions. B. VII. IX ,and X. second molars? contraction?
C, Interactions of lipid and water. C. VIL IX, and XL A. MB cusp. A The extent of motor~unit recruitment.
n lnteractlon of hydrophobic llpid tails with D. IX. X, and XI. B. Distobuccal groove. B. The proportion of each 5ing!e motor unit that
hydrophobic domains of proteins, E. !X. X, and XII. C. Lingual groove. is stimulated to contract.
365. The HIV virus binds directly to the surface 372. The participation of calcium in the D. DB cusp.

l
C. The number of muscle fibers contracting:.
receptors ofCD4 lymphocytes with _ _. contraction of skeletal muscle is facilitated or 380. Which of the following mediators aid in the D, The frequency of stimulation
A, Reverse transcriptase associated with which of the following? kilting of intracellular bacteria? 388. Cytochrome P450 enzymes may be found in
B. lntegrase A. Calcium release from sarcoplasmic A. Histamine. which of the following cellular organelles?
C. Hemaggiutinin reticulum. B. lnter!eukin-2. A. Mitochondria.
D. Giycoprotein 120 B. Calcium binding to the myosin heads, C. Catalase. B" Golgl apparatus.
E. Protease C, Active transport of cakium out of longitudinal D. lgG. C. Lysosome.
366. The maxillary sinus ot)erlies the alveola.7' tubules. £. Lysozyme. D, RJbosome.
processes in particular what teeth? D. Uptake of calcium by T-tubules. 381. Chromosomes line up at a cell's equator E. Endoplasmlc retlnrlum.
A. First and second maxillary molars. ,173. Jl'hat type oF vaccine i• used for Clostridium during which phase of mitosis? 389. A patient /,as a heart rate of 70 bpm.
B, All maxillary molars. tetani? A. Telophase. Her EDV (end-diastolic volume) is 140 mL.
C. First anrl second premolars. A. Capsular polysaccharides. B. Metaphase. Her ESV (endsystolic volume) is 30 mL
D. First and second premolars and first and sec~ R Toxolds. C. lnterphase. Calculate the CO (cardiac output) of this
ond molars. C. Killed bacteria. D, Anaphase. individual.
367. If jaw opening is divided into phases. and it is D. lmmunoglobulins. E. Prophase. A. 9800 mL.
assumed that the surfaces of the articulating E. No vaccine is available. 382, Following the production of Okazaki B. 2100 ml.
bones and disc are associated throughout jaw 374. Where is tire height of contour located relative fragments, which of the following is required C. 7700 enL.
openirlgj what is the relationship of the disc to the following teeth (iiiewed from the mesiaO? to close the gap between the fragments? D. 15.400 rnL
and condyie in the following phases? A. Facial surfaces of all molars, middle third. A. D'.'!A lii..e, 390. Which premolar would be the most likely to
A. In the very ea.rtiest phase, the condyle moves B. Llngual surfaces of all premolars and molars. B. DNA polymerase. have a single pulp horn?
forward before the disc. cervical thlrd. C. RNA polymera,e. A. Maxillary first,
B. Jn the early phase, the disc and condyle move: C. Lingual surfaces of molars and premolars, cer~ D. Reverse transcriptase. I3, Mandibuli,r first.
anteriorly in concert. vical or middle thlrd. 383. The maxillary nenx: passes through which of C. Mandihular second.
C In an intermediate phase, the condy!e moves D. Anterior teeth, cervical or middie third. the following? D. M21xillary .second.
forward nt a slower rate. 375. The gamma motor neurons control which of A Superior orbital fissure. 391. lVhich of thr following groups of
0. In the final phase, the dlsc moves forward at a tire following? R fnternal acoustic meatus, microorganisms produce dipkolinic acid?
faster rate. A, Muscle S!Jindle~. C. Foramen ovale. A. ActinomJ·cetes.
368. Endotoxin consists of _ _. B. Iris of the eye. D. foramen rotundum. R HL<.;topiasmo
A. Lipopolysae:charide C. Voluntary muscle fibers. E. Foramen spinosum, C, Streptococcus.
B. M protein D. Pyloric sphlncter. 384. A productive cough may be seen in all of the D. .5itaphylococcus.
C. Hyaluronh1ar.e 376. Which of the following organelles is following conditions except one. Which one is E, C/o.<;tridfum,
D, Lactic acid surrounded by a double membrane? the exception? .192. Which of the following describes the function
E. Coagulase A. Ribosome. A. Pneumonia. of RNA polymerase?
8. Golgi apparatus. B. Lung abscess. A. Trnnslntes DNA into protein.
C. Lyso5ome. C. Bronchledasls. B. Tennln.ates tran,cription.
D. Cytoplasmic inclusion. D. Asthma. C Removes lntrons during transcription.
£. ~itochondria, E. Bronchogenic carcinoma, D. Synthesizes RNA 5' ➔ 3'.
F. Ganglion cells.
348 • Sample Exam

393. Which of the following bones is formed by 397. The lateral thoracic u:all of the ax/Ila is
intramembranous ossification? cooered by which of the follou.Jing muscles?
A Humerus, A Pectoralis major.
B. Lumbar vertebrae. B. Pectoraiis minor.
C. Frontal bone of the sku!L C. Serrntus anlerioL
D. Ribs. D. Subscapularis.
E. Clavlde. E. Latlssimus dorsL
3,94. The pressure in a capillary in skeletal muscle 398. CO~ generated in the tissues ls carried in
is 37 mmHg at the arteriolar end and 16 ve,;ous blood prima,·ily in which form?
mmHg at the venular end, The interstitial A, CO., in the plasma.
pressure is O mmHg. The colloid osmotic B. H,CO, in the plasma,
pressure is 26 mmHg in the capillary and 1 C, HC0 3 in the plasma.
mmHg in the interstitial fluid, The net force D. CO, in the red blood cells.
producing fluid mot,ement across the E. Carhoxyhemoglobin in the red blood ceHs.
capillary w<,ll is which of the following? 399. Oral epithelium ia composed of _ _
A. l mmHg out of the capillary. epithelium.
B. 3 mmHg out of the caplUary. A. Keratiniz:ed 5impl~ squamous
C J 2 mm Hg out of the capillary. B. Keratinized .itr.titified squamous
D. 3 mrnHg into the capillary. C. Nonkeratinized simple squamous
395. Which of the fo//ou:ing forms of thyroid hormone D. Nonkeratinized .!';tratified ~quamous
is most readily found in the circulation? E. Nonkeratinized stratified columnar
A. Tri-iodothyronin• (T3). 400. Chondroitin sulfate is a major component of I. A. The inferior head of the lateral pterygoid muscle 8. C. lnt-ercalatecl discs are only found in cardiac mus-
B. Thyroxine (T4). which of the following? attaches to the lateral surface of the lateral cle. Multiple, peripherally po"-itlnned nuclei are
C. Thyroglobulin. A Bacterial cell waHs. pterygoJCI plate of sphenoitl bone. lts superior found in the fibers of skeleh:11 niusde. Smooth
D. TSH. B. Muci11. head att.'tches to the infratemporal crest of the muscle cells are ipin<lle-shape(I.
396. Dust cells can be found in the C. lgA. greater wing of sphenoid bone, The deep fibers 9. C The hypogiossal (CN XII) nerve i5 not found in
A. Brain D. Cartilage. of the medial pterygol<l muscle attaches to the the posterior triangle; it is, however, present in
B. Heart E. Hair. medial surface of the lateral pterygoid plate, the suhmandihular triangle. Contemts of the pos-
C, Lungs 2. D. The palatopharyngeus forms the posterior terior triangle lnclmie the external jugular and
D. Liver tonsillar pillar. It also functions to dose off sohclavian vein and their tributarles, the sub~
E. Spleen the na1opharynx and larynx during swallow- c-lavian artery and its branches, branches of the
ing. The anterior tonsmar pillar is formed by the cervical plexus, CN Xl, nerves to the upper limb
palatoglos5us. and muscles of the triangle floor, the phrenk
3. E. The superior and inferior ophthalmic veins nerve, and the brachia! plexus.
drain 1nto the facial vein and cavernous sinus. 10, C. Deoxygenated blood from the transverse slnus
4, C. The masseter originates from the inferior border drains to the slgmnld sinus, which empties into
of the zygomatic arch; spedflcally, its superficial the internal ju~ular velns. The transverse
head and deep head originate from the anterior sinuses receive blood from the confluence of
two thirds or posterior one third of the inferior sinuses, which is irn:ated in the poslerior cra-
border, re,pectively, Its superficial head inserts nium.
into the lateral surface of the angle of the 11. A. The vestigta! deft of Ra.thke's p0uch is !oc,Hed
mandible; Its deep head inserts into the ramus betwt-:en the anterior and posterior lobes-
and body o{ the mandible. spccifkally, betvveen the pars intermedia and
5. A. Lateral crkoarytenoid. The ohlique and tran~- anterior lobe. It con.sis.ts oi cyst-llke spaces
verse arytenoids and thyroartenold also adduct (Rathke's cysts) and represf'nts the ve.stigiat
the vocal folds. The posterior cric0arytenoids lumen of Rathke's pouch,
abducts the vocal cords, The cricolhyroid mus- 12. B~ The thymns is active llt birth and increases ln
cle raises the crlcold cartilage and tenses the size until puberty (around age 1~). after which it
vocal cords. graduaHy atrophies am! ls replaced by fatty tis-
6. A. The site of cen division (mitosis) occurs in the sue.
stratum basale (basal layer, stratum germina- 13. C. The Internal carotid artery is joined to the pos-
tivurn) of oral epitheHurn terior cerehrt'l:I £1rtery via the posterior co1nmu•
D. After branching from the mandibular nerve nicating artery, which is part nf the circle of
(CN V:J, the auriculntemporal nerve Irave!s pos~ Willis.
teriorly and encircles the middle meningeal 14. D. It is ii branch of the rnaxiilary (CN V.)) nerve. The
artery, remainlng posterior and medial to the maxillary nerve branches from th~e trigeminnl
condyle. It then continues up tuwards the TMJ, ganglion and exits the skull through the forami:~n
external ear, and temporal region, passing roiundum. Wh0n it reaches th<~ ptnygopal.ttine
through the parntid gland and traveling with the ganglion, it terminates as the infranrbital and
superficial temporal artery and veln zygomatic nerves.

349
348 T Sample Exam

393. Which of the following bones is formed by 397. The lateral thoracic wall of the axil/a is
intra.membranous ossification? covered by which of the following muscles?
A. Humerus, A. Pecto1alis major.
B. Lumbar vertebrae. B. Pectoralls minor.
C. frontal bone of the skull. C, Serratus anterior.
D. Ribs. D. Subscapularis.
E. Clavicle. E. Latissimus dorsi.
394~ The pressure in a capillary in skeletal muscle 398. CO~ generated in the tissues is carried in
is 37 mmHg at the arteriolar end and 16 ven~ous blood primarily in which form?
m.mHg at lite venular end. The interstitial A CO2 in the plasma,
pressure is O mmHg. The colloid osmotic B. H.1C03 In the plasma.
pressure is 26 mmHg in the capillary and 1 C. HCO,· in the plasma.
mmHg in the interstitial f1uid. The net force D. C0 in the red blood cells.
0

producing fluid movement ac,-oss the E. Carhoxyhemoglobln In the red blood cells.
capillary wall is whic/1 of the following ? 399. Oral epithelium is composed of _ _
A. 1 mmHg out of the capillary. epithelium.
B. 3 mmHg out of the capillary. A Keratinized simple squamous
C. 12 mmHg out of the capillary. B. Keratinized stratified squamous
D. 3 rnmHg into the capillary, C. Nonkeratinized simple s-quamous
395. Which of the following forms of thyroid hormone D. Nonkeratinized stratified squamous
is most readily found in the circulation? E. Nonkeratinized stratHled columnar
A. Trl-iodothyronine (T3). 400. Chondroitin sulfate is a major component of 1. A. The inferior head of the lateral pterygoid muscle 8. C. Intercalated discs are only found in cardiac mus-
B. Thyroxine (T4). which of the following? attaches to the lateral surface of the lateral cle, Multiple, peripl1erally po~itioned nuclei are
C. Thyroglobulin. A. Bacterial cell walls. r,terygoid plate of sphenoid bone. lts sur,erior found ln the fibers of skeletal muscle, Smooth
D. TSH. B, :\1.ucin. head attach•• to the inlratemporal crest of the muscle cells are tpindle-.'lhapecL
396. Dust cells can be found in the _ _. C. lgA. greater wing of sphenoid bone. The deep fibers 9. C. The hypoglossal (CN XU) nerve is not found in
A. Brain D. Cartllage. of the medial pterygoid muse.le attaches to the the posterior triangie; it is, however, present \n
B. Heart E. Hair. medial surface of the lateral pterygoid plate, the 5ubmandibular triangle. Contents or the pos-
C. Lungs 2. D. The palatopharyngeus forms the posterior terior triangle include the e_x.ternal jugular and
D. Liver tonsillar pillar. It also functions to close off subclavian vein and thelr trlbutaries, the sub-
E. Spleen the nasopharynx and larynx during swallow- clavian artery and its branches, branch-es of the
ing. The anterior tonsillar pillar is formed by the cervical plexus, CN XI, nerves to the upper limb
palatoglossus, and muscles of the triangle floor, the phrenic
3, E. The superior and inferior ophthalmic veins nerve, and the brachia] plexu5.
drain into the facial vein and cavernous sinus. 10. C. Denxygenated blood from the transverse sinus
4. C. The masseter originates from the inferior border drains to the sigmoid sinus, which empties into
of the zygomatic arch; specifically, its superficial the internal jugular veins. The transverse
head and deep head originate from the anterior sinuses receive blood from the confluence of
two thirds or posterior one thlrd of the inferior sinuses, which is locater! in the posterior era~
border, respectively. its superficial head inserts niurn,
into the lateral surface of the angle of the 11. A. The vestigial cle!t of Rathke's pouch is located
mandibie; its deep head inserts lnto the ramus between the anterior m1<l posterior lobes-
and body of the mandible. spedfically. between the pars intermedia and
5. A. Lateral cricoarytenoid. The oblique and trans- anterior lobe. It consists of cyst-llke spaces
verse arytenoids and thyroartenoid also adduct (Rathk.e's cysts) and represents the vestigial
the vocal folds. The posterior cricoarytenoids lumen of Rathke·s pouch,
abducts the vocal cords. The cricothyroid mus- 12. B. The thymue: is active at birth and increases in
cle raises the cricoid cartilage and tenses the size until puberty (around age 12). after which it
vocal cordsT gradually atrophies and is replaced by fatty tis-
6, A. The site of cell division (mitosis) occurs in the sue.
stratum basale (basal layer, stratum germina- 13. C. The internal carotid artery is joined to the pos-
tivum) of oral epithelium. terior cerebral artery via the posterior commu-
7, D. After branching from the mandibular nerve nicating artery, which ls part of the circle of
(CN V;;. the auricuiotemporal nerve travels pos- Willis.
teriorly and encircles the middle meningeal 14, D. lt is a branch or the maxillary (CN V.,) nerve. The
artery, remaining posterior and medial to the, maxillary nerve branches from o{e trlfleminal
condyle. It then continues up towards the TMJ, ganglion and exits the skull through the forllrnen
external ear, and temporal region, pas.Jing rotundum. When lt reaches the pterygoplllali11~
through the parotld gland and traveling wlth the ganglion, it tenninates ns the infraorbital ancl
superfidal temporal artery and vein, zygomatk nerves.

349
3 SO .,, Answer Key Ana:ornic Stie:no:n Answer Key Anatomic Scieno;s Y 351

15. /\.. Of tlie cell types :iste,d, only smooth muscle <.:ells nerves lo the cheek, hucn.1! gin_~iva of the poste- the lower abdominal wall. The celiac: trunk and 52. B. The s.uhment;,J artery is a branch uf the facial
are capable ol ceU dlv!sion. rior mandibular teeth, and bucc.a! mucosa. superior mesenterlc arteries are both unpaired artery. Branches of the mandibular division of
16. C,The acinar units o: salivary glands are lined by 26. A. The inferior alveolar nerve (;AN) courses branches to the gut and assodated glands. the maxillary artery include the inferior alveolar.
simp!e cuboirlal epithd:um. This type of epithe- between the s.phcnomandibular ligament and 36. D. Oral epithelium is composed of nonkeratinlzed, de-ep auricular, anterior tympanic, mylohytiid,
lium also lines ~he hrnr.chloles, thyroid gland. the ramus of the mandible before entering the stratified, squamous epithelium. inclslve, mental and middle meningeal arteries.
and ovuy capsule. mandibular foramen. The sphenoman(iibular llg- 37. 0. The carHlage of the trachea is covered by a 5:J. D. The maxillary nerve (CN V.,) t~xits the skull
17. B. The tendon of the ternmr tympani is attached lo arnent may therefore be damaged during the perichondrium. The mucosa is covered with through the foramen rotundt~m. lt then passes
the lwnclle of the mal!eus ln the middle ear. Loud admlnistration oi an IAN hlock. respiratory epithelium. Hyaline cartilage ring5 through the pterygopalatine fossa, where it com-
sounds cause the tensor tympani to cor;tract, 27. C, The lateral thoraclC wall of the a.xilla is covered lie deep to the submucosa. The open end of municates with the pterygnpalntine- ganglion
pulling the malleus and tympanic membrane by the serratus anterior muscle. The anterior the cartilage faces the posterior. Smooth mus~ Contents of the superior orbital fissure indude
inward to reduce vibrations ;md prevent damage wal; is. covered hy pectora.lls major and pec- cle extendS- across the open end of each carti• CN lll, JV, V 1 and VI and thi:: oph1halmic veins. CN
18. C The ratio of inorga,1k to organic matter in toralls minor. Latlssimus dors\ contributt>s to !age. Vil and VHI pass through the internal acoustic
mature dentin is Hpprm:imatdy 70::J0. ln enamel the inferior aspect of the posterior wa!L 38. B. Terminal bronchioles are characterized by cili- rneatus and C:'J V~ (the mandibulr.r nerve)
and cemf'T1tum, it is .npproxim<1te£y 96:4 and 28. A. The tmchlea of the humerus articulate-B with the ated cuboidal cells, passes through the foramen ovalc. The foramen
,S0:50. respectively. u!na of the forearm, The capitulum of the humerus 39, E. The most superficial layer of the epidermis ls spinosum i5 not associated with any cranial
19. D. Oligodendrocytes proc!uce the myelin sheath bone articulates with the radius uf the forearm. the stratum corneum. From deep to superficial, nerves; It contains the middle meningeal ves~
around myt>linated axons in the cent ml nervous The coronoid fogs.a, located just superior to the the layers are basaie, spinosum,. granufosum, sels.
system. Schwanr. cells make up the mye!in trochlea, fits the cornnoid proces:s of the ulna of lucidum, and corneum. 54, E. The ahducens nerve (CN V[} provides innerva-
slH:::nth around mye!inaled axons in the auto~ the forearm. The ,Jlecranon fo~sa of the humerus 40. D, Langerhans celts are located primarily in stra- tion to the lateral rectus: muscle, which moves
nornk: nervous system. flts tht> o!E;cranon of the ulna of lhe forearm. The tum spinos1.1m. the eyeball laterally, L,, abducts the eye. The
2(L D. The facla! nerve supplie!\ special ~,ensory (tasle) 1nedial epkonclyle is on the humerus itself and 41. B. Arteriovenous anastomoses in deeper skin are medial rectus muscle, which is innervated bv
to the anterior two thirds of the tongue, via one serves as an attachment site for musdes. important in thermoregulation. the oculomotor nerve (CN UI), is responsible fo-r
of lts branches, the chorda tympanl (Fig, 1-25). 29. C. The trapezius musde i~ supplied by CN XI. 42. C. The frontal bone of the skull i1 formed by adduction or the eyeball.
The chorda tympani branches from the facial Latissimus dorsi is supplied by the thoraco<lor• intramembranous ossification. The humerus, 55. D. The nucleus amhiguus is found in the medulla of
nerve, carrying both sensory fibers for taste and sal nerve, levator scapulae is supplied by the vertebrae, ribs, and clavide all are formed by the bralnstem. It contains the eel! bodies of
preg;mglionic: parasympathetic fibers. It exits dorsal scapular nerve, and the major and minor endochondral ossification. motor neurons for CN IX. X and XL The cell
from of the tempnral bone to jo!n the lingual rhomhol{I muscles are supplied by the dorsal 43. B. Osteocytes are found in lacunae in mature bone. bodies of CN VII, IX and X's sensory neurons are
nerve (a bnrnch of CN VJ, as lt courses inferi- sc:apu!ar nerv~. 44. B. Eplphyseal closure marks the end of growth ln contained !n the nucleus of the solitary tract.
orly toward the submandibular ganglion 30. C. There are seven pairs of true ribs. meaning they length of long hones. 56. A. The articulating surfaces ot the TMJ are covered
Postgangl!nnic parasympathetic fibers tmerge attach direrUy to the sternum via costal carti- 45. B. The branchial arches disappear when the sec- with fibrocartilage. dlrec:tly overlying perios-
from the ganglion and continue toward the sub- lages, The remaining five p~ir5 are called false ond arch grows down to contact the fifth teum. The non~artlculating surfaces of the TMJ
lingual and suhm,mdibular glands. Sensory ribs because they attacl1 Indirectly to the ster- branchial arch. are covered with periosteum. The articulating:
fibers also branch from the nerve and provide num via costal cartilages. The last pair does not 46. B. Facial nerves are derived from the second surfaces of diarthrodial joints are covered with
taste sensation to the <mtE'rior t,No thirds. of the attach at al!. branchial arch. The trigemfnal nerve is derived hyi'lline cartilage.
tongue. 31, B. Thoraclc vertebrae are characterized by a bf'art~ from the first branchial arch, 57. C. The me~encephalic nucleus contain the nuclei
2L B. Oh!ique olv('>n!odent;-1J flhers rPsist ocdusal shaped hody. 47. E. in the liver, smooth endoplasmic reticulum is of the trigeminal sensory nerves (CN V)
forces that occur along the long axis of the :~2. B. The sternal angle hctv,-een the manuhrinm and involved in glycogen metabolism and detoxifica- hwoived in proprioception and the law ierk
tooth. The rest of the a\veolodcnta! (POL) the stermun marks the position of the second tion of various drugs and alcohols; it therefore, reflex, including perlodo!ltal ligame~t fibers
fibers listed provide resistance against forcets rib, from this location ribs can be counted contains P450 enzymes, which are cytochrome~ involved in the reflex. lt is located in the mid~
which pull the tooth in an occlusal dired!nn externally, Th!s is important. hecause the first that are important in the detoxification process. brain ancl pons,
(i.e .. forces that try to pull the tooth from Hs rib cannot be palpated. 48. A. Type I collagen is the predominant collagen 58. D. The red pulp of the spleen consists of cords,
socket), 33. D. The rectus <1hdomlnu~ muscle nf the anterol;1t- found in cementum. Type lll collagen may be containing numerous macrophages, and venous
22 A. When pulpal nerves Me stimnlatcd, ihey can eral abdominal wall ls described a.s being belt~ present during the formation of cementum, but sinusoids. lt is the site of hlood fl!tration. The
oniy transmit one sign;t!: p~in. or strap-like. The remaining three muscles of the it is largely reduced during maturation. white puip of the spleen contains numerous T
23. C. The mylnhyoid muscle formi the floor of the i'lnterolateral abdominal wn.H (E'Xternal oblique 49, B. In smooth muscle, the binding of calcium to cal- and B lymphocytes.
mouth. RelaxaUon of this muscle would help the musde, internal oblique muscle. and trans- mondulin wiU activate the enzyme myosin light 59. R The two vertebral arteries join together at the
dentist push the film down, to help ensure that verses ahdom!nus muse!~) are all described .as chain kinase. This enzyme phosphorytates border of the pons to form the basilar artery.
the i'\pical root is captured on the rctdiograph. sheet-like, The quadratu.s lurnborum muscle is myosin. llllowing it to bind to actin, and the mus- Branches of the basilar provide blood supply to
2-1. B. Fibers of the !atcrn! plerygolr! muscle are part of the pol'itnlor nhdominal wall. cle contracts. For contraction in skeletal and the pons.
attached to the anterior end of the dist. :S4. D. Tbe posterior and anterior vagal trunks pass cardiac muscle, calcium binds to troponin C. 60. D. Calcitonin is secreted by parafollicular cells
Contraction of this muscle pulls the disc in an through the dlaphrni,;m through the esophageal 50. B, The mandibular incisors, as well as the lower lip, (clear cells) that are located at the periphi!!'ry of
anterior and medlal directior, opening. The aorta enters the diaphragm floor of the mouth, tip of the tongue and chin, thyroid follicles in the thyroid gland. Calcitooin
25. B. The sensory distrllmtinn for 1he maxillary nerve through the medlan arch, the inferlor vena c::rva prlmarily drain into the submental nodes. The play:,; an important role in the regulation of cnl·
(CN V.,) includes the cheek and uppe(· lip. and through its own oµeninl;l. in the central tendon, rest of the mandibular teeth (premolars and cium and phosphi'lte~. It suppresses bone reah-
lower ~Pyelid, uprwr lip, nas0ph11rynx, tonsils, the azy_go5 vein through the right crus, and the molars) mainly drain into the submandibular sorption, re:rnlting in decreased calcium imd
pnlate, and maxil!ary teeth. The sensory distri- ~pl<'lndmic nerves through the crurn. nodes, phosphate relea5e.
bution for the iong lmccal also include5 the 35. C. The inferior aspect of the diaphragm is supplied 51. C. Fibers of the superior head of the Jateral ptery- 61. B. During metaphase, mitotic spindle's form,
(lower) t:h~~k; however, it does not 1ndude the with hlood by the inferlor phrenic arteries. The gold musde attach to the anterior end of the Chromosornes attach to these spindles, \vith
upper lip" The long btH.:cal is a branch of the median sacral artl"'ry 5upplie!i the anterior aspect dlsc, which helps to baiance and stabilize the their centromeres aligned wltll the equator of
m;mdibular nerve (CN V) and provides sensory of the sacral are-a, and the lumbar arteries supply disc during mouth closure. the cell.
352 T Answer Key Ar1atomic Sciucc:~ Answer Key Anatomic Sciences T 353

62. B The oropharynx ls Hned by stratified squamous becomes the braehlal artery when lt leaves the 86. E. Crinary filtrate ls most hypotonk ln the distal coelem that will lead to development of the
epithelium. Thi!i- type ot eplthelium al5o lines the axi!la. convoluted tubule. lt ls lsotonk in the proximal amnion, vitelline sac, and chorion,
oral cavity, Jaryngopharynx, esophagus, vaginal 74. B. The pulmonary vein of the lung carries oxy- convoluted tubule and thick descending llmb of 93. C. Tooth enamel is derived from ectoderm. Dentin
canal and anal canal. genated blood from the lungs to the left atrium Henle's loop. it becomes hypertonic M lt passes and pulp are der'1ved from mesoderm.
63. E. The mitochondria is surrounded by a double of the heart. The pulmonary artery tarries through the thin descending limb o! Henle, 94. C. The olecranon fossa is located on the posterior
(inner and oute-r) membrane. The nuclear mem- unoxygen.ated blood from the right ventricle of loop, .and becomes hypotonlc as it pa~ae5 surface of the hume:·us.
brar.e (not listed), whith surrounds the nucleus, the heart to the lungs. through the thick ascending segment of Henle's 95. D. The latisslmus clorsi muscl0 ls supplied by the
also consists of a double (lnner and outer) 75. B. The left atrioventricular valve of the heart is loop. Finally, lt becomes increasingly hypotonic thoracnrlorsai nerve.
membrane. also known as the mitral valve. The right atri~ as it passes through the distal convoluted 96. C. The middle trnnk nf the brn.chial plexus of
(,4. A. The medulla of the thymua contains Hassall's oventricular valve of the heart is also known as tubule, nerves arises from C7.
corpuscles, which consist of epithelial cells with the tricuspid valve. The aortic valve prevents 87. C. The inner enamel epithelium in the bell stage <lif- 97. A. The first rib cannot be palpated.
keratohyaline granules. The medulla is the reguqJitation of blond from the aorta back into ferentlates into ameloblasts 9:-l. B. Odontogenic infections of a mandihutar indsor
lighter-stainini;t (le5s dense) central area of the the left ventricle and the pulmon?1.ry vaive pre- 88. B. The dental lamina arises from neural crest cells. with an apex belnw the mylohyoid muscle have
gland, where T cells maturation occurs. vents regurgitation of blood from the pulmonary 89. B. The correct order of tooth formation is the poten!ioJ to spread to the .submenta! space,
65. B, The fovea centralis only contains cone cells. lt is artery Lack into the right ventricle. ameloblasts form, Ddontoblasts form, odonto- lf the apex ls ahnve the mylohyoid muscle. the
located approximately 25 rnm lateral to the 76. C. The cricopharyogeus muscle prevent~ swallow~ blasts start to form dentin, ameloblasts start to infection would spread to the s11hlln_gual space.
optic disc: in an yellow-pigmented area (macu!a ing air at the pharyngeal end of the esophagu~. form enamel. Both of these spaces communkate wHh the
luna). Vision is most acute from this area. 77, A. The pancreas is enveloped at its head by the 90. C. The aurkular hmocks are derived from the first subman<ilbu!ar space.
66. C. The roof of the orbit consists of lesser wing of first part of the duodenum. and second brnnchial arch. 99 D. From the retropharyngP.al sp.'lce, Le ... dlmger
the sphenoid bone and the orbital plate of the 78. A. The bile canaliculi drain bile to interlobular 91. C. Reduction division occurs during the first stage space, .. odontogenic infections c,;:in quickly
frontal hone (not listed), ducts. The interlobular ducts form right and left of meiosis. The second st.age mirrors mitosis. spread down this space into the thorax (po:.te•
67. D. The elongation and overgrowth of filiform hepatic ducts. These ducts join to form the com~ There is no third stage of meiosis, rlor mediastinurn) and cause possible death,
papillae results ln hairy tongue. Filiform papil- mon hepatic duct. The gallbladder' arises from 92. C. The embryo develops from the embryonic disc. 100. E. The superior, middle, and inferior constrictor
lae are thin, pointy projectJons that make up the common hepatic duct. The: morula, blastocyst, and trophoblast all muscles all insert into the median pharyngeal
the most numerous papillae and gives the 79. C. The apex of .e. medullary pyramid in the kidney is include structures of the extraembryonlc raphe (the superior constrktor muscle was the
tongue ·s dorsal surface its characteristic rough called the renal papilla. The cortex is the outer only one listed), however, thelr origins differ.
texture. Note; a loss of fiHfonn papillae re~ults layer of the kidney. The medulla is the inner
in glossitus. layer. Minor calyces receive secretions from the
68. B. Tertiary dentin, or reactive/reparative dentin, is renal papillae. Several minor calyces join to form
dentin that is formed ln localized areas in a major calyx.
response to trauma or other stimuli, such as 80. D. Ureters travel inferiorly just below the parietal
caries, tooth wear or dental work Histologically. peritom:~um of the posterior body wall. TI1ey
its consistency and organization varies; it has no pass anterior to the common iliac arteries as
defined dentinal tubule pattern. they enter the pelvis.
69. E. Innervation to the maxlDary second molar, as 81. A. The lumen of the: gastrointestinal tract is lined
wen as the palatal and distobu,:cal root of the with mucosa. The rest of the choices are in
maxlllary first molar and the maxillary sinus, is order from lumen out. fibrosa and adventitia are
provided by the Posterior superior a!veolar synonymous.
nerve. The nerve is a branch of the maxillary 82. A. Gut-associated lymphoid tissue (GALD pro-
nerve (CN V,). duces secretory lgA.
70. D. The suprarenal glands secrete epinephrine. 83. B. The muscularj!I: extema has a third layer in the
Specifically, chrornaffin cells of the the adrenal stomach. lt is an lnner oblique layer of smooth
medulla, which act as modified postganglionic muscle. In the rest of the digestive tract, the
sympathetic neurons that synthesize. store and muscularis externa has two layers; an inner cir-
secrete catecholamines, produce epinephrine. It cular layer and an outer lonaitudinal layer.
also produces norepinephrine. 84. C. The thin segment of Henle's loop contains
7L D. The teres major muscle ls a shouJder muscle, squamous epithelial cells, The proximal convo~
however, it is not a rotator muscle. AU of the luted tubuie, also known as the thick descend-
other four listed ln this question are rotator cuff ing limb of Henle's loop, the thick ascending
muscles. segment of Henle's loop, and the distal convo-
72, D. The brachial plexus of nerves arises from five luted tubule all consist of cuboldal epithelial
roots from the anterior primary rami of spinal cells.
nerves C5 through C8 and TL 85. E. The macula densa is a component of the }uAia~
73. B. The right subdavian artery arises from the bra- g:Iomemlar apparatus which functions in regula- ·
chiocephalic artery, and the left subclavian tion of blood pressure. The proximal convoluted
artery arises from the aortic arch. The subda~ tubule. dlstal convoluted tubule, Bowman's cap-
vian artery becomes the axillary artery upon sule, and glomerulus all function in the produc~
crossing the first rib. The axillary artery tion of urine.
1. B. Cyclo-oxygenase includes tsoenzymes of 8. D. Unlike in noncompetitive inhibition, the inhibitor
prostaglandin endoperoxida~e syntha,;e, which competes for the same site as the substrate. lt is
is required tor the first step in the synthesis therefore possible ( with increased amounts of
of prostaglandins from arachidonic acid. substrate) to reach the V1,.._.. Apparent K.,., is
Phospholipase A 2, which ls involved in the syn- increused since more !ub:strnte is required to
thesis of arachidonic acid, is inhiblted by reach 1/2 Vrn~x (the definition of K1,J
steroidal anti-inflammatory agents. 9. C. NADPH is required as a reducing agent for the
2. A. The initial rate-limiting reaction involves the synthesis oi fatty adds. The hexose nwnophos-
removal of six carbons from cholesterol and phate pathway also produce.,;; ribose 5-pho~-
hydroxylation of the steroid nucleus to produce phate for nudeotid~ synthesis. No ATP or :\'ADM
pregnenolone. Pregnenotone can be further iso~ is produced in the pathway. Fructose t,&.bis~
merized and oxidized to produce the other pho!'tpllat~ is pro-duced during glycoly1is.
steroid hormones. 10. D. Oxidatlon of fatty acids, glucagon reltase, and
3. A. A miceUe is a globular structure that forms when glycogenolysis are all increased to provide
the polar heads of an arnphipathic molecule energy sources for exercising muscle. The syn-
(fatty adds) interact with the aqueous external the,sis oi lipid (llpoge:n~sls) would be decreased
environment and the nonpolar hydrocarbon during periods or exercise.
taUs are clustered inside. 11. A. During a fast. catabolism is increased to provide
4. B. A ketose sugar is on that contains a keto group. aidditionnl sources of energy. This ls eharacterH
Glyceraldehyde, mannose, glucose, and galac- !zed by increased utllization of fatty acids. When
tose are all aldoses since they contain an alde- the production of acetyl CoA {produced by
hyde group. enhanced f}-oxidation) exceeds the oxidative
5. B. Individuals suffering from mucopolysacchari- c.epacity of the citric acid cyde, ketone bodies
doses have normal production of proteoglycans are formed. During fasting conditions, glucagon
and glycoseminogiycan~ but due to genetic concentrations are increased and glycog~ne~t8
defects, Jack the enzymes which degrade is inhibited due to limited energy availability,
mucopolysaccharides. Acetyl CoA production in the liver is increast-d
6. E. Arginine is an amino acid that is deaminated to due to enhanced ~-o'.'t:idntion.
form omithine primarily in the liver as part of 12. C. Glucose 6Mphosphate dehydrogcnase and 6-
the urea cycle, Ornithine, argininosucclnate, pho.,:phogluconate dehydrogenase are irre-
aspartate and c!trulilne are generated in the ver!ible enzymes in the pentose pho.r.phat~
urea cycle but do not provide free ammonia for pathway. Glucokinase is involved with phos-
urea synthesis, phorylation of glucose when hepatic concentra-
7. B.. Both epinephrine and glucagons result in activi- tions of glucose are high. Glucoie 6-pho~phat~~e
ties which serve to increase (maintain) blood glu- hydrolyzes glucose 6-phospha.te to form free
cose. Activation of glycogen phosphorylase wili glucol'e, as the final step~-~~- glucorwog_e_l]esi~.
result in glycogen <legradatlon, ultimately provid- 13. B. Gluco,e transport in (h:~~~ a_g_~,~~
ing a source of glucose, Glycogen synthase inhi- tissue is under the influcncf! of(in~utin-senti:i-
bition results in decreased synthesi~ of glycogen, tive glucose transporters. AlthO.\i'gli sev1.~ral

355
Answer Key Sloche-rni~try and Pt;y,<ology .,. 357

metabolic pathways .are Influenced by insulm, teir.s in enamel that regulate crystallite growth. (I mmHg) at the vemtl.ar encl. The sum total of excl!cd the reahsorptive capacity (T") and it
glucose uptake !nto the liver ls not affected by Phosphoryns are initiator proteins that bln<l the pressures moving into the capillary is P., then appear~ in th1.:: urlne. Ur~a ls filtered amJ
insulin. minerals to facilitate nucl~ation and crystal (0 mmHg) and r:P (_26 mmHg} The net exchange passively n:nbsorbf'd to a slight degre0.
14 D. Phenylketonuria O>KU) is an inherlted disorder growth. pressure on the 21rterlolar end leads to ultrafH~ Creatinine is both freely filterNI atl(I secreted lo
of amino acid metaboli5m in which the affected 24. E. The transport of gluco!-e into masde (and fat) tration (12 mmHg). On the venular end, reabs- a sHght extent. Sodium and water are Wtered
Individual lacks enzymes to metabolize ph-eny~ ceJls is due to the activity of glucose transporter~ orption occurs (9 mmHg). Overall net exchange .and reab!)orhed to various degrees clue to phys•
!alanine. Albinism is a condition that re5ult5 in a (GLUT-4), which does not require ATP and Is inde- results in 3 mmHg of fluld los5 out of th~ capiH~ry. iological regulatl0r: of aldosterone and ADH.
defect in tyrosine metabolism and the inability pendent of any ionic concentration gradient ATP 33. C. Cardiac output is the volume of blood pumped 4(l D. In the ascending (distal; loop ot Henle, active
to produce melanin. Porphyria is an lnherited is required for the activity of all other transport per minute by each ventricle, This is influenced sodium absorption resu!ts ln an increased osmu~
disorcler lnvolving defects in heme synthesis. mechanlsms listed in the question. by cardiac rate and stroke volume ( end-diastolic larity of the interstitial fluid, which.i)fafs..a·m1e
Homocystinuria is a disorder in the metaholism 25. D. Facilitated diffusion does not require energy and volume - end~systo!ic volume). The cardiac out- in the retention of water under conditions of
of homocysteine resulting ln high levels of moves down a concentration gradient. Active put for this patient is 70 x 11n mL or 7,700 ml deh)'(_~ratlO:n:
homocysteine and methionine in plasma and tran&port, in addition to utilizing energy, is often per minute. 41. C. Under conditions of expan5in11 of txtracet-
urine. coupled to transport ions in both directions 34. B. The velocity of blood !low is the rate of dis- lular volume (long-term hyperl~osion). renin
15. B. Guanine base pairs with cytosine which, on a across the membrane. Both active transport and placement of blood per unit time. Velocity release and ald0s-terone secretion are reduced.
molar basis, will also equal 30?4. of the DNA. If facllit.ated dHfuslon are carrier-mediated and chanies inversely with cross-sectional area lm:rc-nsing ADH. i:mginten~in H, and increasing
guanine and cyto!.ine constitute a total of 60'.t) of therefore are influenced hy competitive inhibition. (cm 7). The greatt":r the area ln the vessels, the sympathetic ncUvity would result in an
DNA, the remaining percentage (40%) must be 26. B. The salivary glands are influenced by both the lower the velocity. Since blood is alway·s flowing increase in blood pressure,
equally divided between the base pairs of ade~ sympathetic and parasympnthetic (gloMopha- due to the rlistensnbll!ty of the arterlal tree, 42. D. Potas;;ium is passively (via the cotran~port sys-
nine and thymine (20':.; + 2()%_}. ryngeal and facial) branches of the ANS. The velocity is never zero, tem) secreted from the plasma into the distni
Ht A. Restriction enzymes are also known as restric- oculomotor nerve (CN JH) carries parasympa¥ 35. D~ Decreasing total peripheral resistance would ~nd cnrlectlng tubule~. Under conditions of aci~
tion endonucleases. These enzymes cleave D>IA thetic fibers to the eye which. when stimulated, produce greater reductions in blood press:ure. dosis, an individual may become hyperktdemic
at ,pecific sites to reiease fragments of DNA for produces constriction of the puplL The Due to the: baroreceptor reflex, cardiac heart since the kidneys will retain K' and secrete H;.
further analysis and charac:terization by com- para.sympathetic system innervates primarily rate would increase. lncrea5ed stroke volLJme Under conditions of aika\osis, K 1 sec-relion \Vi!l
plementary probes. smooth and cardiac musde, resulting In ipecifk ( due to increased venous return and sympa- be increased and H+ secretion reduced,
17. B. Tetrahydrofolic acid is the coenzyme form of activation and re~ponses, thetic stimulation of cardiac muscle) would al:to i3. C.
0 Saliva is formed by a prnce5s that first lnvolves
folic acid required for the synthesis of nucleic 27. B. Regulation of blood flow to skin is under the occur. Increased cardiac output would occur the formation of a solution by the at.:inar cells,
acids and normal cell division and replic.ntioo, control of factors acting locally (metaholites) due to increased heart rate and stroke volume, which !s subsequently modified by tiie·ifuCtTfe
18. A. DNA liga!.'le is required to ligate the fragments and u-adrenergk receptors. Slowing of the heart 36. C. Although some CO2 is transported unchanged in cells to produce a more hypoton\c solution
together. DNA and RNA pnlymernses are cata- and activation of gastrointestin~l motility are the plasma and in the red blood cells as car- (compnrecl to plasma). The modifiCatinn prima-
lysts that synthesize DNA and RNA in a continu- mediated through the cholinergic system, a- harninohemoglobin, most ls transported ln the rily involvl'!"~ the reabsorptio1~. QLsodium. and
ou5 proce&s. Reverse tnmicripta.,a~ i5 an enzyme adrenergic receptors produce decreal:ed renal form of bicarbonate ion in the plasma. chloride and secretion 01 pi>tassium and bkar~
found in viruses that makes DNA by using viral blood flow. 37. D, The most potent stimu!ant of the respiratory bonate. As s.ahvary flow incr~aset. the ductile
RNA. 28. ~ a r e the smallest of the sensory and center that increases the rate oi breathing cells have less time to modlly the compositio:1
19. C. RNA synthe!is. is not required for genetic motor fibers, They are postganglion.ic, UJl!!!Yeli- (hvperventilatlon) is increased CO~ tension. CO, of saliva, which results ln greater conc-e_t~~rntions
cloning. All the other enzymes are required to nated, and have the slowe"Slconduction velocity. is Permeab[e to the blood-brain b;rrier and ulti: of socliu.m-. mtd chlC?~~~Je. --· i(H\S. Bicarbonate
synthe,-iiize and splice DNA 29. A. Calcium is stored a"ndrefeased from the sar- mately produces an increase in H"' of the cere- concentration increases as sal!vary f!O\V
20. B. Vitamtn K is involved with the po~ttramdational coplasmic r-etlculum during excitation-contrac- bral spinal fluid. However. plasma H"' and HC(\- increases due to the selective stimu!.ntinn of
modification of glutamlc add to form y-carbox-y- tion coupling. This provides an extensive are not permeable and thus have little direct bicarbonate by the parasyrnpMhetk: system.
glutamic acid. This carboxylation permits pro- reservoir of calcium while permitting Intracellu- effect on the respiratory center. The respiratory The salivary glands are prii"!larily ur:der the
thrombin to interact wlth platelets and ions in lar free Ca2+ to be low when the muscle fiber is center is not sensitive to oxygen tension, unlik~ regulation ot' l::_.i}!h branches of the auton(11nlc
the procesg of clot formation, Hydroxylntion of at rest. The release of calcium is due to confor~ peripheral chemoreceptors. nervous system.
praline requires ascorblc acid and iron. mational changes which open Ca2"' channels in 38. A. Hemoglobin is a g\ohular protein responsible for 4·1. A. Secretory IgA i~ an lmrnunoglobulin tl:at is
2L D. Chondroitin sulfate is a gluco,aminoglycan the :o.arcoplasrnic reticulum. transporting most of the 0, in the blood. The unique to the oral C'~Wity. The secr~tory compo-
found tn ligaments, tendons, anrl cartilage, It is 30. A. Calmodulin is a calcium ➔ hinding protein in perc~nt saturation of hemogiohin i, n function of nent is synthesizt:d by s<1livary epitlu·lml celli
the most abundant glucosnminog!ycan in the smooth muscle which, when bound to myosin, the PO,, of the blood. The relation:,:hip of PO., and and complexe~ with lgA to form secretory JgA.
body. initiates contraction, Calmodulin activates hemoglobin saturation is not linear but, rather. 45. C. Vitamins are not d1gPstf'.<i by enzymes trom the
22. A. The a chain$ are polypeptides which character- myosin kinase, which results in myosin-ac-tin sigmoidal. Percent saturation increases greatly pancreas; hmvcTer, digC'~tion nf carbohydrntcs,
ize the collagen molecule. The most ahunda!1t crosslinklng and contraction of smooth muscle. at low PO> and less at higher PO,,. Increased fats, and prot~in!'t rn.ay be required to n:1ui:~ vita~
amino acids in collagen are praline and glycloe. 31. B. A motor llnit il'I composed of a single motor neu- PCO2 , temPer~ture. diphosphog!yc-e;ate. and H' mins 2ivai!t,hle for abso:ption. The pancreas
Proline (due to its lmino ring structure), inter- ron a?~-- the.. m1,1_scle fibe~_s_jt,ianerv~tes. _Since decrease the affinity or hemoglobin for 0 1 but do produces amy!Mt· for carbohydrate digestioa.
rupts the a chain, resulting in thf: hending/twlst- the n,o!or neur9o·wilL1Hi11uli:1t~ a_ll_ ~':'s~I_e_fi.Ders not alter the charaneristk slgmofdal m1ture of !ipl'l5e for lipid digestion, and sever;:;] prott:olytic
ing of the peptide. Glyclne (due to its small size) it tnrierVAle5, ;,f~act1005" oi a-motor ~unit C~nnot the saturation curve. cn2yn1c3 (trypsinogen. chymotrypslnogen, and
fits Into the smaller spaces of the peptide and is be stimulated. The number of motor units 39. E. Renal clearnnce is the volume of plasma com- procarhoxy--prptidase) for the dige:::;tion of protein.
poi,itione1d at every third position in the chain. recruited, the number of muscle fibers contractD pletely de~red or ll substance by the kidneys per 46, B. St-cretin :c1tlmulates bicarbonate secretion from
2:1. C. Although fluoride may be incorporated in min- ing, and the frequency of stimulation will all unit of time. The renal clearance of glucose is the p,nncreatic ducts. Cholecystokinin (CCK) is
eralized tissues, it ls not required for the miner- aHe-ct the degree of muscle tension prnduced. usually O since. although it i.s filtered, it is comM responsible for stimulating p;mcreatk enzym~
alization process. Matrix vesicles produced by 32. B. The sum total of the pre&.'iures moving fluid out of pletely reabsorbed when the plasma glucose secretion and contraction of the gall bladder.
osteoblasts are considered to be the initial site the capillary is P, (37 mmHi) and x" (I mmHg) concentration ls within normal levels. In the Chyrnotrypsi~ogen i~ activated by trypsin in the
of mineralization. Ame!ogenlns are matrix pro- at the arteriolar end and Pc (16 mmHg) and rr1i uncontrnlle-rl diabetic, plasma glucose levels lntestine.
358 • Answe-r Key
Answer Key Bioch,:smistry ,md Physiology T 359

D, 56. C. Acetyl CoA. !'vfo!onyl CnA is prnduced fnllowlng D. RNA polymerase is an enzyme that tra:1scribes syrn1v:th,,tic po:,;tgangllunic fibers innerval-
carhoxylalinn of CoA in the DNA into Rl\lA chains. Synthesis is similar to heart nor0p!nephrilw.
p:-ncess of fatty Ei\D and are DN \ reproduct!on since sv11theslzed ➔ 3~,
involvect acid oxidation but NADPH B. A probe is single strand
protein the source oi agents in the synthfsls of \Vith a ra<lioisotope that is crnnplimenlary to a
and inositol trispho{'ph~tte fatty acids. segment of DNA of interest.
release oi caklum from intracellular stores. 57. B. This complex cnn be analyzed ""'''"c'cjUC>" Ii, B.
D. Vasopressin, atso known as ADH (antldiuretic technique known as a
hormone), is a peptide secreted from the poste- !)hate. This irreversible reactlon by 66. A. DNA ls a double stranded moleculi;,,
to an increase in sernm ATP and citrate (indicators of energy abundance base to and
nHwemf'nt
osmnlarity. hormones !Jsted are syn• within the The reaction is stlmuiated by guanlne to respon:sibh 0

the.sized and secreted from AMP. amounts o respiration. The cerebral cortex
49. A, Estrogen is steroid hormone and C. Pyridoxal phosphate is the .:u:tive cot:nzyme of RNA or mf performs of perceptinn, cognition.
does not require a mernhrane pyridoxine, which is Involved in a numher of will higher motor iunctions, memory. and emotion.
second messenger cAMP. hormont:'.s essential for normal amino acid D, Th of 78. B. tn the of muscle ton traction, ATP bouud
directly enter the cell and stimulate intracellular Biotln is for carboxylation sugars ( deoxyrlhose or ribose) joined
receptors. Peptide hormones and those derived Tetrahv,lrofol,,to. the coenzyme form 68. B. Transl1:1tlon the conversion of information
from amino acids are not readily lipid-sol~ of fol!c acid, is required for the synthesis of from mRJ'\A into protein. Transduction Is the
uble 1uwnm•crn;uc of membranes) and thus n.udelc adds and lncorporation of gem?tic information carried by are :nvo1vc-d the exdt::ltion-con-
requ! and a second mes~ a virus into bacteria. tral'tion coupling process but do not a
seng etc.), of the 69. A. Nlnety-five percent of enamel is inorganic min- source of energy.
effec enirnent11·ine no1reninenhrine and sources eral with 84% in calculus, 70'}{, in 79, A. Stretching of muscle spindles results in reflex
all o dentin, 60'¼ in bone, that intended to adjust the muscle to its rest-
cAMP. Upolysisoxidation of free fatty 70. C. tRNA is not for the hydroxylntlon of by contracting the rnuscle in vvhich it
50. C. Phosr,hocliesterases are intracellular enzymes that adds are also unnecessary during this time. collagen since process occurs after transla- is The strPtehing of the
cyclic nucleotides, thus the Glucagon release is inhibited due to elevati;:d tion o! rnRNA and synthesls of affererit impulses to the spifml cord r,nd
of cA1\1P to a metabolite, 5" glucose. Gly\oiysis Is increased due to elevated Ascorbic acid is requfred to a monosynaptic reflf'x stimulates muscle con-
second rnessenge!'" properties of cAMP. imraceilular glucose. molecule by uyun>Aymuue permit traction via an alpha motor neuron.
Adenylate cydase is an that catalyzes the 60, A. In the Irreversible oxidative decarboxylation cross~llnking among 8{l A. Muscles are innervated by two types of motor
conversion ot ATP to Monoarnine oxldase of pyruvate to acetyl CoA by pyruvate 71. C. Substitution of most ions of neurons (alpha and gamma)" Tlw alpha motor
(MAO) is an enz_y111e !ocated in presynaptic nerve clelw,clrc,eena,,e. the five coenzymes llsterl are fluoride) will increase the solubllity of fibers
terminats that deb,rr;vle$ dopamine, norepinephr!ne, ta! structure. The ratio of calcium and gamma motor
and epinephrine to inactive substances. Aspirin 6] B. hy1:lroxy;ipatite is 1.67: L neurons innervate the n1usde spindles (intra-
inhibits prostaglandin synthesis inhlhiting the 72. B. fusal musck: fibers). The muscles oi the lrls me
found in cartilaginous structures" controlled
51. E. cn,ira,ctor,z,w as having tensile strength
hyperglycemia would result due to increased and found in skin. tendon, and dentin.
gliirCOne,,gcne;;;s in the liver. Glycogen break~ Type lli collagen is characterlstkally more dls-
connective tissue down be enhanced and glycogen synthe- tensib!e and is found in large biood vessels. system and regulatory in tile
such as cornea, chondro!tln sulfate in cartilage, sis decreased, Since insulin pmmotE's amino IV found ln basement membranes, Gl tracL
and tendons and dermatan sulfate in add uptake in the liver, lack of insulin would 73. C. acids are transported across the lumlnal 81, B. Nitric oxide a molecule produced in
result in decreased uptake. surface of the intestlne Na1 amino acid co- endothelia! cells th.:1t acls clin~ctly on smooth
62. A. Following a meal, chylomkrons are synthesized transr,oners in the nwmhrnne. This is
in the intestine to to the '.Iver. ~n- gradient established in the tlon.
would
53. B, lnhibi
human and therefore we require cell memt,rnr1es reduce blood presscre- since aJ1giote1~sin H
in the from plant or bacterial sources. molecule for storage and transport of stimulates aldoster resulting in
All of the othE'r amlno acids can be synthesized Phnspho!iplds are abundant they ;;ng10Hc1isi11 ll is also
humans from other cnmpou11f!s and there- is not elevated under concHtlons bilayer structure characterlstk of membr;uws.
are not classified as "essential.., t'levated ener1:,,,Y sources in the post-absorptive Proteins are present and serve various functions
5,L A. Aspartame is a derivative composed of state. as receptors, and trnnscrnters'rnrri• would blood pressure,
add phenylc1lanlnr,., 63, A. lnsulln !s a hormone that is secreted under con- cno1este1co1 stabilizes the the baroreceptnr would produce reflex that
5S. B. degrade purine nucleotides to uric acid. dltions of Increased sources of energy (glucose)' for the would reduce cardi~H' output and periplwrni
Xanthine is an intermPrllate in a series of reac- and thecforc results in increased uptake and spnmgn11p1,1s are m1nrnments
tions, hut the fln.:11 product in hurnans is uric storage the form of glycogen, In th~ insulates membranes neurons. 82. B.
acid which is excreted in the urine. is the process, glucose decreased. The trans- C. Monoamlne oxidase is an enzyme located in the
degradation product of cnmpotmds containing port of glucose into the brain is not affected by presynaptic nerve terminal, which degrades extrnc•,•Jl,.,10, fiuld: the other ions arc in gre,.,itt'r
amlno groups (amino acids). the hormone. dopamine, norepinephrine, .:1nd eplnephrine to concc-ntratlons in t:xtrace!!ular fluid corn\J:trvd
inactive substances. with values withln tile cell,
Answer Key Biod,ernfotry and Physiobgy Y 36 1

83. A. All the other factors will prodt:ce vasodilatlon tlnn pressure, Reduced plasma protein concen- and target tissue. Paracrine refers to the target !ol!k,:hlr phase. During menstruation there un:
and hyperemia to increase the exchange oi tration decreases the oncotic pressure ln the tissu~ as a cell in close proximity to the secret~ sharp declines in ~strogcn an<! progcsternnt'
metabolites in the tissue. plasma which would increase llltration. ing tissue. Aotocrlne stimulation refers to cell due to reduced secretion of LH a~1d FSiL
84, D. C~pillary bluod flmv is propNtiona! tD changes Vasodilation of the afferent arterioles would 100. C. Glucagon is secreted from tlw pancreas m
that release!'!: a fat.tor (hormone") whirh stimu~
m the radius (r;-) of the vessel. Decreasing the increase capillary -pressure .- resu-Hing •---In~
!ates the cell frorn which it was released. respons0 to rNluccd plasma glucose due to
radius would produce increa:;ed vascular pres- incre_ased-- fHtratior. .. lnulin, although used to faslint.l and exerdse. !t functions to mobilize
Endocrine tactors are hormones that have an
sure. Viscosity of the blood depends primarily mea:;uri? g!omcru!ar filtration, does not affect .
effect at a distal site in the body. energ}-' stores (glucose) and return hloud glu~
on the hematocrit (percentage of red blood glo01erular fjltration .ra~~:_ .. :·· ----. cose to r.nrmal level!.. Glycogen iynthesis
9ft. B. Proge5terone ctJncentration in blood is highest
cells) and would not be ;::hanged by changes ln 92. C. Under normal conditions, an :il~C~;s·e-,s--rca1)M rollmv!ng the surge or LH (leuteinizing hormone) (glycogene5is) will not occur in respon,e to
the radius of the vessels. sorbed by both secondary active tnmsport sysM glucagon.
fofinwing ovulation, Ouring ovulation, estrogen
85. D. Under conditions of increased intracrnnia! pre~- tems and racilitated diffusion utilizing GLUT 1 surges due to pos:tive feedback during the
sure, lhe vasomotor regions of the medulla are and 2 carriers. Ca)dum is passlve!y reabsorbed,
stlmu!ated due to localized ischemia resulting ln hut its reabsorptlon is aiso reguiated by
an increase in systemic blood pres.sure. parathyroid hormone in the distal tubule, Not al!
Ventricular fibri!lation is irregular. rapid, unco- urea is reahsorbed but ahout halt is excreted. Its
ordinated contractions of the ventricle that do permeabllity ls altered by c:hanges in vnsoM
not resuH in effecting blood movement. preMin. Phosphate reabsorptino is also regu-
Anaphylactic shock is caused by severe allergic late{! by p~r-l'lthyrnid hormone
reactions, widespread release of hista;:nlne, and 93. C. The movement of the tongue against the palate
subsequent vasodHatation. is the only voluntary process among the four
86 A. Surf;H~tant is synthesized by alveolar cells and pos~ib!e answers,
functions to r~duce the surfac,: tension of the 94. A. The gastrointestinal tract is regulated by both
alveoli. This reduction of surface tension the sympathetic and parasympathetic brnncl.es
increases pulmonary compllance and decreases of the autonomic nervous system. The parasym-
the work oi breathing, lt wili also decrease the pathetic system is stimulatory and the sympa~
tendency of the lungs to collapse. Lack of sur- thetic system inhibitory. The CNS does influence
factant results in neonatal respiratory distress motility and secretory activity through the auto-
~yndrome. nomic nervous system. Numerous endocrine
87. C. Residual volume is the volume of air remaintng factors (secretin, gastrin, chole-cystokinin, etc,)
after a maximal forced expiration. Tidal volume are also involved with the regulation of gastroin-
ls the amount of air exchanged (expiration and testinal activity.
inspiration) during normal quiet breathing. 95, C. The salivary glands are under the influence of
Functional r~sidual capacity is a combination of both the sympathetic: and parasympathetic
the expired reserve volume (forced expiration) nervous sy·-stem. Parasympathetic stimulation
pins the residual volume. Vital capacity is the results in a more watery secretion compared
volume of alr expired after maxim-al inspiration. wjth sympathetic stimulatlon, which produces
88. B. The regulatory centers for respiration are in the incr~ased amounts of proteln with reduced vol-
brain stem-specifically. the medulla, The urne. The vagus is parasympathetic. which in
medulla contains three groups of neurons gener?il is stimulatory to gastrointestinal tract.
(medullary respiratory center, apneustlc center In general. sympathetics are inhibitory to the
and pneumotaxlc center). The cerebral cortex gastrointestinal trncL (The exceptions are sail-
may influence the medullary centers but does vary gla:-ids.)
not totally control respiration. 9G. C. Fat in the intestine, low pH, and increased osmo-
89. A. Atdosterone secretion is mediated through the larity of intestinal contents will reduce ga5trk
renin-~ngioten5in system. Anglotensln H stJmu- emptying. This reflex is mediated thro,1gh neu-
lates the ~ynthesis and release of aldosterone ronal and endocrlne factors comprising the
from the adrenal ~land. lt acts on the distal enterogastric reflex. Salme in the intestine will
tubule and collecting ducts to increase sodium not affect gastric emptying.
reabsorption and potassium S<"CTf'tion, 97, B. Thyroxine an<! triiodnthyroninf' are formed by the
90, A. Net filtration depen<l,i: on the hydrostatic pre$- cleavage of thyroglobulin following stlmulatinn
sures in the glomerular capillary, the hydro~ by TSH. Most thyroxine is converted to tri-
static pressures in bowman's space, and the iodnthyronine in liver ,;1nd kidney. Thyroglobulin
oncotic pressures in the capillary and bowman's i.s the storage form of thyroid hormone. TSH (thy~
capsule. Reducing the plasmn protein (reducing roid-stimulat!ng hormone) is prnrltKed in the
the oncotic rressure) will lower the tendency to anterlor pituitary. a is not a hormone secreted
retain fluid in the capillary, Thls would result in from the thyroid hut acts to stimulate the synthe~
a net increase in filtration. sis and secretion of thyroid hormones.
91. B. Increasing the hydrostatic pressure within the 98. A. The terminology U6ed is a classical descriptor of
glom~rulus woutcl decrease the net uitrafiltra- the relationship between the hormone source
l. B. Neutmphils are quick to arrive to the ~ite of group A. !)-hemolytic ~treptococ('a! infections,
infection or injury and are therefore t\ssociated such a~ S. pyogcnc•;.
with acute inflammiltion. All of the other cells 10. A. Tile only disease llsted that is related to an
listed afe mediators of chronic lnflammzition abnorm.ul number of chrornn~omes is
2. C. Dust cells. along with he.art fail cells, are Klineklter's syndrome, This disease is charw::-
macrophages that are found in the lungs, They terlzed by t\vo or mnre X chromosomes and one
are part of the reticuloendothelial system. or more Y thromosnme. TypkaUy, those aHected
3, E. Bacteria that are phhgocytosed by macrophages have 47 chromo.$Otnes witll a karyotype of XXY.
are kept in m~mhrane~bound vacuoles called l l. C. Po!':mphigus is an m1tolmmunc (lisease wherein
p!wgosomes, A phagosome will fuse with a iyso- autnantihodics against -epidermi~ cells are pro~
some, which contains many degradatlve enzymes. duced. Ht:o;tologk:aily, a. phennnwnnf' called acon-
including lysozyme. tholysis, wherein epidermal cells ilfJl)E"ar lo detach
4. C. Although lgE is the least prevalent immunoglobu- and separate from each other. is observed
lin in the body. lt is the major antibody mediAtor 12. E. Chronic brnnf'hitls prerlispnses those r(ffected
of type I hypersensitive reactions. These reac- to squ.:unuus neop!asia of the bronchi;:il epithe-
tions are caused primarily by prior sensitization llum (Le., hronchngenlc carcinoma)
and the acr:umulation of lg£ to a specific allergen. B. A, Chmn!c gram1iomatosis results from neutropbils
S. B, DlGeorge·s !'lyndromc is characterized by a defl~ with D defective NAPDH oxidase system, This
ciency of T cells. It is caused by the failu1·e of the afft·cts their ability to kill mlcrnorg~rnisms, since
third and fourth brachia! pouches to develop they an: 1.mahli~ to produce siqwmxide radical!>
normalfy, resulting in a lack of thymus -,nd 14. B. Dys plastic cells are ahnnrm;il in appearance and
parathyroid development. organization. The potential to develop lnto a
6. C. lntraoral Streptococcu.'f ;Jiridans is the most com- nrnllgnant tumor l~ pres{"llt; hmn'ver, tll!s risk
mon cause of subacute e-ndncarditis. Staphy- varit>s.
lococcus uureus is the most common cause oi 15. 0. HiV's enveiopt' contains two g!ycoprntelns,
acute endocardltfs. gpl~O and gp4L gp120 binds specifically with
7. D, Aschoff bodles are the classic lesions observed CD4 surf;;.(·e receptors,
in rheumatlc fever. They are areas of foc:al necro- lfi. A. The most common cause o! gastroenteritis in
si~ surrounded by a dense inflammatory infiltra~ children i~ tht• rnt,wirn,. lt is found in the
lion and may be observed in heart tissues. reovinis Family.
8. A. Fndotoxln or lipopolysaccharide is found in the R Radlogrnph!c flndingi;. of a cotton-wool appear-
cell walls of gram-nt-gative bacteria, ;111,e ls common in p,1:iell!H dic1gno~ed with
9. D. Endocarditis 1S most corr:monly caused by rniteltis clefonrnui:> or Pag~~i':-. disease.
group B, ct-hemolytic streptococcl, such as S. 18. B. lwtodavcs ftrncthm by denaturing proteins.
oirirfons, S. matans, S. sanguis, and S..m!icarius. They c.rc eifective agalnst spores.
All of the other conditlor.s ztre associated wlth 19. B. Ehiers-Dmilos syndro1m:s are dwracterized hy
defects in collagen (i.e., cnrmective tis:-ues)

363
Answer Key Micr-obiology and P11thology 'f' 365

20, R increased levels of alkaline ph(Jsphatase n:ay throat), lt is characterized by al! of the symp- tion factors. HemophHia A is cau.sed by a clefi- G3. D. flBeAg is present when lhere is active vim! repli·
be observed in diseases that di6pl~y extensive toms listed except po!yuria. A decrease in urlna- clency or factor 8. while hf'mophilia Bis caused caUt,n and the carrier is. highly mff'ctious.
bone loss. Osteoporosis is characterized by a Uon is usually observed. bv a deficiencv of factor 9. 64. C Opportuni~ric infrx:tions are seen in p<'llients wlth
decrease in bone mass. There ar~ either nor~ 35. B. O,;te-omyetitis, or an infection of bone, i5 most 50. C. G~mmas are g~anulomas that may be seen inter- AIDS. AH of the following microhE's re-prt'sent
ma! or decreased levels of alkaline phos- commonly caused by Staphylococcus aureus. tiary syphilis. Since syphills is caused by opportunistic organ is mt except coxsackievlrus.
phatase ln patients diagnosed with this Symptoms indude pain and systemic signs of Treponem,o pa!lidum, the correct answer is C. 65. L Rl1r11m~1toid arthritis is characterized by inflam-
disea:se. infection (i.e., fever, malaise). Granulnmas may also be seen in tuberculosis, an mation of the synovla\ membrane. Granulatinn
2L C The leading cause of death for diabetic patients 36, E. All of the features listed are commonly observed infection caused by Mycobacterium tuberctJiosis: tissue will form in the :i;ynovium and expand
is cardiovascular disease. Other complicalion5 in malignant cells, except a low nuclear- cytoplas~ however. in this disease, they are known as over the 1:1rtkuiar cartilage. This causes the
include retinopathy, nephropathy, and perlph- mic ratio. Due to the large nuclei present, there i! tuhercles. destruction of the underlying cartilage and
eral neuropathy. usually a high nucicar<ytoplasrnic ratio. 51. A, CD8 lymphocytes recognize class l MHC mole- results !n flbrotlc changes or ankyln5i5,
22. A Neuramlnldase is found on the surface envelope 37. E, Carcinoma in situ. Although the cells have not cules on antigen-pre~~nting cells. CD4 lympho- 66. A. The most common form of bre,1s.t cancer ls ade-
of influenza viruses. lt functions to attach the yet lnvaded the basement membrane, the likeli- cytes recognize cla.,;5 U MHC mnlecules on nocarcinoma arising from the duct;:il epithelium.
virus to the host ce]L hood of invasive growth is presumed to be high. antigen.presenting cells, -67, £. Both tetracycline and streptomycin, an arnlno-
23, D. Actinic keratosis f}roduces dry, scaly plaques 38. D. The Western blot tests !or HIV protem•. The 52. E. IL-5 is released by helper T cells to stirnula:e B glycoside. inhibit proteir: synthesis in bacteria.
with an erythematous base. They may be pre- ELISA tests for HIV antibodies. When these two lymphocytes to differentiate into plasma cells, It Streptomycin, however. is hacterictdnl, not ba<>
maHgnant Both compound and intraepldermal tests are used together, a 99'¼ accuracy rate is also activates eosinophils and increases the teriostatk.
nevi are benign. As to nevi, only junctional nevi achieved. production of fgA. 68. D, Rickets ls a vitamin D deficiency seen ln Infants
are considered to be premaJlgnant. 39. C. Since 90'X, of hlood transfusion-related hepatitis 53. B. Tuberculin reaction (Le. PPD skin te,t). is an and children. Oral findings indude a delayed
24, A. Hepatitis A is spread via oral-anal/fecal contam~ cases are caused by hepatitis C, and the patient example of a type IV delayed (cell-mediated) eruption 0f teeth mKl abnormal dentin.
inatlon. The other hepatitis viruses can he trans- has a history of having had a blood tran~fusion, hypersensitivity. Type rv reactions are the only 69. D. 12 hours is requir~ci.
mitted through blood, sexual contact, and/or the answer is C. type of hyp~uengitivity immune reartions that ?n. C. Cretinism is hypothyroidism ln children. Oral
perinatally. 40. D. Opportuni~tic infectlnns are a serious problem are nDt mediated by antibodie~. They are mainly findings include macroglossla, prolonged reten-
25. E. Cardiac tamponade is a serious condition with patients infected with HIV, Th.is patient's mediated by T cells. tion of primary teeth, and delayed eruption of
caused by accumulation of fluid in the peri- intraoral thrush is such an infection. which is 54. D. Mast cells contain dense granules with inflam~ permanent teeth.
cardium, This can result in impaired ventricular caused by Candida a!bicans. Treatment for can- matory mediators, inrludlng histamine and SRS- 71. A. In general, plaque b first colonized by grnm-pos~
filUng and can rapidly lead to decrea~ed cardiac dld!asis Includes oystatin. A, which is a leukotriene. itive, cocci bact-e-ria, such af: Streptococci, They
output and dealh. 41. C. The H!V is an RNA virus thnt contains three 55. C. Symptom~ of a myocardial infarction lndude are followed by gram-negative, rod-shaped
26. C. Pyelonephriti1 ls usually caused by gram-n.ega- enzymes: reverse tnmscripta!'Je, integra1e, and chest paln, shortness of breath, diaphoresis anaerobes, like Bocteroides, Frtsoba(·terium,
tive, rod-shaped bacteria that are part of the protease, Neuramlnldase ls a protein found on (sweating), clammy hands, nausea, and vomit- Porphymrrwnas, Prcuote!!a, and then filament•
normal flora of the enterk tract. lt is most com- the surface of the influenza virns. inir:. type h3cteria. such M Actitwmyces.
monly caused by Escherichia coli, fot!nwed by 42. C. Although all of the microbes Hsted cause pneu- 56. B. A quelling reaction occurs in a laboratory when 72. A, Peutz-Jeghers syndromP is n hereditary disease
Proteus, K!efuiella, and £ntemhacter. monia, the most common cause oi pneumonia in the polysaccharide c:npsule of a bacterium swells that ls transmitted by <mtosomal domlnr1nce.
27. D. Polycystic kidney disease is characterized by patients wlth AIDS is P. jiroveci ( carinii). after being treated with antisen1m or antjbodies. 73. C. Although all of the structures listf'd are commnn
the formlltion of cysts in kldney. U is commonly 43. A. McCune-Albright's syndrome is a type of fibrous 57. E. DipicoUnic acid i6 only found in spores. Since sites for oral cancer, the lower vermllion border
associated with berry aneurysms. dysplasla that presents with a triad of sympM C1ostridium is the only ~pore-forming mkroor- of the lip is the most common oral site.
28. D. Squamous cell carcinoma is the most common toms indudlng polyo5totic fibrous dy5plasia, ganJ5m listed, the correct answer is E. 74. C, Auer rods lire observed in blast cells character-
oral cancer, occurring in 90'.Y, of all cases, it is a cate au lait spot~, and endocrine abnormalities. 58. D. Dextran:s consist of glucose molecules linked istic of acute myelogenous leukemia.
cancer of squamous epithelium, speclfical!y a 44, B. Fibrous dysplasia is caused by replacement of together. Th~y not only act as the structural 75. A. Complications of Barrett's esophagus include:
tumor of keratinocvtes. normal bone with an irret,rular bone containing component of pJaque, but they also contribute t1denocarcinoma, stricture formation, or hemor-
29. E. Both Epidermoph}·um and Trichophyton can fibrous connective tissue. This gives the radi- to the retention of lactic add near the tooth. rhage (ulceration).
cause tinca pedis. ographs a characteristic ground glass appear- Fructans or levans are also found in plaque; 76. A. M-protein is an import,mt virulent factor found
30. A. Steno.sis describes fibrotic and thickened ance. however. they contain fructose. only in the spedes Stn.•ptococrn5 pyog('nes.
valves, resulting in reduced blood flow through 4S. B. Fibrous dysplasia is caused by replacement of 59, C. The most common r;;use of bacterial menln_gltis 77. D. An infection by molds, such as mucormycosis.
the valve. normal bone with an irregulnr bone containing in newborns ls Escherichia coli. Haernophilus results in thP lnvaslon oi blood vessel cndotlw-
31. c. Cyilk fibrosis results from an ahnormal chlo- fibrous cnrmective tissue. Therefore, the pathol- influen.:.:.ue and Neisseria mcningitide,1 are the limn, Thls leads to hemorrh;igic iniarction ,md
ride channel that affects all exocrine glands, ogy report usuc.lly de-scribes abnorn1ally shapt!!d most common cause of bacterial men!ngit!s in Ussue necrosis,
32. B Trisomy 21, which is also known as Down .~yn• trabecula.e in loos~ly arran~ed fibrous tissue. lnfants/children or young adults, tespective:ly. 78. 0. Chronic lymphocyti-:· l0,ukemia ls charnctel'ized
drome, is a disorder affecting autnsomes. It is L16, D. The panial thromhopla~tin time (PTT) test 60. D. Teichoic acid may be found only in the cell bv Uw proliferation of B cells. It is both the most
generally caused hy meiotic nondisjunction in measures the intrinsic and common pathways of walls of gram-positive bacteria and contain c;Hnmnn and least m,1:!ignant type of leukc-mi<1.
the mother. which results in an extra copy of the co<11'ulation cascade. A prolonged PTT could antigenic properties. All of the other answers 79. B. Consequences of asbestosis include mesothe-
chromosome 21 (trisomy 21). result from a dcfidency of factor V, VIII, IX, X, XI, IJ:"'lted are found in the cell walls of gram-nega- lioma an<! bronchngcnic carcinoma.
33. D. These oral findin1:s can be observed ln patients or Xfl or of prothrombin or fibrinogen. tive bacteria, 80 D. J<1rn1flke Ls characterized by yellowness of tls~
with hyperthyroidism. The oniy endocrlne dis- 47 C Hemophilia A is caused by a de.fidency of factor 6 L B. The tetanus vaccine consists of tetanus to:xoid. sues, including skin, eyes. and mucous mem-
order listed that ls relnted to hyperthyrolcHsm is 8 (<t0tihemo11hllic factor). It is part of the DPT vaccine and should he glven branes. I! is caused by diseases that increase
Ptummer·s disease. It is caused hy a nodular 48. C. Hemophilia A is a hereditary disorder thal Is X- about every 10 years. serum conjugated and/or 111\tonjug<.HNl billru~
growth or adenoma of the thyroid. Hnked; thus, lt only affects mal-es. However, 62. A, lgA is found in mucosa! secretions of the geni- bin. These conditions include liver {liM.ase, such
34. D, Poststreptococcal glomerulonephritls is a clas- females can be carriers. tourinary, intestinal, and respiratory tracts, as cirrhosis or hepatitis; he-mnlyl.lc anemias:
sic example of nephritic syndrome, which 49. B. ~!lie only way to distinguish hemophilia A from including tears, saliva and colostrums. obstruction of the common bile duct, as caused
occurs after a Streptococcus infection ( e.g., strep hemophilia B is to assay the levels of c.oagula~
by gailstones or cholelithlasis; and carcinoma 91. E. Treatment for tuberculosis includes a multi-
involving the head of the pan..:reas. drug therapy. Rifampin, isoniazid, and ethambu-
81. D. Asthma is an obstructive lung disease caused by tol are three of the first~line drugs used.
narrowing: of the airways. Common symptoms 92. E. Ghon complex describes the calcified scar that
indude dy!pne-a, wheezing on expiration, and a remains following the primary infection. It is
dry cough. usually found in the lung and includes the pri~
82. C. Sy~temic lupus ervthemato~us, also known as mary lesion and its regional lyinph node.
lupus, is com~ider'ed an autoimmune disease, 93, A. Given the patient's history of rheumatic fever,
althou2h the exact cause is unknown. It is char~ the heart murmur is most likely from a dysfunc~
acterized by the presence of antinuclear anti~ tioning mitrai valve. Rheumatic fever most com~
bodies (ANA). Common ANA findings mdi;de monly affects the mitral valve, resulting in
anti-D1~A. anti-RNA, and anti~Sm antigen. mitral vaive stenosis, regurgitation, or both.
83. A. Due to the excessive l:evels of serum calcium, 94. B. Rheumatic fever ts usually preceded by a group
osseous changes, such as metastatic calcific:a~ A streptococcus respiratory infection (e.g.,
tions and kidney stones, will occur in patlents strep throat or pharyngitis).
with hyperparathyroidiinn. 95. A. The most common cause of subacute endocardi-
84. C. Osteopetrosls. also known as Alben-SchOnberg tis is Streptococcus viddans. S. viridans is a a-
disease or marble bone disease, ls caused by hemolytic streptococci, representing incomplete
abnormal osteoclasts, The lack of bone resorp- lysis of red blood cells in laboratory
tion results in defective bone remodeling and culture-,.
increased bone density, which may invade into 96. E. During subacute endocarditis, vegetations, or
the bone marrow space. thrombi, form on previously damaged heart L C. Universal tooth numbering system for primary elongated appearance; C, Crowns are a milk-y
85. A. Whooping cough is caused by Borderella pertus~ valves. Complication! can arise if the thrombus dentition. A though T. white, not translucent white; D, There is no root
sis. All of the factors listed contribute to lts viru- embolizes. (i.e. when a fragment separates and 2. A. Said to be the most atypical of human molars. trunk.
lence except lgA protease. This enzyme is enters the circulation), causing septic Infarcts. 3. C. A, Smaller in all dimensions; B. AH roots seen from 11. C. Chronologies from textbooks vary so that a~e of
produced by Haemophilus influenzae, another Other complications include valvular dysfunc- buccal aspect; D, roots nearly the same length. eruption for the primary m.Andibular central ind~
gram-neiative, rod-shaped bacteria. tion or abscess Jormatlon. 4. D. Mesia! aspect: A, Pronounced convexity present; !Or may be given l'l:5 6 months (Woelfel and Scheid,
86. E. Ce:liac sprue ls characterized by malabsorptlon 97. C. Aminoglycosides block the 30S ribosomal sub, B. Curvature ln an occlusal direction; C, 2002); 6.5 months (Kraus et al., 1969); 7.5 months
and mucosal lesions of the small intestines that unit to lnhiblt protein synthesis. Cervical third greater occlusal third; D, (Charlick et al.); or 8 {6 to 10) months (Ash and
is caused by an allergy to gluten. A complication 98, C. Burkitt's lymphoma is an aggressive lymphoma Mestolingual cusp longer and sharper than Nelson. 2003).
of this disease incJudes T-cell lymphomas. that affects B lymphocytes. mesiobuccal cusp. 12. C. The most favorable !'lequence for prevention of
87, B. Tuberculosis ls caused by the bacteria 99. E. Hlstologic evaluation of Burkitt's lymphoma 5. D. Lingual aspect: D, Well-Oefined developmental malocclusion.
fl.1ycobacterium tuberculosis. These bacteria can- reveals a characteristic "starry-sky" appear- groov~ separating ML and DL cusps. A, ML cusp 13. B. Textbook chronologies indicate that the maxillary
not be Gram stained due to their waxy cell walls. ance, which results from the lighter-colored large and well-developed; B, DL cusp well-devel- primary second molar is most commonly tl1e last
Therefore, to Identify these bacteria. ll:11 acid-fast macrophages present. oped, more so than that of the primary first to empt: 24 months (Woellel and Scheid, 2002): 20
stain must be ordered. 100. A. Epstein-Barr virus is responsible for causing molar; C, There is a supplemental cusp apical to to 30 months (Kraus et al., 1969); 25 to 33 months
88. D. iii. tuberculosis infects macrophages, which are mononucleosis, a disease that also affects B the ML cusp, but poorly developed. (Ash and Nelson. 200'l).
initially unable to kill the phagocytosed bacteria. lymphocytes. 6. D. A developmental depre5~ion but not a groove 14. C. The length of the prlmary maxiilary central inci-
89. A. Cord factor is a glycolipld found on the surface divides the two buccal cusps. A, It does not sor ls I 6 mm (Black -GV, cit(",d by Ash and Nelson,
of M. Wberculosis, resemble other primary teeth; B, 1t has a rhom- 2003); 16 mm (Kraus et al., 1969) or 17.2 mm
90, 0. Type IV delayed type hypersensitivity reactions boidal outline from the occlusal a,;;pect: C, The (Woelfel and Sc.:heicL 2002). The permanent max~
are the only type of hypersensitivity immune mesial buccal cu:i,;p is larger. illary incisor is 23.6 mm ( A_.,_h and Nelson, 2003):
reactions that are not media1ed bv antibodles, 7. A. The MB. DB. and distal (D) cusps are almost 23.5 mm (Kraus et al., 1969); 2:16 mm (Woelfel
They are mainly mediated by T cells. equal in size, whereas the D cusp of the perma- and Scheid. 2002). ! 6i23.5 • 0.68 or 68~,; 17 .2/23.6
nent molar ls smaller. B, The primary molar 0. 73 or 73'¾,. The answer is about 70'/4',.
crown is narrower, not wider BL: C The primary l!i C. The correct answer for A would be about 1.5
molar outline is somewhat rectangular; D, The months or 6 weeks; B, About 4 lo 5 years: C, About
ratio of crown/root not the same, 25 months ot JO weeks; D, About 4 lo 5 ,Yf!ars.
8. B. Comparative1Y, roots of primary anterior teeth 16. A. (Ash and Nelson. 2003; Woeliel and Schied. 2()()2).
are narrower and longer. A. Crowns of anterior 17. C. No, or almost never does the mesia! marginal
primary teeth are wider, not narrower; C, Cervical development;il groove cross the mesial mar-
ridge& of prirnary anterior teeth more prominent; gina( ridge.
D, B, and L surt2!lces are more flat, not less flat, 18. B. M,: DB height equal• MR height; M,. Dll msp
9. B. The overall average length of the primary height !'!lightly less than MB cusp height; M::'.
mandibular central incisor is 14 mm (Black GV, DB cusp height much shorter Lhan MB cusp
cited by Ash and Nelson, 2003); lfi mm (Woelfel height.
and Scheid. 2002); 14 mm (Kraus et al.. 1969). 19. D. Marked a:n-mmetrv of mesia1 and distal halves.
The anwer is 14 to 16 mm, not 16 to 17 mm. 20, D. From MD- aspect, all posterior teeth have a
10. A. Greater MD di.:i.meter reiatlve to crown height trapeznlda! outline with shortest unevi;:;n side
than permanent teeth. B, Squat appearance, not toward the occlusa! surface. A, Present prima-

367
3 68 Y Answer Key Denial Anato;r.y i>r:d Ocdus!on Answer Key Deni<>! An<)t-omy and Ocdusio-n -. 369

rily on first premolar; B, E..xceptlons may be the 43. C. The mech::misrns of the two type!'!- of head.aches (B!ack GV, cited by Ash and J\:elson, 200'.1); 76.X 19 A. The :rnric:1liltF1~1pt)r;1! nf•rv~ provkles prlmary
DB root of the maxillary second molar; C, From are quite different A, 8. and D are considered nun (\Vo(:!fel ant! Scheid. ZOU2). innervation tu tti>:.' T!\1.L
mesial and distal a5pect, al! posterior mandlhu- true. 61. A. On average, the height of curvature is 3.5 m:n on 80. D, The DL cusp !~ sm:dles: 011 a five~lohecl
lar teeth, not maxJilary posterior reeth, have a 44. B. ~-amyloid deposits are a component of the nH:•sial and 2.S mm on the clista! of the max• n:andibular s0cond ;Jrt·rnoi{:.r.
rhombn1clal outline. Ah:heimer disease hut not of migraint" headaches. illarv central indsot; lt ls ::Ln mm on the rnfisla.l SL D. The primary m;rndihulor first mnlm typlr.:illy
2L C. Be!ow the apices, 6:)'X) of the time. 45, A. (Montgnrnery et aL, 1996 j and ·2.0 mm on the distal of the rn:ind'.bu!or cen- Jws four pulp ho:·ns.
22. C. Greater than 5'.f, (even up to 40';/,_) in populations 46. B. Infection is most likely to involve the submaxn. tral incisor. 8') D. Clink:-dly iu !'f•!dlon to intenKclusai space
with Mongollan trai::s. A, Does not exceed 4.2'1, lary space. con~id".::ring the cHnica! findings. 62. 0. The width and deplh of arch <1.re more or le.t55 83. C. (Ash and Nefaon, 200::n
in Caucasians: B, Less than 5'.K, in Eurasian and 47. A. Given the clinlcal findings and the Involved constant aHer 9 months of D:ge. However, a sub- 84. B. Root bHutc<i~ior. ls more Hkely ln the mandlhular
Asian populations; D, Less than 3\!;', in African molar position, !t is the lateral upper deep stantial increase in the anterior-posterior dis- camne.
populations. cervkal node group that would most likely be tance occurs for penmment molars. 8S. O. All postf>rlor tee-th viewed from dishil a-sped ha-ve
23. D. G~neral:y a jaw~closing muscle. May stabilize the first involved with lymph drainage. 63. C. These are the most common 5equenccs of a rhl)mhnklal outline
and act with lateral jaw movements. 48. B. Oral Tl3 most often involves the tongue eruptlon. 86. A. A distinct den,!opnwntal groove, prom\nt•nt
2 ◄. A. ln the earliest phase, the condyle moves forward 49. B. Langerhans' giant cells, epitheliod cells, and 64. A. First evidern:e of the maxillary central incisor buccal trinngular ridge. two ,:osps. and clisttnct
in cona~rt with the disc, not before. caseous necrotic areas are highly suggestive of cakifkation Is given as 14 (1:3 to 1G) weeks by nwslal and di~tal occ!usal pits an:~ rnost charac-
25. D. Occlusal contact relations do not interfere with TB; however, 5pedal stains for acid•fa:st hacilll Kraus and Jordon, 1965 and by Ash and Nelson, teristic of the mandibular first premnlm,
jaw opening, but TM.I and muscle disorders mav. (AFB) and isolation of M. tuberculosis may be 2003; this is givt:n as 4 months by \h/oelfei and 87. A. From the incisal aspect, the maxillary central
26. C. Mediotrusive contact, but may be called a no~- necesary, keeping in mind that staining may not Scheid, p, I 11. incisor has a greater MD dlameter (8.5 mm)
worki)lg (balancing) contact differentiate TB from other mycobnclerial infec- 65. B. (Kraus et aL, J 969) than FL diameter Cf.ll mn1), B. The MD c!iarneter
27. D. Temporomandibuiar (TMJ) (Ash and Nelson, tions. Other molecular tests may be indicated. 6G. A. The correct answer (7 to 8 years) is based on ls 7.5 mm and the FL diameter is 8J} rnm; C. The
2003); craniomandihufar (\Voelfel and Scheid, 50 D. There is no evidence of a connection between d1Tonologi~s in Ash and Nelson, :,;003 and MD ditrnwter is 7.0 mm ar:d the FL diameter is
2002). tnyositis and bruxing. Woelfel and Scheid, 2002, whlc-h are bast.. d on 7.5 rnm; D, The iV1D diametf•r is S,5 mm and the
28. A. Vertical component not seen on horizontal 51 C. The antico;lguiant effect of warfarin is inhibited orlginal chronologies by Schour am! Mnssler, FL diameter is 6.0 mm
traclng. by rifampin. A, Rifampin dPcreases the anlicoag~ modified for permanent dentition by Kronfekl &">. A. Maximum conrly!ar mtation takes place with rnax-
29. C. Height of contour on lingual surfaces of molars ulation eHect of warfarin; B, Rifamp!n interferes (Bur) and by Krnnield an<l Schorn· (1939) for lmum i-'W op~nlng.
and ptf:!molan is at the cervical or middle with cyclic conversion; D, Rifampin increuses the deciduou,; teeth, Also, with slight modific;-1tions. 89. R Use of tht> formula indicates on the basis of :r 6
third. metahohc clearance. by McCall and Schour (cited by Orban. 1931). months that the ADF is 2 to 3 years.
30. C. Transverse ridge connecting MB and ML cu~ps. 52. B. Combination name for jolning buccal and lingual 67. B. The permanent maxillary first premolar usually 90. C, A and Hare correct.Dis per tht' m,mn!rictur<'rls
31. B. Mandlbular first premolar is most likely to have cusp triangular ridges. has two roots but they are buccal and lingual, recomrncndatJc,ns on prndtKt, 2005.
a single pulp horn. 53. B, C, D. For A, the crown outline converges only lin- not mesial and distal. 91. B. Sys.temic etiologic factors for hypopl;1~i;1 are
32. A. Bifurcated roots are nol a normal feature of gually, B. The small transverse ridge has 1:>ieen fl8. D. (Ash and Nelson, 2003) thought to occur possibly after birth and belore
mandibular incisors. called an oblique ridge. 69. C. The labiollngunl diameter of crown near the G years nf age,
:33. D. Height of contour on distal ~urface of permanent 54. D. A. The mic•siolingual cusp is most prominent; B, cervix ii greater than in mandflmlar canine. 92. C. Me~1~lf'5 is not known to <:,rnse eiuune1 l1ypopfo.*
mandibular central incisors is at the incisal The disto!ingual cusp is poorly defined; C, The 7(L D. The labial, mesiofnctsal angle of tooth 8 and 9 sia.
third). distobuccal cusp may be seen because it is (central incisors) is a sharp, right angle, not 9:l D. lntcrmedlute filament is a smooth musde fila-
34, C. The dimension of the penmment maxillary longer. 5,IJghtly rounded. The labial, me~ioincisal angle ment. Conn-ectin is another nnme for titin.
canine at the widest \1D diameter is 75 mm r7.5 55, C. The central groove is well-defined. A, The pri- of tooth 7 and 1i1 (lateral lndsors) ls ~lightly Statenwnts rC'lativc to A, B, and Care true and
mm (Ash and Nelson, 2003); 7.6 mm [6.3 to '9.5 mary maxillary second molar does have a well~ rounded, not a sharp rlght angle. refer to sl::.eietaf muscle rnaments.
mm (Wo~llel and Scheid, 2002)]). defined mesial triangular fossa; B, The oblique 7L C. Distal to the second premolar, 56 1\;. 94. D. Masscter muscles prnvide mo$l of the for('.<.'
:15. B. DO development.al groove is not found on ridge is prom;nent; D, The supplementary cusp is 72. A. (McCauley. 1943; Ash and Nelson, 200:1) between molar teeth with clcnd,lng in the lnter-
mancl!holar second molars. not well-developed. 73, D. Internal aprmeuros-es do move and deform. cuspal po.,ition.
36-. D. Y-shaped central developmental groove is found S6, A. B, !t bas four well.developed cusps; C, lt has only 74. A. B, Tooth grinding in sleep brux!sm lasts up 95. D. lndivid11als who br;Jx during the day do not
on tht? mandibular second premol~r. one 5;upplementary cusp; D, lt has a well-devel- to 5 minutes., not 20 minutes; C. Sleep bru.xism nece5i-larlly brnx at night.
37, D. In lhe cusp-embrasure occlusal relationship. oped me5i21I triangular fnssa. often eoin..-:ldes with passage from deeper !)6. D. Sequential changes trom ;rntorwmi.::-
the maxmary first 1ucmolar is most likely to S7, A. 11, The outline of the crown converges dL•ually: to lighter sleep, not iighter to deep sleep: D. carcllac/hrain cortlcal acrivitics prPCt'lfr SB-
articulate wHh the first and second pr<?molars. C, The three buccal cusps are similar in slze; D, Sleep bruxism occurs approximately ev12ry 90 related jav,r motor activity.
38. C. Equal bilateral contraction of the lateral ptery- Welt-defined triangular ridges extend from the cusp minutPs, not every 2D minutes ln the sleep 97, D. Malocdw1ion has not heen s11ggest0d as caus~
goid muscles results in protrusion of the tip~. cycle. ing or aggrava! ing broxism.
mandible. 58. C. A, Enamel cap thfnner and more consistent in 75. A. Bruxism ls now thought to be mainly regulatt•d 98. D. Fluoro5i~ versus NF opi:icHies nwy he difficult
39. D. Molecul<lf tests like the polymerase chain reac- depth: B, Comparatively greater thickness; D, centrally, not peripherally to di;1gnose dln!Cally
tion have a fast, not slow, tumarou1HI time. Puip horns are higher in primary molars. 76 D. Ma11dibul<'lr suppOliing cusps are buccal cusps. 99. B. Thick :md thin filaments do not change in length.
40. A. RHampin causes orange, not green. urine, tears. 59. D. A, Enamel rods slope occfww!Jy. not gingivally. in 77. A. To measure horizontal overlap, measure from 100. C. SHLP helps stabi!!ze- the head ,and conclyl(i
and s,.veat the primary mo!ars: 8, Enamel rods at cervix !abl.i.l of mandibular central incisor to labial of again~a tile ,errincnce in denching; A, B. and Il
'1 L B. The drugs binds tightly only to prokaryntic Rl\A slope gingica!ly, not occlusa!ly, in permanent maxillary central incisor are trne.
polymera~e. D, RH1'mpin induces activity of molars; C, The buccal cervical ridge is more pro- 78. B. (Ash and Nelson, 2003)
hepatic mlxed oxidases. nounced.
42. C. Naproxen (naprosyn) has not been reported to Gtl B. The total of avt>ragc overall mesiodlstal din.me~
exacerbate bruxism, ter5' of primary maxillary crowns is 682 mm
L D. Mandibular supporting cusps ate buccal cusps. fasting and exercise. It fundi{lns to mohllizf'
2. A. The lnferlor head of the lateral pteryg:oid muscle energy stores (glucose) 0.nd return blood glucose
attaches to the lateral surface of the lateral to normal levets. Clycogen synthesis (_glycogeneM
ptery~oid plate of sphenoid bone. Its is.uperior sis) wm not ocetir in response to glucagon.
head attachc, to the infratemporal crest of the 10. D. Unlike in noncompNltive inhibition, the
greater wlng of sphenoid bone. The deep fibers inhlbltor cmnpc:"tett for the samf' site as the sub-
of the medial pterygoid muscle attaches to the strate. ll is thererore possible (with increased
medial surface of the lateral pterygoid platt-. amounts of sub::-trate) tn reach the V,..,,~·
3. B. Combln<:1tion name for jolning buccal and lingual Apparent Km is incrertsed !-Ince more :-ubstrate
cusp triangular ridges. is required to reach 1/2 V,,1.. x (the dcfo1ition of
4, B. Consequences of asbestosis include mesothe-- Kml.
lioma and bronchogenic carcinoma. i 1. D. The primary rn:indibu!m lirsl mola;" typically
5. C. Under normal conditions, all giucnse is reab- has four puip hurns.
sorbed by both !econrlary active tran5port ~ys~ 12. C. Burk:itt'i lymphoma is an aggrc3~ivc !ymphorn0
terns and facilitated diffusion utilizing GLUT 1 that affects B lymphocytes.
and 2 carrlers. Calcium ls pas5ively reabsorbed. 13. E. i-Hstologic r,,.alualion of Burkrtt's lymphoma
but its reabsorption i~ hh,o regulatect by parathy- reveals a characteristic "starry-~ky .. appear-
roid hormone !n thie distal tubule. Not all ure~ is ance, which results from the lighter-colored
reabsorbed but about half Is excreted. lts per~ rmwrophagt•s present.
me,1bllity ls altered by changes ln vasopre.ssin 14. A. Epste-in-Barr virus ls ret'-pons.i!)le for causing
Phosphate reabsorption is also regulated by monnnuc!eosis. a diseas(' that also affects B lym-
parathyroid hormone. phocytes.
6. C. Although lgE is the !east prevalent imrnunogloh- 15, A. To me;:1:nrrc horizontal overlap, mcastirc from
uHn in the body, it is the major antihody media- labial ol mandibular central indsur lu 1ahi.al of
tor of type l hypersensitive reactlnns. These maxii!ary cf"ntral lndsor (Ash and Nelson, :2003:
reactions are caused primarily by prior sensiti~ Woelfel and Scheid. 2002_;.
zation and the accumulation of lgE to a specific 16, D, Capillary blood flow is propor!iona! ln changes
allergen. in the radius (r-1) nf tht:> vessel. Oecn.•,Hing the
7. A, Since DNA ls a clouhle stranded motecu!e, cyto- radlus would produn:: incT('.i:tsed vascular pres-
.sioe wlil always be base paired to guanine and sure. VlscosUy of the blood depends prirn;i.rily
guanine to cytosine. Therefore there wlll be on the hernatocrit (p~rcentagc of red blood
~qua! amounts of these compounds. It !s uniikt:•ly cells) and would not be changed by changes in
that RNA or mRNA. being a single-stranded mol- the radius of the vessels.
ecule, will have equal ammmts of these bases. 17. D. The facial nerve supplies specia! sensory {taste;
8. B. The branchial arches disappear when the sec~ to the anterlor two thirds of the tongue, vla one
ond arch grows down to contact the fifth of its branches. the c-horda tympanL The chorda
branchial arch, tympani bnmche-s from the facial rn;rvl>, carry-
9. C. Glucag:on is secreted from the pancreas in ing both sensory fibers for taste and prt'g<m-
response to reduced plasma gluco.se due to glionk p,1n1sympathE'tic fibers. !t exit~ from ol

371
37 2 'f SampJe [xam Answer Key Sample Exam Answer Key • 37 3

the lemporal bone to join the lingual nerve (a coup!e<l to transport ions in both directions 38. B. The overall average length of the primary 51. A. \\'hen pulpa! 111".:rves ~tre stimu!att-d, they can
branch of CN V3). as it courses infcriorlv toward across the membrane. Both active transport and mandibular central incisor Is 14 mm (Black GV. only transmit one .sign<1!: paio
the suhmandibular ganglion {Tig~ 1-19), facilitated diffusion are carrier~mediated and cited by A.1-1'. and Nelson, 2003): J 6 mm (Woelfel s•) C. Th~ mylnhynid mu$clt:: forms the floor of the
Postganglionic paruympathetic fibers emerge therefore are inftuenced by competitive inhi- and Scheid, 2002); 14 mm (Kraus et aL, 1969). mouth. Rt>laxation of this r:rnscie would help th-e
from the ganglion and continue toward the sub- bition, The answer is 14 to 16 mm. not 16 to l 7 mm. denfo•t push !he i\!rn clown. to help ensure Urnt
lingual and submandibular glands (Fig. 1-20). 29. A. ln genera!, plaque ls first colonized by gram-pos- 39. A. Stenosis describes fibrotlc and thickened lhe apical ro0t ls captured on thr! radingraph.
Sensory fibers also branch from the nerve and itive, cocci bacteria, such as Streptococci. They valves, resulting ln reciuc~d blood flnw through 53, B. Fibers of the lateral pterygoid rnuscle are
provide taste sensation to the anterior two are foHowed by gram-negative, rod-shapt:d the valve. attached to the anterior end n-f the dlsc.
thirds of the tongue. anaerobes, like Bact~roides. Fusobacterium, 40. A. B, It has four well••developed cusp!; C, Tt has Contraction uf this muscle pulls the disc in an
IR C. The lahiolingual diamet~r of crown near the Porphymmonm, Preootefla, and then filament- only one supplementary cusp; D, It has a welf- anterior and medial direction.
cervlx is great~r than In mandibular canine type b.acteria, such as ,4.ctinom)1ces. deudoped mesia! triangular fossa. 54. B. The sem;ory distribution for lhe maxillary
(Kraus et ol., 1969). 30. D. Triacylglycerol is not commcmly a component of 4L A, All the other factors will produce vasodllatinn nerve (CN Y2) includes lhe cheek and upper
19, A. The gastrointestinal tract is regulated by both cell membranes since It serves primarily as a and hyperemia to increase the exchange of lip, and lower eyelid, upper lip, nasopharynx,
the sympathetic and parasympathetic branches molecule for storage and transport of fatty metabolites in the tissue. tonsils, palate, and maxillary teeth. The sen~
of the autonomic nervous system. The parasym~ acids. Phospholipicls are abundant since they 42. C. The inferior aspect of the diaphragm ls supplied sory distribution for the long buccal alS(J
pathetic system is stimulatory and the l'5yrnpa- form the bilayer structure characteristic of with blood by the inferior phrenic arteries. The includes the (lower) cheek; however. it does
thetic system inhibitory. The CNS does tnfluence membranes. Proteins are present and serve var- median sacral artery supplit>s the anterior not include the upper lip. The long bucca! i5 a
motility and secretory activity through the auto- ious functions as enzyme:5, receptors, and trans~ aspect of the sacral area, and the lumbar arter- branch of the mandibular nerve (C~ V3) and
nomic nervous system. Numerous endocrine porters/carriers. Cholesterol stabilizes the ies supply the lower abdominal walL The celiac provides sensory nerves to the cheek, buccal
!actors (secretin, gastrin, cholecystokinln, etc,) membrane and is responsible for the mainte- trunk: and superior mesenterk arteries are both gingiva of the pnsterior m;;ndihulM teeth, and
are also involved with the regulation of gastroin- nance of fluidity. Sphingolipids are components unpaired branches to the gut and associated buccal mucosa.
testinal activlty. of myelin, which insulates membranes in glands. 55, A. The inferior alveolar nerve (JAN) courses
20. C. Although all of the structures listed are common neurons. 43. C. Pyridoxa! phosphate is the active coenzymt" of heh\'een the ,,;phcnomandilrnfar ligament and
sites for oral cancer, the lower vermilion horder 31. A. Lateral cricoarytenold. The oblique and trans- pyri<loxine, whlch is involved in a number of the rarnus of the mandible before entering thl:''
of the lip is the most common oral site, verse arytenoid.5 and thyroarytenoid also enzyme systems essential for normal amino acid nrnncHbular foramen. The sphe:nomandibt!IM lig-
21. A. Vertie•! component not seen on horizonlel tracing. adduct the vocal folds. The posterior cricoary- metabolism. Biotin is re-qllired for carboxylation ament may therefore he dam<11,;ed during the
22. D. The palatopharynge:us forms the posterior ton- tenoids abducts the vocal cords. The cricothy, reactions. Tetrahydrofolate, the coenzyme form .'ldmlnlstration of an IAN block.
sillar pillar. It also functions to close of! the rold muscle raises the cricoid cartilage and of folic acid, ?s required for the :isynthesi~ of 56. D. Polycy~tic kidney disease is charncterized by
nasopharynx and larynx during swallowing. The tenses the vocai cords. nucleic adds and porphyrin. Niacin participates the formation of cysts in kidney, It ls cnmrnn-n!y
anterior tonsillar pillar is formed by the 32. E, Carcinoma in situ. Although the cells have not in eneq:,1y met.?lboli~m (oxidation-reductfon reac~ associated wlth berry aneurysms.
p:1letoglossus, yet lnvaded the basement membrane, the likeli- tions) 57. B. The nropharynx is lined by stratified sq11amous
23. D. Decrea6ing total peripheral resistance woul-cl hood of invasive growth is presumed to be high. 44. C. Equal bilateral contraction of the lateral ptery- epithelium. This type of e1)itbdium also lines the
produce greater reductions in blood pressure. 33, D. The rectos ahdominus muscle of the anterolat- gold muscles results In protrusion of the oral cavity, laryngophmynx, es0phngus, vagln.al
Due to the baror~ceptor reflex, cardiac heart eral abdominal wall ls described as being beltor mandible. canal and anal canal.
rate would increase, Increa~ed stroke volume strap-like, The remttining three muscles of the 45. D. Oligodendrocytes produce the myeiin sheath 58. C. Phosphodiesterase$ are intracellular enzymes
(due to increased venous return and sympa- anterolatern.l abdominal wall (external oblique around myelin~ted axons in the central nervous that degrade cyclic nudeotklt>s, thus catalyzing
thetic stimulation of cardiac mue;cle) would also ruusde. internal oblique muscle, and trans- system. Schw11.nn cells make up the myeHn the hy(lrolysis of cAMP to a metabolite. 5· AMP,
occur, Increased cardiac output would occur verses abdominus mu~cle) are all described as sheath around mye!inated axons in the parasym- whlch lack~ the ;<wcond messenger properties of
due to increased heart rate and stroke volume. sheet-like. The quadratus lumborum muscle is pathetic nervous system. cAl'vlP. Adenyintt:~ cyclase is an enzyme thnt cat~
24, 8. Insulin is a polypeptide hormone that promotes part of the posterior abdominal wall. 46. A. Ninety-five percent of enamel is inorganic mln- alyzes the com·ersion of ATP to cAMP.
energy storage when it is readily available. The 34. D. A, Enamel rods slope ocdusally, not gingivally, in enll compared with 8•1% in calculus, 70:X, in Monoamine oxid.ase (MAO) ls an enzyme
lack of insulin would interfere with this proce!!!s. the primary molars; B, Enamel rods. at cervix dentin, and 60% in bone. located ln presynaptic nerve t€rminals lhat
Therefore, under conditions of reduced insulin, slope gingil,afly, not occlu~~lly, in permanent 47. D. Poststreptococc.al glomerulonephritis is a clas- degrade5 dopamine, norepint~phrine. and epi-
hyperglycemia would result due to increased molars: C, The buccal cervical ridge is more pro- sic example of nephritic syndrome, which nephrine to inactive substances. Aspirin inhibits
gluconeogene,ri:il!: in the llver. Glycogen break- nounced. occurs after a Streptococcus infection (e.g., strep prnstaglandin synthesis by inhibiting the
down would be enhanced and glycogen synthe- 35. A McCune-Aibright's syndrome is a type of thro-,:t). It is characterized by all of the symp- cyclon:'(ygenase enzyme.
sis decreased, Since 1nsulin promote:$ amino fibrous dysplasia that presents with a triad toms listed except polyuria. A clecrease In urina~ 59. A. Said to be the most atypical of human molars.
acid uptake in the liver, lack of insulin would of symptom~ including polyostotic fibrous tion is usually observed. 60. B. O::itr"omye!iti:-:., or an infN:tinn of hone, is mnst
result in decreased uptake. dysplasia, cafe au lait spots. and endocrine 48. D. ln the cusp-embrasure occlusal relationship, commonly caused by ~'~raphy!nco<X1.H <Wr('us.
25. D. 12 hours is required. abnormalities. the maxillary first premolar is most likely to Symptom~ include pain and .systemic signs of
26. C. The roof of the orbit consists of lesser wing of 36. B. Fibrous dysplasia is caused by replacement of articulate with the first and second premoln.rs. infection (i.e,, fever, malaise).
the spheno!d bone and the orbital plate of the normal bone with an irregular bone containing 49. C. Unlvetsal tooth numbering system for primary 61. C. lntercalated discs are: only found in cardi<.1c mus-
frontal bone (not listed). fibrous connective tissue. This gives the radiM dentition, A though T. cle. Multiple, periphernlly positioned nuclei are
27. C. The dimension of the pf:":rmanent maxillary ographs a characteristic ground glass appear- 50. B. Oblique alveolodental fibers resist occ!usal found in the fibers of skeletal muscle. Smooth
canine at the widest MO diameter is 7.5 mm (7.5 ance. forces tha1 occur along the long axis of the muscle cells ar(· hpindle-shapecL
mm, Ash and Nelson. 2003; 7.6 mm [6.3 to 9.5 37. B, Fibrous dysplasia is caused by replacement of tooth, The rest of the alveolode-ntal (PDL) 62. D. The DL cusp is snrn!!est on a five-loherl mandibu-
mm] Woelfel and Scheid, 2002). normal bone with an irregular bone fibers listed provide resistance against forces lar second premolar.
28. D. Facilitated diflusion does not require energy and fibrous connective tissue. Therefor~. the which pull the tooth in an occlusal direction 63. C. Although fluoride may be incorporated ln min~
moves down a concentration gradient. Active (Le., forces that try to pull the tooth from its eraiized tissues, it is not required for the miner-
transport, in addition to utilizing energy, is often socket). ali2atlon process. Matrix vesicles p:mducec! by
374 .., Sample f:xam Ans,n::s hey Sample Exam Answer Key • 375

osteob!asts !Ire considered to be the lnitlal site the urea cycle. Ornithine. arginlnosuccinate, hemoglobin saturation Is not linear but, rather, 98, D. The Western blot tests for HIV prott:ins. The
of mineralization. Amt!logenins are matrix pro- aspartate and dtrulline are generated tn the sigmoidal. Percent saturation increases gr~aUy £USA tests for HlV antibodies. V-/hen the~e two
teins in enamel that regulate crystallite growth. urea cycle but do not provide free ammonia for at !ow PO., and less at higher PO.,. Increased tests are used together, a 99'K, <:1ccuracy rate is
Phosph-oryns are initiator protein• that bind urea synthesis. PCO.,, temPerature, diphosphoglycefate, and H' achieved
minerals to facliitate nudeation and crvstal 76, E. The superior, middle and lnferior constrictor decrease the affinity of hemoglobin tor 0 .., but do 99. C. Since ~10';;, ot blood transJus'inn.related ht~patil:ls
growth. ~ musdes all insert into the median pharyngeal not alter the characteristic sigmoldal n~ture of cases are cau!Sed by h~patitis C and the 1u1tient
64. E. Celiac sprue ls characterized by malabsorption ra1Jhe (the superior constrictor muscle was the the saturation curve. has a history of having had a hl0od transfusion,
and mucosa! lesions of the small intestines that only one H~ted), however. their origins dHfer. 87. A. On average, the height of curvature ls 3.5 mm on the answer ls C.
is caused by an allergy to gluten. A complication 77. C. The most favorable sequence for prevention of the mesial and 25 mm on the distal of the max- 100. D. Opportunistic infections art a serious problem
of this disease includes T<ell lymphom,15, malocclusion. il!ary central incisor; it is 3,0 mm on the mesial with patients infected with HlV Thls patient's
65. E. Renal clearance is the volume of plasma com~ 78, D. lmmedi,1t~ly following a meal, energy sources and 2.0 mm on the dlsta! of the mandibular cen- intraora! thrush is such an inff:ction, which is
pletely cleared of a substance by1he kidneys per are high. Glycogenolysis (slycogen breakdown) tral incisor, caused by Candida afbirnm. Treatment for c;rnw
unit of time. The renal clearance of glucost is is reduced. Lipolysis and oxidation of free fatty 88. B. Dysplastic cells are abnormal in appearance and dldiasi.::: includes nystaUn.
usually O since, although it ls filtered, lt ts com- acids are also unnec-essary during this time. organization. The potential to develop into a lOL C. The HIV is an RNA virus that contains three
pletely reabsorbed when the plasma glucose G!ucagon reiease is inhibited due to elevated malignant tumor is present; however, this risk enzymes: reverse trJnscripti'\se, integ:rasc, and
concentration is within normal levels. ln the glucoie. Glycolysis is increased due to elevated varies. proteas.e. Neurnrninidase i~ a protein foond on
uncontrolled diabetic, plasma glucose levels intracellular glucose. 89. D. Generally !'l jaw-closing musde, May stablltze the surface of the influenza virus.
exceed the reabsorptive capacity (Tm) and it 79. A. Hepatitis A is spread via oral-anal/fecal contam- and act with laternl jaw movements. 102. C. Although all of the microbes listed cause pneu
then appear~ in the urine, Urea is filtered and !nation. The other hepatitis viruses can be trans~ 90. D, An infection by molds, such as mucormycosis:, monia. the rn()St comn,on cause of pnemnoni<, in
paMively reabsorbed to a sliw:ht degree. mitted through blood, sexual contact, and/or results in the invasion of blood vessel endothe- oatlents with AIDS is P jirooeci ( carinil).
Creatinine is both freely filtered and secreted to perinatally. lium. This leads to hemorrhagic infarction and 103. C. ·A, Smaller in all climensious; B. All roots seen from
a slight extent Socllum and water are filtered 80. 8. Guanjne base pairs with cytosine which, on a tissue necrosis. buccal aspect: D, roots nearly the same length.
and reabsorbed to various degrees due to phys- molar basJs, will also equal 30:Xi of the DNA lf 91. A. The first rib cannot be palpated. Hi4. B. Cells degrade purine nuc!eotldes to uric acitL
iological regulation of aldosterone and ADH. guanine and cytosine constitute a total of 6mt of 92. D. Mast cells contain de-nse granules with inflam- Xanthlne is an intermediate in a series of reac~
66. R DiGeorge'ia .~:yndrome is characterized by a defi- DNA, the remaining percentage ( 40'?0,) must be matory mediators, including histamine and tions, but the final product in humans ls uric
ciency of T cells. It is caused by the failure of the equally divided between the base pairs of ade- SRSA. ~hich is a leukotriene, acid which is excreted in the urine. Urea is the
third and fourth brachial pouches to develop nine and thymine (20% + 209[;). 93. B. The hypothalamus coordinates many acUvities degradation product of compounds containing
normally, resulting 1n a lack of thymus and 8L B. The muscularis externa has a thlrd iayer in the mediated by the ANS, including food intake. amino groups (.amino acids).
parathyroid development. stomach. It is an inner oblique layer oJ smooth thirst, and temperature regulation. The cerebel- 105. B. An essential amlno add is one that cannot be
67, A. Restriction enzymes are also known as restric- muscle. In the rest of the dige~tive tract, the lum functions to regulate movement and pos~ 5ynthe.~.ized by hum;,n an<l therefore we require
tion endonucleases. These enzymes deave DNA muscuiaris extema has two layers: an inner dr~ ture. The medulla Is rciponsible for coordinating them in the diet from plant or bacterial sources.
at specific sites to release fragments of DNA for cular layer and an outer longitudinal Jayer. respiration, The cerebral cortex performs func· All of the nther amino acids can he synthesized
further analysis and characterization by com~ 82. C. Gummas are granulomas that may be seen in ter~ tions of perceptlon, cognition, higher motor func- by humans from other compounds and there-
plementary probes, tiary syphilis. Since syphilis is caused by tions. memory, and emotion. fore are not classified as "essential.,.
68. A. The bile canaticuH drain bile to interlobular Treponema pallidum, the correct answer is C, 94. E, Chronic bronchitis predisposes those affected W6, 8. The fovea central!s only contains cone cells. lt is
ducts. The interlobular duds form right and left Granulomas may also be seen in tuberculosis, an to squamou~ neoplasia of the bronchial epithe- located approximat...,1y 2,5 mm lateral to the
hepatic ducts, These ducts join to form the com- infection caused by Mycohacten·um tuberculosis; lium (i.~ .• bronchogenic carcinoma). optic disc in an yeHow~pigmented area ~macu!a
mon hepatic duct. The gallbladder arises from however, in this disease, they are known as 95. A. The MB, DB, and distal (D) cusps are almost !una). Vision is 1nost acute from this area,
the common hepatic duct. tubercles. equal in size, whPreas the D cusp of the perma- 107, C. The middle trunk of the brachia! j)lexus of
69. D. There is no evidence of a connection between 83. A. The medulla of the thymus contains Hassa.ll's nent molar is smaller. B, The primary molar nerves arlses from C7.
myositis and bruxing. corpuscles, which conslst of epithelial cells with crown Is narrower, not wider BL; C, The primary l 08. E. Both F,pidermorh.~1on and Trichophytnn can
70. E. The superior and inferior ophthalmic veins keratohyaline granules. The medulla is the molar outline is somewhat rectangular; D, cause tlnea pccli!'.
drain into the facial vein and cavernous sinus. llghter-staining (less dense) central area of the The ratio of crown/root not the same. 109. B. Comparatively, roots of primary anterior teeth
7!. D. The partial lhromboplastin time (PTT) test gland, where T cells maturation occurs. 96. B. ln smooth muscle, the binding of calcium to cal- are narrower nnd longer. A, Crowns of anterior
measures the intrinsic and common pathways of 84. C. The mass-eter originates from the inferior border mondutin wlH activate the enzyme rnyosin light primary teeth are wider, not narrower; C.
the coagul.~tion cascade. A prolonged PTT could of the zygomatic arch; specifi.cally, its superficial chain kinase. This enzyme pho5phorylate5 Cervical ridges of primary anterior teeth mnre
result from a deficiency of factor V, VIII. IX, X, XI, head and deep head originate from the anterior myosin, allowing it to bind to actin, and the mus- prominent; D, B. and L surfaces are rrwre fiat.
or XH or of prothrombln or fibrinogen. two thirds or posterior one third of the inferior cle ('0ntracts. For contraction in skeletal and car- not less flat.
72. C. Hemophilia. A is caused by a deficiency of factor border, respectively. Its superficlaJ head inserts diac muscle, calcium binds to troponin C, 110. C. Cystic fibro::;ls results from an abnormal chto-
8 (•n!ihemophilic factor). into the lateral surface of the angle of the 97. A. Estrogen is a steroid hormone and therefore does rlde channel that affE>cts all exocrine glands.
73. C. Hemophilia A is a hereditary disorder that is X- mandible; its deep head Inserts into thf': rarnus not require a membrane receptor and the second 111, A. The site of cell dlvjsion (mitosis) occurs in the
linked; thus, it only affects males. However, and body of the mandible. messenger cAMP, Steroid hormones diredly enter stratum basale (basal layer, stratum gerrnlna-
females can be carriers. 85. D. The width and depth of arch are more or less the cell and stimulate intraceBular receptors. tivum) of oral epithelium.
74. B. The only way to distinguish hemophilia A from constant after 9 months of age. However, a sub~ Peptlde hormones and those derived from single 112. A. IgA b;. found ln mucosJl sccn~tl0r.s of the gcni~
hemophilia B is to assay the levels of coagula- stantlal increase ln the anteriorwposterlor dis- amino acids are not readily lipid-soluble (charac- tnurinary, lnlestinal, and respiratory tract;-;,
tlon factors. Hemophilia A 1s caused by a defi- tance occurs for permanent molars. teristic of membranes) and thus require mem- including tears, saliva and colostrums.
ciency of factor 8, white hemophilia B is caused 86. A. Hemoglobin is a globular protein responsible for brane receptors and a "second m~senger system 113. A, Chronk' granulnm,,to5is results from neutrophils
hy a deficiency of factor 9. transporting most of the o. in the blood. The (cAMP, IP3, etc.). Many of the effects of epineph- wlth a defectivP. NAPDH oxidase system. This
75. E. Arginine is an amino acid that is deaminatetl to percent :,aturation of hemoglobin is a function of rine and norepinephrine and all of the effects of affects their ability to kill micrnorganisms. since
form omithine primarily in the tiver as part of the PO, of the blood. The relationship of PO2 and glucagon are mediated by cA1v1P. they are unable to produce supernxlde radicals,
376 v Sample Exam An:-;wer KeJ Sample Exam Answer Key ,. 377

114. B. Tdrahydrofo!k acid ls the coenzvrue form of the s:Jhmandihular triangle. Contents of the ]4 L A. The most common cause of subacute endo- 1:14, A. The terminology nsed is a classical d<.:sz:riptor
folic acid requlred for the synthe;is of nucleic poslerior t!"langle include the external jugular carditis is Strept-:JCo<:cus viridon.-;. S. viridons is a oi tht: relationship hetweP.n the hormone source
acid" and normal cell divtsi(;n and replication_ and subc!avian vein and their tributaries, the n-hemolytk streptococci, representing incom~ and target tissue. Paracrine refers tn the target
115. 0, Plwnylketonuria (PKUJ is an inherited disorder subclavian artery and Its branches, branches of µ!ete lysis o! red blood cells in laboratory tissue as a cell ln close proximity to !ht.~ secn~t-
Df amino acid metabolism in whict: the affected the cervical plexus, CN XL nerves to the upper cultures. ing tissue. Autocrine stimul.:.1tion n.•fers to cell
individual lacks enzymes to metaboUze pheny~ limb and muscles of the triangle floor, the 142. E. During subacute end0c1Jrditls, vegetations, or tha! releases ;:i factor (hormone) which sUmu-
lalanlne. Albinism is a conclition that results in a phrenic nerve, and the brachial plexu5. thrombi. form on previously dam~ged heart iates the cell from which it was released,
defect in tyrosine metabolism and the inability to 132. A. The a-chains are polypeptide5 which character~ valves. Compiicati..ons: can arise if the thrnmbu, Endocrine factors l'lre twrmoncs that ht:ve an
produce melanin. Porphyrla is an lnherlted di~- ize the tollagen molecule. The most abundant embo!izes. (Le. when a fragment separates and effect at a distal site in the body.
order irwolving defects In heme svr:thei,is. amino aclds ln collagen are proline and glycine. enters the circulation), causing septi-c infarct5. 155, C. Pye!onephritis ls usually caused by gram-neg.1~
Ho,wxyst!nurla is a disorGer in the m;tatmlism Proline (due to its amino ring structure), lnter- Other complications include valvular dysfunc- Eve, rod~shaped )Jacteria that are pcirt oi the
of homocysteine resulting ln high levels of homo- rupts the a-chain, resulting in the bending/twist- tion or ahscess fom1ation. normal flora of the enteric tract lt is most com-
cytteine and methionine Jn plasma and urine. ing of the peptide. Glyclne (due to its small size) 143. C. Aminoglycosides block the 30S ribosomal sub- monly ..::aused hy E,;cherichia cori, followed by
JI 6. D. Molecular tests llke the polymerase chain reac- flts into the smaller spaces of the peptide and ls unlt to inhibit protein synthesis. Proten,;, fi;.7ehswlla. nn<l Entemhacter.
tion have a fast. not slow, turnaround time. positioned at every third position in the chain, 144. 8. Translation ls the conversion of information 156. C. The salivary gland~ are lined by simple cuboid.al
I l 7. A. RJfampin causes orange, not green, urine, tears, 133, 8. Increasing the hydrostatic pressure within the from mRNA into a protein. Transduction is the epitheHurn. This type o! epithelium al~n lines the
and sweat. glomcrnlus would decrease the nee uftrafiltra. incorporation of genetic information carried by bronchiole~, thyroid gland, and ovary capsule,
118. B. The drugs binds tightly only to prokaryotrc R'.>!A tion pre~sure. Reduced plasma protein concen- a virus into bacteria, 157. E. The macula dens.a is a component of the juxt.a-
polymerase, D, Rifampin induces activity of tration decreases the oncotic pressure in the 145. A. Maximum condylar rotation takes place with glomerular .apparatus which functions in regul.a-
hepatic mixed ox!dases. plasma which would increase filtration. maximum jaw opening. tion of blood pres:.ur~. The proximal conv0lut,-xl
I 19. C. Naproxen (naprosyn) has not been reported to Vasodilation of the afferent arterioles would 146. B. The right subclavian artery arises from the bra- tubule, diMal convolut12d nibule, Bo-wrnan's cap•
exacerbate bruxlsm. For A, refer to Romane!H increase capHlary pressure resulting fn chiocepha!ic artery, and the left subdavian sule, and glomerulus all function in the produc-
et al., 1996; B. Reier to Gerber and Lynd, 1998; D. increased Hltration. lnulin, although used to artery arises from the aortic arch. The subcla- tion of urine.
Refer to Milosevic; et al., 1999. rneasure glomerular filtration, does not affect vian artery becomes the axmary artery upon I58. D. internal aponeurose5 do mov(~ <1nd deform.
1'.W. C. The mechanisms of the two types of headaches glomerular flltration rate. crossing the first rib. The axillary artery 159. C. The cricopharyngeus rnuscle prevents swallow~
are quite different. A, B, and Dare considered true. 134. A. The mo5t common form of breast cancer is ade- becomes the brachia! artery when it leaves the ing air at the pharyngeal end of the esophagus,
121. B. ,B-arnyloid dt-posit& are a romponent of AJzheimer nocarcinoma aristng from the ductal epithelium, axilla. IGO. C. The most common cause of bacterial mcningit1,
disease but not of migraine headaches. ] :35. D. The labial, mesioincisal angle of tooth #8 and #9 147. D. Clinically ln relatlon to lnterocclusa\ space. in newborns ls E'>cherichia coli. Hocmnphtlu,tt
122. A. (Montgomery et al., 1996). (central incisors) is a sharp, right angle, not 148. B. The concentratioo of potas5ium ions within influenzae and N(:iss.eria meningitides are the
123. B. Infection is most likely to involve the submaxil- slightly rounded. The labial, mesioincisal angle cells is approximately 30 times greater than the most common cause of bi'lcterial m<"ningiti::; in
lary space, considering the clinical findingiiL of tooth #7 and #10 (lateral lndsors) is slightly extracellular fluid~ the other ions are ln greater infonts;children or young adults, respectively.
l 2,1. A. Given the clinical finding~ and the involved molar rounded, not a sharp right angle. concentrations in extraceUular fluid compared Hil. B. Glucose transport tn muscle und acll[)ose tis~
position, it is the lateral upper d~ep cervical node 136. C. The apex of a medullary pyramid in the kidney is with values within the cell, sue is under the influence of ins.u!in-s-ensitive
group that would most likely be the first involved called the renal papilla. The cortex is the outer 149, B. Neutrophils are quick to arrive to the site of glucose transporters. Although several meta-
with lymph drainage (Shaler et al.. l98:l), layer o( the kfdney. The medulla i~ the inl)er layer. infection or injury and are therefore associated bolic pathwr;ys are influenced by insuHn. glu-
125. B. Oral TB most often involves the tongue. Minor calyces receive secretions from the renal with acute inflammation. AH of the other cells cose uptake intn the llver is not affected by
126. B. L.c1.ngerhans giant cells. ep1theliod cells, and pap!Hae. Several mlnor calyces join to form a Hsted are mediators of chronic inflammation. insulin.
caseous necrotic areas are highly 5ugge5tive of major cal:yx. 150, A. The correct answer (7 to 8 years) is based on 162. C. UeOA'}'genated blood from the transverse sinus
TB; however, special stains (or acid-fast bacilli 137. D. Endocarditi~ is most commonly caused by chronologll~5 in Ash and Nelson, 2003 and drains to the sigmoid sinus. which empties into
(AFB) and i:o:olatton of M. tuberculosis may be group B. ct-hemolytic streptococcl, such as -5'. Woelfel and Scheid, 2002, which are based on the internal jugular veins, The transverse
necesary, keeping in mind that staining may not viridanJ, S mutans, S sanguis, and S. salwarius. original chronologies by Schour and Massler. sinuses receive blood from the confll!('nce of
differentiate TB from other mycobacteri<:11 infec~ All of the other conditions are associal ed with modified for p~rm,enent dentition by Kronfeld s[nuses, which is located in the posterior cra-
Hons, Other molecular tests may be indicated. group A j3-,.l1~molytic streptococcal infections. (Bur) and by Kronfeld and Schour (193~) for nium.
127, C. The anticoagulant effect oJ warfarin is inhihtted such a5 5. pyogene:c (leciduou~ teeth. Also, with slight modifica- 16:i. C. The cf;'ntral groove- is well--<leflned. A, The pri-
by rifampin. A Rifornpin decreases the anticoag- 138. D. After branching from the mandibular nerve (CN tions, by McCall and Sch our ( cited by Orban, mary maxillary second molar does have a well~
ulation effect of warfarin; B, Rifampln interferes V3), the atiriculotemporal nerve travels posteri- 1981). defined me~ial triangular fossa; !3-, The oblique
with cyclic conversion; D. Rifampin increases orly and encircles th_. middle meninge;,I artery, 151. E. Heparin, an anticoagulant is unlike the other gly- ridge is promineni; D. The supplerr:entmy cusp
the metabolfc clearance, remaining posterlor and medial to the condyle. co5.tuninoglycans since it is an extrace[lufar is not wcll-developt>d.
128. A. Gut-assoclated lymphoid tissue (CAL1) pro- It then continues up towards the TMJ, e>,;ternaI compound. Hyaluronic acid is found in synovial I 64. B. Facial nerves are derived from the second
duces s:ecretory IgA. ear, and temporal region, passing through the fluid, kerat8n sulfate in loo::;e connective tissue branchial arch. The tri~E-rninal nerve is derivPd
129, C, Systemlc lupus ery!lwmatosus, also known as parotid gland and traveling wlth the superficial such as cornea. chon<lroltin sulfate in cartilage, from the first branchial arch.
lupus, ls considered an autoimmune disease. temporal artery and vein, ligament5, and tendons and dermatan sulfate in 165, B, A probe is a single strand of DNA synthe-sized
although the exact cause is unknown. Il is char- 139. A. Given the patient's history of rheumatic fever, skin and blood vessels. with a rndloisotope tlwt ls comrlimentary to a
acten.zed by the presence of antinndear anti- the heart murmur ls most likely from a dysfunc- 152. A. The pancreas ls enveloped at its head by the segment of DNA of interest. The radio-labeled
bodies (ANA). Common A'.'IA findings include tioning mitra.1 valve. Rheumatic fever most com~ first part of the dundenum. complex can be analyzed snhsequently on a gel
anti-DN.-'., anti-RNA, and anti-Sm antigen. monly affects the mitral valve, resulting in 153. B. lndivirluat~ s11fferingfrorn mucopolysaccharidoses by a technique knmvn a~ a Southern Blot
J;!O. D. The backbone of DNA and RNA is composed of mitra! valve slenosis, regurgitation. or both. have normal production of prnteoglyc.ans and gly- 166. A. B, 'The outline of the c?'O\\'tt converges di_-;tai!y:
5ugars (deoxyribose or ribose) joined 3' ➔ 5'. 140. B. Rheumatic fever is usually 1Jreceded by a group eosaminoglyc.ins but, due to genetic defects, Jack C, The three buccd cusps are simffot in size; D,
1:-n. C. The hypoglossa! (CN XII) nerve is not found in A streptococcus respiratory infection (e,g., the enzymes which degrade mucopnlysaccha- Well--defincd triangular ridges extend from the
the posterior triangle~ lt is, however, present 1n strep throat or pharyngitis} rides. cusp tips.
378 v- Sample Exam An'.'.wer i\ey Sample Exam Answer Key .- 379

167. B. '.\11: DB height equals MB height: Jv12: DB cusp 18~ D. PhfJspholipas.e C, activaled by a t:ompont>nt 193. C. Greatei· than 5'.;;~ (even up to 40'';;) in pnpulfltinns 205. D. Ureters travel inferiorly ju,;t below tl"w p;i,rleial
height slightly less than iv1B cusp height: M3; OB IJf the- G-protein (Ga) complf!Xed with GTP, cat- with Mongolian traits, A, Does not exceed 4,.'.!'X periton-e-um of the postt;rior body wall. They
cu~p height much shorter than MB cusp height. alyzes the prc,cluction of diacylglycerol and in Caucasians: B, Le..ss than 5'X. in Eurasiar: and pass anterior to the common iliac Mlt:·ries .:is
168. B. Radiographk findings oi a cotton-wool a1)pear- inositol trisphosphate. Diacylglycerol activates Asian populations; D, Less than :n, In African they er:ter the pelvis.
a:1ce is commou in patients dia.gnos~d with proteln kinases. which phosphorylMes pro~ populatio1:s. 20(i A. Aspartame i,i; il peptldf' rieriYntive con:pn-.t·d of
osteitis deformuns or Pagefs disease, teins, and inositol trisphosphate produces 194. C. The salivary glands are under the Influence of aspartic acid and phenylalcrninc
1-69. B. /:1 tbe process of muscle contraction, KfP bound increas~d release of calcium from intracellular both the sympathetic and p-0rasympathetic 207. E. The abducens ne.-rve 1CN VI) provides innerva~
to myo,ln i! cleaved to form ADP and PL l'vlyosin stores. nervous system. Parasympathetic stimulation tion to the lateral rectus muscle, which moves
is rdeas.t"d to blnd to a new actin site producing 183. E. JL.S is released by helpe:r T cells to stimulate !3 results in a more watery secretion c:omparto:d the eyeball latera!iy, Le. abducts the eye. The
a force-generating stroke. Sodium influx and cal- lymphocytes to differentiate into plasma cells wlth sympathetic .1timulation, which prnduces medial rectus muscle, which is Innervated by
cium binding are involved in the excitation-con• It :,lso activates eos!nophils and increases the inuea5ed amounts of protein with reduced the ocu!omotor nerve (CN II!\ i:l responsihlP for
traction coupling 1>rocess but do not serve as a production of lgA. volume, The vagus ts parasympathetic. which ln adduction of the eyeball.
source of ener1;,ry 184. B. The salivary gl.lmds are influenced by both the general is ~timulatory to gastrointestinal tract. 208, D. Potassium is passively (via the co1 mm.port sys-
170. A. C fibers are the smallest of the sensory and sympathetk and para~ympathetic (glossopha- In general, sympathetics are inhibitory to the tem) secreted from !he: plasma into th~ distal
motor fibers. They are postg.:mglionic, unmyeH- ryngeal and facial) branches of the ANS, The gastrointestinal tract. (The exceptions are sali~ and coll~cting tubules. Under conditions of aci-
nated, and have the slowest comhl(:·tion velocity. oculomotor nerve (CN Ill) carries pant:!lympl'l- vary glands). dosis, an individual may become hyperkalemk:
171. B. The tendon of the tensor tympani is attached to tbetic fihers to the eye which, when ~tlmulnted, 195. C. Transverse ridge connecting MB and ML cusps. since the klclneys v..-m retain K and secrete M.
the hanct!e of the mctlleus in the middle eac prnduces constriction of the puptL The ] 96, D. Jaundice is char,icterized by yellowness of tis- Under conditions of alkalosis, K secretion will
Lou<I sounds cause the tensor tympani to con- parasympathetic system innervates primarily sues, including skin, eye"s, and mucous mem~ be increased and B secretion reduced.
tract, pulling the malleu.s ancl tympnnlc mem- smooth and cardiac muscle, resulting in specific branes. It is caused by diseases that increase 209, B. Eplphyseal closure marks the end of growth ln
brane inward to reduce vibrations and prevent activation and responses, serum conjugated and/or unconjugated biliru- length of long bone.<J..
damage. 185. A. from the incisa! aspect, the maxillary central bln. These conditions include liver disease, 210. A. Mhprotein is 1.111 important virulent factor found
172. B. Increased levels of alkaline phosphatase may be incisor has a greater MD diameter (8.5 mm) such as cirrhosis or hepatitis; hemolytic ane~ nnly in the species Strepf(Kocn1s r,yngcnes.
observed ln diseases that diso!av extensive than FL diameter (7.0 mm). B, The MD di,i_meter mias; obstruction of the common bile duct, 211. A. The initi<1! rate~limltlng n:::,actlon involves the
bone loss. Osteoporosis is cha;acterized by a is 7.5 mm and the FL diameter is 8.0 mm; C, The as caused by gallstones or cholelithlasis; and removal of six carbons from cholesterol and
decrease in bone mass. There are either normal l'vlD diameter is 7.0 mm and the FL diameter is carcinoma involving the head of the µanc:reas. hydroxylifttinn of the steroid nudeus to produce
or decreased levels of alkaline phosphata~e in 7.5 mm; D, The MD diameter is 5.5 mm and the 197. B. Arteriovenous anastomoses in deeper skin are pregnf'nn!on,?, Pregnenolone can be further iso--
patient! diagnosed with this disease. FL diameter is 6.0 mm. lmportant in thermoregulation. merizecl and oxidized to produce the other
173. A. Peutz-Jeghers syndrome is a hereditary disease 186, A. The ve5tlgial cleft of Rathke's pouch is located 198. A. Complications of Barrett's esophagus include: steroid h,;rmones.
that is transmitted by autosomal dominance. between the anterior and posterior lobes- adenocarcinoma, stricture formation, or hemor- 212. D. Use of the formula indicates on the basis of± 6
174. I). Ternporomandibular (T:VIJ) (Ash and Nelson, specifically, hetween the pars interrned!a and rhage (ulceration). months that the ADF is 2 to 3 years.
2003): craniomandibulr11 (Woelfel and Scheid. anterior lobe. lt consists of cyst-like spaces 199, D. The elongation and overgrowth of fiHform 213. C. A and Bare correct Dis per the manufaC'turer·s
2002). (R..lthke"s cysts) and represents the vestjgial papillae results in hairy tongue. Filiform papil- recommendalions on product, 200S.
175, B. Tuberculosis ls caused by the bacteria lumen of Rl'lthke'! pouch. lae are thin, pointy projections that make up 214. B. Systemic etiologic factors for bypopla~ia are
:'v!ycobacteriwn toberc11{0.,is. These bacteria can~ 187. C. A, Enamel cap thinner and more consistent in the most numerous papillae and give:!I the thought to occur possibly a!tcr birth and
not be Gram stalned due to their waxy cell walls, rtepth; B, Compartttively greater thickness; D, tongue's dorsal surface its chllracteri5tlc rough before 6 years of age.
Therefore, to identify these bacteria, an acid~ Pulp horns are higher 1n primary molars. texture. Note: a loss of fl!Horm papillae results 215. C. Measles ls not known to cause enamel hypoplasia.
fast stain must be ordered. 188. B. The regulatory centers for respiration are in the in giossitus. 2.16. D. Intermediate fllument is a smooth muscle fHa-
17G, D. M tuberculosis Infects rnanophages, whkh are brain stem-spetificatly, the medulla, The 200. B. Tertic.iry dentin, or reactive/reparative dentin, is ment. Cnnnectln is another name for titin.
initlally unable to kill the phagoc-ytosed bacteria. medulla contains three groups of neurons dentin that is formed in localized areas in Stntements relative to A, B. and C are true and
177, A. Cord factor i.s a glycnlipid found on the surface ( nwdullary respiratory center, apneustic center respon5e to trauma or other stimuli, such as refer to sln'!ctaf muscic filaments,
of ilJ tubf'rculnsis. and pneumotaxic center). The cerebral cortex caries, tooth wear or dental work, Hi5tologically, 217. 0. l\.fasseter muscles providf;' most of lhc force
l 7R D. Type IV delayed type hypersensiti\'ity reactions may influence the medullary centers but does its consistency and organization varies; it bas no between molar teeth with dcnd1ing ln the inter•
are the only type of hypt""nH~nsitivity immune not totally control respiration. defined dentinal tubule pattern. cuspal position.
reactions that are not mediated by antibodies, 189. D. The brachia! plexus of nerves arises from flve 201. E. Innervation to the maxillary second molar, as 218. D. lmiiv1du.il5. who brnx during llw cl~iy dn r~ot n:?"c-
They are mainly mediated hy T cell5. roots from the anterior primary rnmi of spinal well as the palatal and cHstobuccal root of the essarily brux .at night,
179. E. Treatment !or tuherculnsls includes a multl~ nerves CS through CS and Tl. maxillary first molar and the maxHlary sinus, is 219. D. Sequential changes froru autonomic cardiac;
drug therapy. RHampin, i~oniazid, and ethamhu- 190. D, A,;choff bodies are the cl,:issic lesions observed in provided by the Posterior superior alveolar brain cortkal <"Crivitie-~ pn:'cede SB~relat0d jaw
tol are three of the first-line drugs used. rheumatic: fever. They are areas of focal necrosis nerve. The nerve ls a branch ,nf the maxiH,c1ry motor activity.
180. E. Ghon complex describes the caldfied scar l hat surrounded by a dense inflammatory infiltration nerve (CN V2). 22fL D. Maiocclus!on has not ber~n sug~csn:d as caus-
remains following the primary infection. It is and may be observed in heart tissues. 202. D. The supraren<:1! glands secrete epinephrine. ing or a~gravz1tlug bruxism.
usually found ln the lung and includes the pri- l9L B. Cydo-oxygenase includes isoenzymes of Specifically, chromaffin cells of the the adrenal 22!. D. Fhmrosis versus NF opacJtlc~ rnay ht.' diffkuit to
mary lesion and its region.al lymph node. prostaglandin endoperoxiclase synthase, which medulla, which act as modified post-ganglionic diagnose clinkally.
181. B, Odontogenic inft:ctions of a mandibular incisor is required for the first step in the synthesis of sympathetic neurons that synthe5ize, store and 222. B. Thick and thin fliaments do not change in lenglh.
wlth an apex helow the mylohyoid muscle have prn5taglanclin,; from arachidonic acid. secrete catecholamines. produce epinephrine. !t 223. C SHLP help, stahilize the he.ad and condyle
the pot<·ntlal to spread to the suhmental space, Phospholipa!'le A2. which is involved in the syn- a!so produces norepin~phrine. a1-;ainst tlw eminence in clenching; A. B, and D
H the apex is above the mylohyoi<l muscle, the thesis of arachidonic acid. is lnhlbitecl by 203. A. The most common cause of gastroenteritis in are true.
infedlon would spread to the subHngual ,pace. steroidal antl~inf1ammatory agents. children ls the rotavirus. ft ls found in tbe 224. D. In the asn•nding ( disU11) loop of Henle, activt'
Both of these spaces communicate with the 192. B. Terminal bronchioles are characterized by reovirus family, sodium ab~orplion results in an irH'ffa.sed
::.tihmandihular space-. ciliated cuboida! cells. 204. C. (Ash and '.'!elson, 2003.) osnwlctrlty of the interstitial fluid, which plays a
380 T Sample exam Answer hey Sample Exam Answer Key ~ 381

role ln the retention of water undE-r conditions 235. B. Cretinism is hypothyroidism ln children. Oral tose are all aldoses 5ince they contain an alde- spread down this space intn the thorax (poste~
oi dehydrat:on. fincling5 include macroglossia, prolonged reten- hyde group. rlor ir.ecli.astinum) and cause possible de11.th.
22:1. C. Under ..:onclltions of expansion of extracellular tion of primary teeth, and delayed eruption of 248, E. Both tetracycline and snepto1nycin, an amino- 26L B, Trlsomy 21, which is also known as Down syn•
volume (longAerm hype;tension). renin relea5e permanent teeth. glycoside, inhibit proteln synthesis in bacteria. drome, is a disorder affecting autosomes. 1t is
and a\do5terone secretion are reduced. 236. A. Surfllctant ls synthesized by alveolar ceHs and Streptomycin, however, is bactericidal. not hac~ generally caused by meiotic nonriisjtmction in
increasing ADH, angiotensln II. and increasing functions to reduce the surface tension of th~ teriostatic the mother, v,.·hlch results in an extra copy of
sympathetic activlty woulrl result ln an increase alveoli. This reduction of surface tension 249. C. The auricular hillocks ate derived from the first chromosome 21 (trisomy 21),
ln blood pressure. increases pulmonary compliance and decreases and second branchial arch. 262. A. Type I coi1agen is the predominant c:nllagen
226. E. The most superficial layer of th~ epldermis is the work of breathing. It will afso decrea.,e: the 250. C. The 1-ength of the primary m/\xillary central inci- forn~d ln cementum, Type lll collagen may be
the stratum corneum. From deep to superficial, tC'ndency of the lungs to collapse. Lack of sur~ sor is 16 mm (Blbck GV, cited in Ash and Nelson. present during the formation of cementum, but
the layers are basale, splnosum, g:Jndosum, faz::tant results in neonatal respiratory distress 2003;; 16 mm (Kraus et al.. 1969)·, or 17,2 mm it is largely reduted during m.ituration.
lucidurn, and corneurn syndrome. (Woelfel and Scheid. 2002). The permanent max- 263. A. Net filtration depends on the hydrostatic pres-
227, D Calcitonin is secrett:d by parafo!llcular cells 2:n. D. The red pulp of lhe spleen consists of cords, illary incisor is 23.6 mm (Ash and Nelson, 2003); sures in the glome:rular capiilary, the hydro~
(clear cells) that are located at the periphery of containlng numerous mncrophages, and venous 23.5 mm (Kraus et al., I 969); 23.6 mm (Woelfel static- pre5sure5 in bownrnn·s space, imd the
thyroid follicles in the thyroid gland. Calcitonin sinusoids. !t is the site of blood filtration. The and Scheid, 2002). 16/23.5 = 0.68 or 68~,; oncot\c pressures in the capillary and bow-
plays an important role in the regulation of cal~ whlte pulp of the spleen contains numerous T 17,2/23.6 ... (t73 or 73:t. The answer is about man's capsule. Reducing the pliit:!tma protein
cium and phosphMe-s. lt suppresses bone reab- and B lymphocytes. 70%. (reducing the oncotic pressure) will lower the
sorption, re5uJting in decreased calcium and 238. B. The velocity of blood flow ls the rate of dis~ 25L B. Ehlers·Danlos syndromes are characterized by tendency to retain fluld in the capillary. This
pho5phate reiease. placement of blood per unit time. Velocity defects in collagen (Le., connective tissues). wou!d result in a net increase in filtration.
228. C. The ratio of inorganic to organic matter Jn changes inversely with cross-sectional area 252. C. Acetyl CoA. Malonyl CoA ls produced following 264. C. Auer rods a.re observed in blast cells character-
mature dentin is approximately 70:30. in enamel (cm:;). Tht greater the area in the vessels, the carboA-ylation of ac:ctyl CoA in the :,;ynthetic istic of acute myelogenous leukemia.
and cementum, it is api,rnx!mately 96:4 and lower the velocity. Since blood is always flowing process of fatty acids. FAD and NAD are involved 265. D. Vasopressin, also known as ADH (antidiuretic
50:50, respectively. due to the distensnhllity oi the arterial tree, in fatty acid oxidation but NADPH ls the source of hormone), ls a pepUde secreted from the poste-
229. B. Tuberculin reaction (Le, PPD skin te:Jt), is an velocity is never zero, reducing agents ln the synthesis of fatty acids. rior pituitary in response to an increase in
example of a type IV delayed (cell-mediated) 239, A. Neuramini<lase i~ fotrnd on the surface envelope 25:l A. Insulin i~ a hormone that is secreted under con- serum osmolartty. The other hormones listed
hypersensitivity. Type lV reactions are the only of infhienza viruses, It functions to attach the ditions of increased sources of energy (glucose) are synthesized and secreted from the anterior
type of hypers~n!itivity imm'Jne reactions lhat virus to the host cell. and therefore results in increased uptnke and pituitary.
are not mediated by antibodles, They are mainly 240. C. These are the most common sequences of erup- storage of glucose in the form of glycogen. In 266. A. Secretory lgA is an fmmunoglobulin that is
rnerHated by T cells. tion. the process, blood glucose ls decreased. The unique to the oral cavity. The secretory com-
230. C. Vitamins are not digested by enzymes from the 241. E. All of the features listed are commonly observed transport of glucose into the brain is not ponent is synthesized by salivary -epitheHal
pancrea5; however, digestion of carbohydrates. in ml1lignant cells, except a low nuclear-cytoplas- affected by the hormone, cells and complexes with lgA to form secretory
fl1ts, and proteins may be requlred to mak~ vita- mic ratio, Due to the large nude? present, there is 254. B. Osteocytes are found Jn lacunae in mature bone. lgA.
mins avtiilable for absorption. The pancreM usually a high nuclear-cytoplasmic ratio. 255. D. From MD aspect, all posterior teeth have a 26-7. D. Marked asymmetry of mesial and rlistal halves.
produces amylase for c-arbohydr.!!ite digestion, 242. B. The two vertebra! arteries join together at the trapnoidal outline with shortest uneven side 268. D. Oxidation oi fatty adds, glucagon release, and
lipase for lipid digestion, and several prote- border of the pons to form the basilar artery, toward the ocdusai surface. A, Present prima- i:;lycog~noly,ls are all increa.5ed to provide
olytic enzymes (tryp5inogen, chymotrypsino- Branches of the bastlar provide blood supply to rily on first premolar; B, Exceptions may be the energy sources for exercising muscle, The syn-
gen, and procarboxypeptidase) ror the <lig:e!-tion the pons. DB root of the maxillary second molar; C, From the:-;i~ of llpid (lipogenesi5-) would be decreased
of protein. 24:i, C. R~A synthesis is not required tor genetic mesial and distal aspect, all posterior mandibu- during periods or exercise.
231. C. The me:u.•ncephalic nucleus contain the nuclel of cloning. All the other enzymes are required to lar teeth, not maxillary posterior teeth, have a 269. B. The thymus is active at birth and increa~e~ in size
the trigcminat sensory nerve:s (CN V) involved in synthesize and splice DNA. rhomboidal outline. until puberty (around age t2). after which it grad-
proprioception anfl the jaw jerk reflex, including 244. D. The most potent stimulant of the respiratory cen- 256, A. Due to the excessive levels of serum calcium. ually atrophies and is replaced by fa.tty tissue.
periodontal ligament fiber!'I Involved in the ter that increases the rate oi breathing: (hyper- osseous changes, such as metastatic calcifka- 270, C. Monoamine oxldase Is an enzyme located in lhe
reflex. It is located in the mldhrain and pons. ventilation) is increased C(J, tension. co._, is tions and kidney stones, wilt occur in patients presynaptic nerve terminal, which degrades
232. A. Aldostcrone secretion is mediated through the permeable to the blood-brairl barrier and lllti- with hyperparathyroidi~m. dopamine, norepinephrine, mid epinephrine to
renin-nngiotensin system. Angiotensin Il stimu~ mat0ly p!"oduces nn increase ln H' of the cerebral 257, B. This enzyme catalyzos the pho~phorylation of inactive substances
!ates the :synthesis and release o! aldosterone splnal fluid. However, plasma Wand HCO., are fructose &.pho•phate to fructose l,&.bispho•- 271. A. B, Tooth grinding in sleep bruxism lasts up to 5
from the adreru~I gland. It acts on the distal not permeable rind thu5 have little direct eff;ct on phate. This irreversible reaction is inhlblted by minutes, not 20 minutes; C, Sleep bruxism often
tubule anrl collecting ducts to increase sodium the respiratory center. The resplratory center is ATP 11.nd citrate (indicators of energy abundance coincides with p<tssage from deeper to lighter
re,i,ibtwrption an<l pota~sium -secretion, not sensitive to oxygen tension, unlike peripheral within the cell). The reaction is stimulated by sleep, not lighter to deep ~leep: D. Sleep brux-
233. A. First evldence of the maxi!lary central incisor chemoreceptors. AMP. ism occurs apprmdmt\tP!y every 90 minute,, not
calclflcation is givf'!n as 14 (13 to 16) \.\leeks by 245, D. It is a branch uf the maxillary (CN V2) nerve. 258, B. Type II coHagen is one of the fibril-forming col- every 20 minutes ln the sleep cycle
Kraus and Jordon, 1965, and by A5h and Nelson, The maxillary nerve branches from the trigemi- lagens found in cartilaginous structures, Type i 272. D. These oral findings can be ob!erved in patients
2003; this is given as 4 months by Woelfel and nal ganglion and exits tbe skull through the is characterized as having high ten.sHe &trength wlth hypNthyroi<lisrn. The only endocrine dis-
Scheid, 2003. foramt>n rntundum, When it reaches the ptery- and found in skin, tendon, bone, and dentin. order iisted that is related to hy1wrthyroidism
23-1, D. Lingua! aspect: D, Well-defined developmental gopalatine ganglion, it terminates as the in.fraor~ Type m collagen ls characterlstically more dls- is Plummer·s diseast'. It is c.,msed by a nodular
groove $eparating ML and DL cusps. A. ML cusp bital and zygomatic nerves. tensible and ii found in large blood vessels. growth or adenoma of the thyroid.
large and well-developed; 8, DL cusp well-devel- 2·16. B. Root bifurcation is more likely in the mandlbu- Type IV is found ln basement membranes. 273. C. Fat in the intestine, low pH:, and increased osmo-
oped, more so than that of tbe primary first lar canine. 259. B. (Kraus et al., 1969.) larlty of intestinal contents will reduce gastric
mol"-r; C, There is a supp!ementzil cusp apical to 24 7. B. A ketose sugar is on that contains a keto group. 260. D. From the retropharyngeal space, Le. "danger emptying, This reflex is mediated through neu-
the ML cusp, but poorly developed. Glyceraidehycle, mannose. glucose, and galac- space," odontogenic infections can quickly ronal and endocrine factors comprising the
382 .- Sample Exam Answer i\cy Sample Exam Answer Key T 383

C'nterog:astrk reflex. Sa!ine ln the intestine will 286. C. The movement of the tongue against the palate 30!. E. Urinary filtrate is most hypotonk in the distal 31:1. D, The l.l.ti:-slmus dorsl niuscle is ::supptic:d by the
nol affect gastric emptying. ls the only voluntary prm;ess among the four com-oluted tubnle, lt is isotonic in the proxlma! thoracndorsa! nerve,
274. D. Occiusal contact relations do not interfere with pos5ible answers. convoluted tubule and thick descendlng limh of 314. B. Regula.tion of blood flow to skin is under the
jaw opening. but TM.! and muscle disorders 287, A. CDS lymphocytes recognize class I MHC mo!e-- Henle's loop. lt becomes hypertonic as it pa55e~ control of factors acting locally (metaho-lites)
may. cules on antigen~pre~enting cells. CD4 lymphoM through the thin descending limb of Henle's and (f-Mirenerglc receptors. Slowing oi tlie
275. C The o!ecrannn fossa is located on the posterior cytes recognize cla>s 11 MHC molecules on loop, and becomes hypotnnic as it passes hea:t nnd actl\'at:,)n of gastrointcsti.ial mntilitv
surface of the humerus. ~nt!g:en-pre&t-nting cells, through the thick a11cending segment of Henle's are medlatecl t!mrngl1 the- cholln<'rgic systen~.
276. D. Chronic lymphocytk leukemia is characterized 288. A. Bifurcated roots are not a normal feature of loop. Flna!ly, lt becomes increasingly hypotonic a-,:uircncrgic receptors produce decreased
by the proliferation of B cells. It is both the most mandibular incisors. as H passes through the distal cn;ivoleted renal hlood flow.
common a:1.d least maiignant type of leukemia, 289. C. The inner enamel epithelium in the bell stage tubule. 315. A. The auriculot(•mpornl nerve provides primary
277, C. tRNA is not required for the hydroxylation of differentiates into ameloblasts 302. C. There are seven pairs of true ribs, meaning they lnnervation to the TM.L
collagen since the process occurs after trans!a- 290. D. The cartilage of the trachea is covered by a peri, attach directly to the sternum via coital carti- 3lfi, B. The subm,;:nl;1[ artery is a branch of the iacla!
tion of mRNA and synthesis of collagen, chondrium. The mucosa ls covered with re!pi- lages, The rernalning flve pa!rs are called false artery. Branches of the rn,:indiht1im division of
Ascorbic add is required to modify the collagen ratory epltheiium. Hyaline cartilage rings lie ribs because they attach Jndlrectly to the ster- the maxillary artery include the Inferior alveow
molecule by hydroxylating prolln~ to permit cteep to the submucosa. The open end of the num via costal cl'.\rtilages. The last pair does not lar, deep auricular. anterlor tympanlc, mylohy-
cross-linking among collagen in tissues. cartilage faces the posterior, Smooth muscle attach at al!. oid, incisive. mental and middle mrninge,:d
278. C. Distal to the second premolar, 56'Jf), extend• across the open end of each cartilage. 303. B. The total of average overall mesiodistal diame~ arteries.
279. D. A, The mesio!ingua! cusp is most prominent: 8, 291. C. Pemphigus is an ,1utoimrmme dtsease wherein ters of primary maxill.11ry crowns is 68.2 mm 317. E. Rh0umatoid arthritis. is chnractc!"ized by lnflam-
The dij/olingoa! cusp ls poorly defined; C, The autoantibodies against epiderm.ls cells are pro- (Black GV, cited by Ash and Nelson. 2003): 76.8 matlon of the s.ynovial nwmhrnne. Grnm:lntion
distobuccal cusp may ~ seen bec,111.se it is duced. Hisfolog:ically, a phenomenon <'alled mm (Woelfel and Scheid, 2002), tissue wtll form. in the synovium and expand
longer, acantho{ysis, wherein epidermal cells appl':ar to 304. C. Symptom! of a myocardial Infarction lndude over the articul<)r cartUage. This causes the
280. D. The posterior and anterior vaga! trunks pass detach and separate from each other, is chest pain, shortness of hreath, diaphoresis destruction of the underlying cartilage and
through the diaphragm through the observed. (sweating), clammy hands, nausea, and vomit- results in flbrotic chan!{es or anky!osls.
esophageal opening. The aorta enters the 292, A. following a meal, chy!omicrons are synthesized ing, 318. D. (A•h and Nelson. 2003.)
diaphragm through the median arch. the infe- in the intestine to tran,iport Hpid to the liver. 305. C. Amino acids are transported across the luminal 319. C. The trapezius muscle is ~upplic-ci by CN XL
rior vena cava through its own opening in the Aceto.:icetate (ketone body} is found under con~ surface of the tntesllne by Na' amino acid Latis.simus dorsi is supplied by the thorac:odor-
central tendon, the azygos vein through the ditions of increased II oxidation (!a,ting). cotran5porten1 in the apical membrane. Th.ls is saI nerve, !evator scapulae is !.upplied by lhe
right crus, and the ,ptanchnic nerves through Glucagon is secreted in response to reduced facilitated by the Na' g:adient established In the dorsal scapn!ar nerve. and the major and minor
the crura. b!ood sugar. which usuaHy is not present fol- intestinal cells. rhomboid muscles are supplied by the dorsal
281. B. The sympathetic postganglionic fibers lnnervatw lowing a meal. Lactate, a product of anaerobic 306, A. Of the cell type! Hsted, only smooth muscle $Capular nerve.
ing the heart release norepinephrlne. glycolys.is, is not elevat~d under conditions of cells are caoable of cell division. 320. B. A quelllng reaction occurs in a laboratory wben
Acetylcholine is neurotransmitter in sympaw elevated energy sources in the post-absorptive 307. B. Vltamin Ki~ involved with the po5ttrnnslatiomtl the poly~.-.ccharide capsule of a bacterium swells
thetic preganglionic fibers, parMympathetic state. modifi<'ation of glutamjc acid to form y-car- after being treated with antiserum or antibodies.
po5tganglionic fibers, and parasympathetic 293. A. Bruxlsm is now thought to be mainly regulated boxyglutamic add. This carboxylation permits 321. D. l.'nder conditions of Increased intra.cranial pres~
preg2t.nglionic fibers. centrally, not peripherally. prol hrombin to interact with platelets and ions 3Ure, th~ vasomotor rl?gi()ns of the medulla are
282. C. o,t~opetro5iis, also knmvn as A!bets.SchOnberg 294. A. A di!.tinct deveh)pmental groove. prominent in the proce54 of dot formation. Hydroxylation stimulalect due to localized iscllemla resulting
disease or marble bone disease, is caused by buccal triangulll.r ridge, two cusps, and distinct of prolin~ requires ascorbic acid and iron. in an increase in systemic bloocl pres,ure.
abnormal osteodasts, The lack of bone resorpw mesial and distal occlusal pits are most charac- 30R D. Squamous cell carcinoma is the most common Ventricular fibrnlation i5 !rregul,ar, rapid. unco-
tion results ln defective bone remodeling llnd teristic of the mandibular first premolar. oral cancer, occurring in 90'.)(- of all cases. It is a ordinated contractions of the ventnde that do
increased bone density, which may invade into 295. B. Autodaves function by denaturing proteins. ft is cancer of squamous epithelium, spedfically a not result in efff'cting blood movemc-nL
the bone marrow space. effective again~t spores. tumor of keratlnocytes, Anaphyliiclie shock is caused by severe t1llergic
2&1, D. Langerhans cells are located primarily in stra~ 296. D. In the initial stPp of the synthesls of porphyrin, 309. B. The permanent maxillary first premo[ar usually reactions, wirlespread release of hi3Uunine, and
tum splnnsum, sucdnyl CoA and glycine are condensed in a has two roots but they are lluccal and linguBL subsequ0nt vasodilatatinn.
284. B. Nitric oxide is a molecule produced in rate limiting ~tep in the liver. not mesial and distal. 322. D. Teichoic acid may be found only in the ePll walls
endothelial cells that acts directly on smooth 297. B. The dental lamina arises from neural crest cells. 330. D. Actinic keratosls produces dry, scaly plaques of gram-positive bact£:>ria and contain antig.Pn!t"
muscle to produce relaxation and vasodilata- 298. A. The only disease listed that is related to an with an erythematous base, They may be pre- properties. All of the other answers listed ure
tion. Therefore, inhihition of its synthesis abnormal number of chromosomes is malignant. Both compound and lntraepidermal found in the cell walls of gram-negative bacteria.
would be expected to .raise blood pressure. Klinefelter·s svndrnme. This disease is charac- nevi are benign, Only jLmctiona! nevi are con~ 323. A. The articut;:iting ,urfaces. of the TM.I are cov~
Inhibitors of angintensin 11 synthesis should terized by twC: or more X chromosomes and one s!dered to be premaligmi.nt. ered witl1 fibrocartilzige, directly over!ying
reduce blood pressure since angiotensin II or more Y chromosome. Typically, those 31L B. Secretin stimulates bicarbonate secretion from perioste-um. Thf> no1Htrticuialing ,urfacet- oi the
stimulates aldosterone 5ynthe~is resulting ln affected have 47 chromosomes with a karym the pancreatic ducts. Cholccystoklnin (CCK) is TM.I are covered wlth periosteum. The artku-
increased fluid retention. Angioten!\.in II is also otype of XXY. responsible for stimulating pancre.atic l':nzyrne latin!:! surfac~s of diarthrodial joints are covered
a potent vasoconstrictor. Blod:fni vasopress:n 299. A. Stretchlng of muscle spindles results in a reflex secretion and contraction of the gall bladder. with hy11Jine cartilage,
(also a potent vasoconstrictor) receptors that is intended to adjust the muscle to !ts rest- Chymotrypsinogen ls activated by trypsin in the '.J24. B. Both epinephrine and glucagons result in aclivi~
would reduce blood pre~sure. St!mu!ation of ing length by contracting the musde in which intestine. ti~s which serve to increase {mnlntaln·) blood
the haroreceptor would produce a reflex that It is found. The stretching of the :!lpindle in- 312. B. Textbook chronologies indicate that the maxil- glucose. Activation of giyrogcn phosphorylase
would reduce cardiac output and peripheral creases afferent lmpulses to the spinal cord and lary primary second molar is most commonly wili r.::sult in glycogen cle-grndat!on. ul!irnately
sympathetic tone, through a monosynaptic reflex stimulates mus- the iagt to erupt: 24 months (\Noelfel and Scheid, providing a source of ~lucose. Glycogen sy1;-
285. A. (Ash and Nelson, 2003: Woel!el and Scheid. cle contraction via an alpha motor neuron. 2002): 20 to 30 months (Kraus et al., 1969): 2S to thase lnhibitkm results in d0creascd synthesis
2002.) 300. C. Below the apices, 63'¾, of the time. 33 months (Ash and Nelson, 20(13). of g!ycoge:n.

I
Sample Ex:;nn Answer h. cy -.. 385

A. hy lfrm!r-'teff:1 perrus :1s1. C. a rf'ducing agent tor the A. A :niccl:e i~ a gloilu!;ir structure
\Vorking ( balancing) factors contribute to it.s vir- adds- The mo:r.ophos~ wlwn the pn!ar he;id:, of m,
intern;;! carotid ar!er:,- jni:H:d t(l rhe pos~ lgA prnte<tsc. Thi:,:; c·uzynie also produces ribose (fatty acids·: !ntPrfif't with .:iqueous
the ;Jnsreriur nu('l0otide synthesis, Ni, ATP or envlrnnnw11t th.::· r;(lJlJJnbr hvdro-
part /Jf the of is produced in the 1;;,\tlv.vay. Fructose l,(i-bis~ dustered insld(, -
Wil:is C. produced during
::t'.:7, E. intl, (and fat) C. ,\tr,entw·,,m,s t'iriduns
cells ls due to the activlty of glucose lrnnsporters ume ctfl)mmt of air exdrnr.ged \ exp!ra- endocasdltis. Sraphy-
\•/hkh (ioes not require ATP and l ion and ii1spiratli'1l) norm.Jl qult't fococcus aurew; c-orrunc111 cause 1Jf
pendent of any ionic cr-:wt'nl r;;tlnn h:eat!1ing. Function;1I acutt· endneardltis for-
is requir0d !or the actlvity of ali combir.ation of the expired for the crnwn outline converges on!y not before.
(fnrced t::<piratl<rn) plus the residual volume. R has h0t>n '.:\6K A. Emlntnx!n is found in frw
C. Vital capacity the vo!;mw of expired z:dt,::r ridge ceU wails of g:rnnH10g;;:tive
inspiration. as well as th,~ lower A.. The trm·h!e.a of tlie hunwrus art'lcu\ates with
3-12, D. ,ih:sful aspect: A, Pronounced convexity present and the ulna nf the /o;earn), The of the•
3'29. C. (Ash ;rnd chi;;1, primarl!y drain into the lrnrnerus bnne articu1nt(•$ oi the
devE:lnpmental groove the rnesiai marginal The rest of the mandihuiar teeth (premolars and fon;arm. The cornnoid fossa, located just sulH'~
ridge-. occlusal third: D, !'v1e-slollngual longer and molars) mainly <lrnin Into the suhnrnnriibul0r rior to ihe- trochiea, fits the coron-old proce~s of
330. D. A!l posterior teeth vi(~wed irorn distal aspect sharper than mesiohuccal cusp, nodes. the ulna the forearm. The olccranon of
have a rhnmboldal rnJtltne. 34'.t C. The deve!ops from 355 D. Rickets ls a vHamin D deficie-ncv seen in infants the hurncrn.s Hts the nlecrnnon of the ulna of the
D. Dextrans t1H1leeu!0s ilnked The blastocyst, and tcr,nhnhh,t and children. Ora! findings lnducle delayed furean;1. The medial epicnndv!e on tlie
the structurn! include structures of of teeth i.lml abnormal dentin. humerus itself and serves a.s :ittachment
contribute to cnelern Uwt v.·m lead to devt:'•lnpment A. In irreversible oxidative for muscles
near the tooth. amnion, vitellin<.'. sac. and chnrion. pyruvate to acetyt CoA 370. A. During a fast, catahnlism ls: increased to provide
Fructans or lev.ans are al.so found Ill plaque; how~ :344_ C. lmmnln"•ics SO that of nase, the five additional smm:es of enerh'Y· This i.s ch<ffacter~
ever, they contain fructose . central 357. D. A oe,;eHmrncrlf,11 increasrd utilization of laHy aclcls, When
a32. C. Substitution of most ions (wlth the of inctsor months (Woelfel and divides the production of acetyl CoA (produced by
fluoride) will increase the !;o!ubillly of Scheif!, months (Kraus et al., 1969): 8, It has a rtwm- enhanced the oxidative
ta! structure, The ratio of calcium and 7.5 months (Charlick or 8 (6 t-o 10) The
lmiro:n,anatite is l.67: I. months (Ash and Nelson. mesia! buccal cusp is
C. from ectodenn. Dentin 345. It Prr,c,estemr,c entrntion ln blood is highest 358. D. Fibers of the superior of the lateral and u!vrnc·em,sis
are derived from rnesode,m. of U-1 (leutcini?ing hormone) muscle attach to the anterior encl of inhibited due to !imHed enc-rg:y
'.t34, of Henle's loop contains sqiw- which to balance and stabilize the CoA production in the liver is
mous cells. The prf1 xim:1! convoluted disc closure. Pnlrnncc,d ;J-m:idation
tubule, also known the thick descending limb 359. B. (Ash and 2003.) :HJ, D. The nu deus is found in ! he medulla
of Hen!e's loop, the thick segment of 360. D. HBe-Ag is present when there is active vlrnl contnins the eel! bodies of
Henle's Ioop. and the di~ta! tubule replication and the rarrier is motor neurons for CN lX. X and Xl The cc•il bod•
all of cnhnld:1! epithelial cells. B. left ,itriovPntrlcnlM valve of the he,1rt is also C. Saliva Is fornwd by involves ies of CN Vil, IX and x·s
df'h_ydrogenase and known the mitral The the fonnatlon of solutlon contained in the nudeus of
(ir-hyc!rogena!H'. are irre- trkul.:,r valve of the heart is also which is subsequently A. Calclurn is storf:d and released from sar~
verslble enz:•it:nH~s in the pcntose phnspilr!le tricuspld va!ve. The aortic valve prevents cells to a more hypotonic solution rPtknlum during excilatlon-<:ontrac~
ivi.thway, Gluc0kinase is i:woh.:t~d with gitatinn of hlood from the aorta h<Kk into The modification pri:na· This an extensive
rylatlon of glucose when ventride and pulmonary valve of $Odium and pi:•rmitting lntr;K<-l!u-
of high. Glucose 6-!Jl1ospli,1taf.e regurgitt1Hon of blood from the and s(~cretiN1 potassium and bicar~ !ar free ca::, to be lnw when the mus-de fiber is
6-phosplrnlP- to form artery bark Jntn the ventricle. bonate. As salivarv flow increases, thr: riuctilf: at rest. The rele<1s(• of calcium is due to confnr-
final step in '.i/47, C. The leading cause of cells have l(~ss tlmf'~ to modify the composition of matinn;i! chrmges tvlllch opens ch•rnnds in
33fi. D. of contour on dislal is can:!iovnsr:u!nr dlsease. Other rnmn11c,,,t1,rns saliva, \.•,;hkh results gret1ter concentrations of lhe sarcop!asmlc retictd11rn.
include retinopathy, nephrupathy, ;md periph· sodium and chloride ions, Bicarbonate concen- 373. B. The vaccine consists of tetanus toxold
'.J'.57. E. Caf'diac tarnponad(~ is a .<:H:rious eonclltion em! ne11ropathy. tration incn,ascs salivary flow increases due 1t is part of the DPT vaccine aml should be i.;iw:11
caused by an::un1ttlatinn of fluid the peri- '.):18, C. The corr('d answer for A would be abnut 1.5 to the st imul<tilon of bicarbonate by about everv rn
cardium. This can r<·sul1 in impnlred ventricnl.ar months or 6 weeks: About 4 to C, the pr1rasympt1thNic The salivary C. of c~_mtnur on lingual surfaces of mo!ars
filling and rnn lead to denPascd cardiac About 2.5 monlhs or JO n:eel:.s.' D, About to 5 glands are prcinnl:irs is at thC' cervical or rnidt!Ie thinl.
output rmd branches 375. A. \1usclcs are innervated by two lype~ ot motor
'.1:38 B. The correct order of tooth formati,m D. centrnl deve·lopmental groove is ~162. B. The oxy- netirons (~1lpha and g;;mnrn). The alpha rno\1):i-
arneloblasts fnrrn. od,mtnblnsts form. <,donto- found on {he mandtbulilr second premotnr. :wurons innervate musde ill1r•rs
blast.s start to form dentin, ;1rnf:'!(lblasts start !.o :):10. B. Tht' sternal between rile manubrium and (ex1rafusnl skeletal m;1scle.s).
form enamel. the sternum marks tlie position of second iwuron;'-. innervate the ;rrnsde
]'.19. C. Reduction dlvlslon o(:curs during the first stage rib. Frnm this location ribs c;m be (·01mtecl fusnl musd<• filwrs"}. inuscks (lf lh<:' irio-
of mt•ioSais. The second stag0 mirrors mitosis exlernal!y. This Is bt?cmise the first 363, are characterized by <'I contr0Hed both sympatl!clic i11nerva!io11
Tlwre ls no third of mt::iosis. rib cannot be heart-shaped body. (di lat inn) par;1sympntl1et!c innervation (ron-
386 .., Sample Exam Answer Key
Sample Exani Answer Key 'f 387

striction). The pyloric sphincter in the stomach 387. B. A motor unit is composed of a single motor neu* anterior wall is covered by pectoralis major 399. D. Ora! epithelium is composed ot nonkernt!nizcd,
is regulated primarily by the enteric nervous sys~ ron and the muscle fibers lt innervates, Since
tem and regul.tilory hormones in the GI tracts
and pectoralis minor. Latissimus dorsi con- :stra!iflect, squamous epithelium.
the motor neuron will stimulate all muscle tributes to the inferior aspect of the posterior 400. D. Chondroitin sulfate is a gluco!'>aminoglycan
376. E. The mitochondria is surrounded by a double fibers it innervates, ""fractions" of a motor unit wall. found ln !lgaments, tendons, and cartilage. It is
(inner and out.er) membrane. The nuclear mem- cannot be stimulated. The number of motor 398. C. Although some CO., is transported unchanged in the most abundant glucosaminoglycan in the
brane (not listed), v,1hich surrounds the nucleus, units recruited. the number of muscle fibers the plasma and in- the red biood cells as car~ body.
aiso consists of a double (inner and outer) contracting, and the frequency of stimulation
membrane. baminohemoglobin, most is transported in the
wm all affe-c:t the degree of muscle tension form of bic1.1rbon~te ion in the plasma.
377. A. Calmodulin is a calcium-binding protein in produced.
smooth muscle which, when bound to myosin, 38-8. E. In the Ever, smooth endopla~mic reticulum Is
initiates contraction, Calmoduiln activates involved in glycogen metaboH,rn and detoxifi-
myosin kinase, which results in myosin-actln cation of various drug~ and alcohols; it there~
crosslinking and contraction of smooth muscle. fore, contains P450 enzymes, which are
378. A. The fumen of the gastrointestinal tract ls lined cytochromes that are important in the detoxifi-
with mucosa. The rest of the choices are in cation process,
order from ]umen out. Ffbrosa and adventitia 389. C. Cardiac output is the volume of blood pumped
are synonymous. per minute by each ventricle. This is influenced
379, B. DB developmental groove is not found on by cardiac rate and stroke volume (end-diM-
mandibular second mot.a.rs. tolic volume - endMsystolic volume). The car-
380. E. Bacteria that are phagocytosed by macrophage, diac output for this patient is 70 x I 10 mL or
are kept in membrane-bound vacuoles called 7700 ml per minute,
phagosome.s. A phagosome will iuse with a lyso- 390. B. Mandibular first premolar is most likely to have
some, which contains many de.gradative a gingle pulp horn.
enzymes, including lysozyme. 391. E. Dipicolinic acid is only found in spores. Since
381. B. During metaphase, mitotic spindles form. C1o.stridiom is the only spore~lorming microor-
Chromosomes attach to these spin-dies, with ganism listed, the correct answer is E.
their centmrneres aligned with the equator of 392. D. RNA polymerase is an enzyme that transcribes
the cell. DNA Into RNA chains. Synthesis Is similar to
382. A. DNA ligase is required to ligate the fragments ONA reproduction since it 1s synthesized 5' ➔3~.
together, DNA and RNA polymeraiea are c-ata~ 393. C. The frontal bone of the skull is lormecl by
lysts that synthesize DNA and RNA in a continu- intramembranot1s ossification. The humerus,
ous process. Reverse. transcriptase is an vertebrae, ribs, and clavicle all are formed by
enzyme found In viruses that makes DNA by endochon-dral ossification.
using viral RNA 394. B, The sum total of the pressures moving fluid
383. D. The maxillary nerve (CN V2) exits the skull out of the capillary is P" (37 mmHg) and rc11
through the foramen rotundum. It then passes (1 mmHg) at the arteriolar end and P, (16
through the pterygopalatine fossa, where it mmH&J and >:11 (I mmHg) at the venular encl, The
communicates with the pterygopalatine gan- sum total of the pressure~ moving into the
glion. Contents of the superior orbital fissure cap!llary is P., (0 mmHg) and rr, (26 mmHg} The
include CN III, IV, VJ and VI and the ophthalmic net exchange pressure on the arteriolar end
veins. CN VU and VIII pass through the internal leads to ultraflltratlon (12 mmHg), On the venu-
acoustic meatus and CN V3 (the mandibular lar end, reabsorption occurs {9 mmHg). Overall
nerve) passes through the foramen ovale. The net exchange results in 3 mmHg of [luid loss out
foramen spinosum is not associated with any ol the capillary.
cranial nerves: it contains the middle meningeal 395. B. Thyroxine and triiodothyronine are formed by
vessels. the cleavage of thyroilobulin following stimu-
384, D..A.sthma is an obstructive Jung disease caused lation by TSH. Most thyrm::ine is converted
by narrowing of the airways, Common symp- to triiodothyronine in liver and kidney.
toms include dyspnea, wheezing on explraUon, Thyroglobulin is the storage form of thyroid
and a dry cough. hormone, TSH {thvroid-~tlmulating hormone) is
385. A. Greater MD diameter relative to crown height produced in th"'e ~nterior pituitary. It is not a
than permanent teeth, B, Squat appe,inmce, not hormone secreted from the thyroid hut acts to
elongated appearance; C, Crowns are a miik'Y stimulate the synthesis and seeretion of thyroid
white. not translucent white: D, There is no root hormones.
trunk. 3%. C. Dust cells, along with heart fail cells. are
386. D. The teres major muscle is a shoulder musde, macrophages that are found ln the lungs. They
however, it is not a rotator muscle. All of the are part of the reticuloendothelial system,
other four listed in this question are rotator cuff 397, C. The lateral thoracic wall of the axiHa is cov-
muscles, ered by th-e serratus anterior muscle. The

You might also like